Inverse, Exponential, and Logarithmic Functions: Not For Sale

You might also like

Download as pdf or txt
Download as pdf or txt
You are on page 1of 92

NOT FOR SALE

4 Inverse, Exponential, and


Logarithmic Functions

The magnitudes of earthquakes,


the loudness of sounds, and the
growth or decay of some populations
are examples of quantities that are
described by exponential functions
and their inverses, logarithmic
functions.

4.1 Inverse Functions

4.2 Exponential Functions

4.3 Logarithmic Functions

Summary Exercises on Inverse,


Exponential, and Logarithmic
Functions

4.4 Evaluating Logarithms


and the Change-of-Base
Theorem
Chapter 4 Quiz

4.5 Exponential and


Logarithmic Equations

4.6 Applications and Models


of Exponential Growth
and Decay

Summary Exercises on
Functions: Domains and
Defining Equations

405
Copyright Pearson. All Rights Reserved.

M04_LHSD7659_12_AIE_C04_pp405-496.indd 405 06/10/15 6:45 PM


NOT FOR SALE
406 Chapter 4  Inverse, Exponential, and Logarithmic Functions

4.1 Inverse Functions

■ One-to-One Functions
One-to-One Functions   Suppose we define the following function F.
■ Inverse Functions
■ Equations of Inverses
F = 51 - 2, 22, 1 - 1, 12, 10, 02, 11, 32, 12, 526
■ An Application of (We have defined F so that each second component is used only once.) We can
Inverse Functions to form another set of ordered pairs from F by interchanging the x- and y-values of
Cryptography each pair in F. We call this set G.
G = 512, - 22, 11, - 12, 10, 02, 13, 12, 15, 226
G is the inverse of F. Function F was defined with each second component
Domain Range
f used only once, so set G will also be a function. (Each first component must
1 6 be used only once.) In order for a function to have an inverse that is also a func-
2 7
3
tion, it must exhibit this one-to-one relationship.
8
4 In a one-to-one function, each x-value corresponds to only one y-value, and
5 9 each y-value corresponds to only one x-value.
Not One-to-One The function ƒ shown in Figure 1 is not one-to-one because the y-value 7 corre-
Figure 1 sponds to two x-values, 2 and 3. That is, the ordered pairs 12, 72 and 13, 72 both
belong to the function. The function ƒ in Figure 2 is one-to-one.

Domain Range One-to-One Function


f 5
1
A function ƒ is a one-to-one function if, for elements a and b in the domain
2 6
3 of ƒ,
7
a 3 b implies ƒ 1 a 2 3 ƒ 1 b 2 .
4 8
That is, different values of the domain correspond to different values of the
One-to-One range.
Figure 2

Using the concept of the contrapositive from the study of logic, the boldface
statement in the preceding box is equivalent to
ƒ1a2 = ƒ1b2 implies a = b.
This means that if two range values are equal, then their corresponding domain
values are equal. We use this statement to show that a function ƒ is one-to-one
in Example 1(a).

Classroom Example 1 Example 1 Deciding Whether Functions Are One-to-One


Determine whether each function
is one-to-one. Determine whether each function is one-to-one.
(a) ƒ1x2 = - 3x + 7 (a) ƒ1x2 = - 4x + 12 (b)  ƒ1x2 = 225 - x 2
(b) ƒ1x2 = 249 - x 2
Solution
Answers:
(a) We can determine that the function ƒ1x2 = - 4x + 12 is one-to-one by
(a) It is one-to-one.
showing that ƒ1a2 = ƒ1b2 leads to the result a = b.
(b) It is not one-to-one.
ƒ1a2 = ƒ1b2
- 4a + 12 = - 4b + 12  ƒ1x2 = -4x + 12
- 4a = - 4b   Subtract 12.
a=b   Divide by -4.
By the definition, ƒ1x2 = - 4x + 12 is one-to-one.
Copyright Pearson. All Rights Reserved.

M04_LHSD7659_12_AIE_C04_pp405-496.indd 406 06/10/15 6:45 PM


NOT FOR SALE
Inverse Functions
4.1  407

(b) We can determine that the function ƒ1x2 = 225 - x 2 is not one-to-one by
showing that different values of the domain correspond to the same value of
the range. If we choose a = 3 and b = - 3, then 3 ≠ - 3, but

ƒ132 = 225 - 32 = 225 - 9 = 216 = 4


and ƒ1 - 32 = 225 - 1 - 322 = 225 - 9 = 4.

Here, even though 3 ≠ - 3, ƒ132 = ƒ1 - 32 = 4. By the definition, ƒ is not a


one-to-one function.

✔ Now Try Exercises 17 and 19.

As illustrated in Example 1(b), a way to show that a function is not one-


to-one is to produce a pair of different domain elements that lead to the same
function value. There is a useful graphical test for this, the horizontal line test.

y Horizontal Line Test

5 f(x) = Ë 25 – x 2 A function is one-to-one if every horizontal line intersects the graph of the
(–3, 4) (3, 4) function at most once.

x
0
–5 5
Note In Example 1(b), the graph of the function is a semicircle, as
shown in Figure 3. Because there is at least one horizontal line that inter-
sects the graph in more than one point, this function is not one-to-one.
Figure 3

Classroom Example 2 Example 2 Using the Horizontal Line Test


Determine whether each graph is
the graph of a one-to-one function. Determine whether each graph is the graph of a one-to-one function.
(a) y (a) y (b)  y

4
y1
(x1, y1 ) (x2 , y1 ) (x 3, y1 )
x
–2 0 2 y1 y2
0 x3
x x
–4 x1 x2 x3 x1 x 2 0
y3
(b) y

4
Solution
–2 2
x
0
(a) Each point where the horizontal line intersects the graph has the same value
of y but a different value of x. Because more than one different value of x
(here three) lead to the same value of y, the function is not one-to-one.
Answers:
(a) It is one-to-one.
(b) Every horizontal line will intersect the graph at exactly one point, so this
(b) It is not one-to-one.
function is one-to-one.

✔ Now Try Exercises 11 and 13.

The function graphed in Example 2(b) decreases on its entire domain.


In general, a function that is either increasing or decreasing on its
1
entire domain, such as ƒ 1 x 2 = − x, g 1 x 2 = x3, and h1 x2 = x , must
be one-to-one.
Copyright Pearson. All Rights Reserved.

M04_LHSD7659_12_AIE_C04_pp405-496.indd 407 06/10/15 6:45 PM


NOT FOR SALE
408 Chapter 4  Inverse, Exponential, and Logarithmic Functions

Tests to Determine Whether a Function Is One-to-One

1. Show that ƒ1a2 = ƒ1b2 implies a = b. This means that ƒ is one-to-one.


(See Example 1(a).)
2. In a one-to-one function, every y-value corresponds to no more than
one x-value. To show that a function is not one-to-one, find at least two
x-values that produce the same y-value. (See Example 1(b).)
3. Sketch the graph and use the horizontal line test. (See Example 2.)
4. If the function either increases or decreases on its entire domain, then it
is one-to-one. A sketch is helpful here, too. (See Example 2(b).)

Teaching Tip  This is a good Inverse Functions   Certain pairs of one-to-one functions “undo” each
place to review composition of other. For example, consider the functions
functions.
1 5
g1x2 = 8x + 5 and ƒ1x2 = x- .
8 8
We choose an arbitrary element from the domain of g, say 10. Evaluate g1102.
g1x2 = 8x + 5   Given function
g1102 = 8 # 10 + 5  Let x = 10.
g1102 = 85   Multiply and then add.
Now, we evaluate ƒ1852.
1 5
ƒ1x2 = x -       Given function
8 8
1 5
ƒ1852 = 1852 -    Let x = 85.
8 8
85 5
ƒ1852 = -    Multiply.
8 8
ƒ1852 = 10    Subtract and then divide.

Starting with 10, we “applied” function g and then “applied” function ƒ to the
result, which returned the number 10. See Figure 4.

Function Function
These functions
10 85 10 contain inverse
g(x) = 8x + 5 f(x) = 1 x – 5 operations that
8 8
“undo” each other.

Figure 4

As further examples, confirm the following.


g132 = 29  and  ƒ1292 = 3
g1 - 52 = - 35 and  ƒ1 - 352 = - 5
g122 = 21  and  ƒ1212 = 2
3 3
ƒ122 = -  and ga - b = 2
8 8
Copyright Pearson. All Rights Reserved.

M04_LHSD7659_12_AIE_C04_pp405-496.indd 408 06/10/15 6:45 PM


NOT FOR SALE
Inverse Functions
4.1  409

1 5
In particular, for the pair of functions g1x2 = 8x + 5 and ƒ1x2 = 8 x - 8 ,

ƒ1g1222 = 2 and g1ƒ1222 = 2.

In fact, for any value of x,

ƒ1g1x22 = x and g1ƒ1x22 = x.

Using the notation for composition of functions, these two equations can be
written as follows.

1ƒ ∘ g21x2 = x and 1g ∘ ƒ21x2 = x   The result is the identity function.

Because the compositions of ƒ and g yield the identity function, they are inverses
of each other.

Inverse Function

Let ƒ be a one-to-one function. Then g is the inverse function of ƒ if

1 ƒ ° g 2 1 x2 = x  for every x in the domain of g,


and 1 g ° ƒ 2 1 x2 = x  for every x in the domain of ƒ.

The condition that f is one-to-one in the definition of inverse function is


essential. Otherwise, g will not define a function.
Classroom Example 3
Let functions ƒ and g be defined
respectively by
Example 3 Determining Whether Two Functions Are Inverses
ƒ1x2 = 2x + 5
and Let functions ƒ and g be defined respectively by
1 3
g1x2 = x - 5. ƒ1x2 = x 3 - 1 and g1x2 = 2 x + 1.
2
Is g the inverse function of ƒ? Is g the inverse function of ƒ?
Answer:  no Solution  As shown in Figure 5, the horizontal line test applied to the graph
indicates that ƒ is one-to-one, so the function has an inverse. Because it is one-
to-one, we now find 1ƒ ∘ g21x2 and 1g ∘ ƒ21x2.
y
1ƒ ∘ g21x2 1g ∘ ƒ21x2
   = ƒ1g1x22    = g1ƒ1x22
3
   = A 2 x + 1B - 1
3 3
x    = 2 1x 3 - 12 + 1
ƒ(x) = x 3 – 1
3 3
   = x + 1 - 1    = 2x
   = x    = x

Figure 5 Since 1ƒ ∘ g21x2 = x and 1g ∘ ƒ21x2 = x, function g is the inverse of function ƒ.



✔ Now Try Exercise 41.

A special notation is used for inverse functions: If g is the inverse of a func-


tion ƒ, then g is written as ƒ−1 (read “ƒ-inverse”).
3
ƒ1x2 = x 3 - 1  has inverse  ƒ -11x2 = 2 x + 1.  See Example 3.
Copyright Pearson. All Rights Reserved.

M04_LHSD7659_12_AIE_C04_pp405-496.indd 409 06/10/15 6:45 PM


NOT FOR SALE
410 Chapter 4  Inverse, Exponential, and Logarithmic Functions

Caution  Do not confuse the − 1 in ƒ−1 with a negative exponent.


1
The symbol ƒ -11x2 represents the inverse function of ƒ, not ƒ1x2 .

Teaching Tip  Remind students By the definition of inverse function, the domain of ƒ is the range of ƒ−1,
that composition of functions and the range of ƒ is the domain of ƒ−1. See Figure 6.
is not commutative. Therefore,
1ƒ ∘ g21x2 and 1g ∘ ƒ21x2 must
both be checked. Use an example X
Y
such as Domain of f f Range of f
ƒ1x2 = x 2 and g1x2 = 2x x y
Range of f – 1
to illustrate the necessity of Domain of f –1
f –1
checking both compositions if
you do not verify the functions
are one-to-one. Figure 6

Example 4 Finding Inverses of One-to-One Functions

Number of Find the inverse of each function that is one-to-one.


Year Hurricanes (a) F = 51 - 2, 12, 1 - 1, 02, 10, 12, 11, 22, 12, 226
2009  3
(b) G = 513, 12, 10, 22, 12, 32, 14, 026
2010 12
2011  7
(c) The table in the margin shows the number of hurricanes recorded in the
North Atlantic during the years 2009–2013. Let ƒ be the function defined in
2012 10
the table, with the years forming the domain and the numbers of hurricanes
2013  2 forming the range.
Source: www.wunderground.com
Solution

(a) Each x-value in F corresponds to just one y-value. However, the y-value 2
Classroom Example 4 corresponds to two x-values, 1 and 2. Also, the y-value 1 corresponds to
Find the inverse of each function both - 2 and 0. Because at least one y-value corresponds to more than one
that is one-to-one. x-value, F is not one-to-one and does not have an inverse.
(a) F = 51- 2, - 82, 1- 1, -12,
10, 02, 11, 12, 12, 826 (b) Every x-value in G corresponds to only one y-value, and every y-value cor-
(b) G = 51- 2, 52, 1- 1, 22, responds to only one x-value, so G is a one-to-one function. The inverse
10, 12, 11, 22, 12, 526 function is found by interchanging the x- and y-values in each ordered pair.
(c) Let h be the function defined G -1 = 511, 32, 12, 02, 13, 22, 10, 426
by the table in Example 4(c)
if the number of hurricanes for Notice how the domain and range of G become the range and domain,
2009 is decreased by 1. respectively, of G -1.
Answers:
(a) F -1 = 51- 8, - 22, 1- 1, -12,
(c) Each x-value in ƒ corresponds to only one y-value, and each y-value corre-
sponds to only one x-value, so ƒ is a one-to-one function. The inverse function
      10, 02, 11, 12, 18, 226
is found by interchanging the x- and y-values in the table.
(b) G is not one-to-one.
(c) h is not one-to-one. ƒ -11x2 = 513, 20092, 112, 20102, 17, 20112, 110, 20122, 12, 201326

The domain and range of ƒ become the range and domain of ƒ -1.

✔ Now Try Exercises 37, 51, and 53.

Equations of Inverses   The inverse of a one-to-one function is found by


interchanging the x- and y-values of each of its ordered pairs. The equation of the
inverse of a function defined by y = ƒ1x2 is found in the same way.
Copyright Pearson. All Rights Reserved.

M04_LHSD7659_12_AIE_C04_pp405-496.indd 410 06/10/15 6:45 PM


NOT FOR SALE
Inverse Functions
4.1  411

Finding the Equation of the Inverse of y = f 1 x 2

For a one-to-one function ƒ defined by an equation y = ƒ1x2, find the


defining equation of the inverse as follows. (If necessary, replace ƒ1x2 with
y first. Any restrictions on x and y should be considered.)
Step 1 Interchange x and y.
Step 2 Solve for y.
Step 3 Replace y with ƒ -11x2.

Classroom Example 5 Example 5 Finding Equations of Inverses


Determine whether each function
defines a one-to-one function. Determine whether each equation defines a one-to-one function. If so, find the
If so, find the equation of the equation of the inverse.
inverse.
(a) ƒ1x2 = 2x + 5 (b)  y = x 2 + 2 (c)  ƒ1x2 = 1x - 223
(a) ƒ1x2 = 0 x 0
(b) g1x2 = 4x - 7 Solution
(c) h1x2 = x 3 + 5 (a) The graph of y = 2x + 5 is a nonhorizontal line, so by the horizontal line
Answers: test, ƒ is a one-to-one function. Find the equation of the inverse as follows.
(a) ƒ is not one-to-one.
ƒ1x2 = 2x + 5 Given function
(b) g is one-to-one;
1 7
g -11x2 = 4 x + 4 . y = 2x + 5 Let y = ƒ1x2.
(c) h is one-to-one; Step 1 x = 2y + 5 Interchange x and y.
3
h-11x2 = 2 x - 5.
Step 2 x - 5 = 2y

*1+)1+(
Subtract 5.

x-5 Divide by 2. Solve for y.


y= Rewrite.
2
1 5 Replace y with ƒ -11x2.
Step 3 ƒ -11x2 = x - 
a - b 1 b
2 2 c = AcBa - c

x - 5 1 5
Teaching Tip  To emphasize Thus, the equation ƒ -11x2 = 2 = 2 x - 2 represents a linear function. In
the relationship between inverse
the function y = 2x + 5, the value of y is found by starting with a value of x,
functions, for ƒ1x2 = 2x + 5 and
x = 3, find multiplying by 2, and adding 5.
x - 5
y = 2 # 3 + 5 = 11. The equation ƒ -11x2 = 2 for the inverse subtracts 5 and then divides
1 5
by 2. An inverse is used to “undo” what a function does to the variable x.
Now, for ƒ -11x2 = 2 x - 2 and
x = 11, find (b) The equation y = x 2 + 2 has a parabola opening up as its graph, so some
1 5 horizontal lines will intersect the graph at two points. For example, both
y= 1112 - = 3. x = 3 and x = - 3 correspond to y = 11. Because of the presence of the
2 2
x 2-term, there are many pairs of x-values that correspond to the same
This shows numerically that when
y-value. This means that the function defined by y = x 2 + 2 is not one-to-
we substitute the original y-value
in ƒ -11x2, we obtain the original one and does not have an inverse.
x-value. Proceeding with the steps for finding the equation of an inverse leads to

y = x2 + 2
x = y 2 + 2   Interchange x and y.
x - 2 = y2    Solve for y.
Remember
both roots.
{ 2x - 2 = y.    Square root property

The last equation shows that there are two y-values for each choice of x
greater than 2, indicating that this is not a function.
Copyright Pearson. All Rights Reserved.

M04_LHSD7659_12_AIE_C04_pp405-496.indd 411 06/10/15 6:45 PM


NOT FOR SALE
412 Chapter 4  Inverse, Exponential, and Logarithmic Functions

y f (x) = (x – 2)3 (c) Figure 7 shows that the horizontal line test assures us that this horizontal
translation of the graph of the cubing function is one-to-one.
8

ƒ1x2 = 1x - 223 Given function

y = 1x - 223 Replace ƒ1x2 with y.

Step 1 x = 1y - 223 Interchange x and y.

*1++)1++(
1 3 3
x Step 2 2x = 21y - 223 Take the cube root on each side.
0 2
3 Solve for y.
2x = y - 2 3 3
2a =a
3
This graph 2x + 2 = y Add 2.
passes the
horizontal 3
line test.
Step 3 ƒ -11x2 = 2x + 2 Replace y with ƒ -11x2. Rewrite.
–8 ■
✔ Now Try Exercises 59(a), 63(a), and 65(a).
Figure 7

Example 6  inding the Equation of the Inverse of a


F
Rational Function

Classroom Example 6 The following rational function is one-to-one. Find its inverse.
The following rational function is
one-to-one. Find its inverse. 2x + 3
ƒ1x2 = , x≠4
- 3x + 1 x-4
ƒ1x2 = , x≠5
x-5
2x + 3
5x + 1 Solution ƒ1x2 = , x ≠ 4 Given function
Answer:  ƒ -11x2 = x + 3 , x-4
x ≠ -3
2x + 3
y= Replace ƒ1x2 with y.
x-4
2y + 3
Step 1 x= , y ≠ 4 Interchange x and y.
y-4

*11++++)++++11(
Step 2 x1y - 42 = 2y + 3 Multiply by y - 4.

xy - 4x = 2y + 3 Distributive property
Pay close

attention here. xy - 2y = 4x + 3 Add 4x and -2y.
Solve for y.
y1x - 22 = 4x + 3 Factor out y.
4x + 3
y= , x ≠ 2 Divide by x - 2.
x-2
In the final line, we give the condition x ≠ 2. (Note that 2 is not in the range of
ƒ, so it is not in the domain of ƒ -1.)

4x + 3
Step 3          ƒ -11x2 = , x ≠ 2 Replace y with ƒ -11x2.
x-2
■✔ Now Try Exercise 71(a).

One way to graph the inverse of a function ƒ whose equation is known follows.

Step 1 Find some ordered pairs that are on the graph of ƒ.


Step 2 Interchange x and y to find ordered pairs that are on the graph of ƒ -1.
Step 3 Plot those points, and sketch the graph of ƒ -1 through them.

Another way is to select points on the graph of ƒ and use symmetry to find
corresponding points on the graph of ƒ -1.
Copyright Pearson. All Rights Reserved.

M04_LHSD7659_12_AIE_C04_pp405-496.indd 412 06/10/15 6:45 PM


NOT FOR SALE
Inverse Functions
4.1  413

y For example, suppose the point 1a, b2 shown in Figure 8 is on the graph of
(a, b) a one-to-one function ƒ. Then the point 1b, a2 is on the graph of ƒ -1. The line
b
segment connecting 1a, b2 and 1b, a2 is perpendicular to, and cut in half by, the
y=x line y = x. The points 1a, b2 and 1b, a2 are “mirror images” of each other with
respect to y = x.
Thus, we can find the graph of ƒ−1 from the graph of ƒ by locating the
(b, a) mirror image of each point in ƒ with respect to the line y = x.
a
x
0
a b

Figure 8 Example 7 Graphing f −1 Given the Graph of f

In each set of axes in Figure 9, the graph of a one-to-one function ƒ is shown in


Classroom Example 7 blue. Graph ƒ -1 in red.
Determine whether functions ƒ Solution In Figure 9, the graphs of two functions ƒ shown in blue are given
and g graphed are inverses of
with their inverses shown in red. In each case, the graph of ƒ -1 is a reflection of
each other.
the graph of ƒ with respect to the line y = x.
y
y = g(x)
y=x y y
f –1 f
5
y=x
x y=x
y = f(x) (1, 3)
(3, 1) 5 (2, 4)
f
(–4, 0)
Answer:  They are not inverses. 0
x
f –1
–2 5
–2 (4, 2)
0 (1, 1) x
(0, –4) –2 5
–2

Figure 9

✔ Now Try Exercises 77 and 81.

Classroom Example 8 Example 8 Finding the Inverse of a Function (Restricted Domain)


Let ƒ1x2 = x 2 + 4, x … 0.
Find ƒ -11x2. Let ƒ1x2 = 2x + 5, x Ú - 5. Find ƒ -11x2.
Answer: Solution  The domain of ƒ is restricted to the interval 3 - 5, ∞2. Function ƒ is
ƒ -11x2 = - 2x - 4, x Ú 4 one-to-one because it is an increasing function and thus has an inverse function.
Now we find the equation of the inverse.

ƒ1x2 = 2x + 5, x Ú - 5 Given function

y = 2x + 5, x Ú - 5 Replace ƒ1x2 with y.

Step 1 x = 2y + 5, y Ú - 5 Interchange x and y.


2
Step 2 x 2 = A 2y + 5 B
*1111)11+1(

Square each side.


2
x2 = y + 5 A 2a B = a for a Ú 0 Solve for y.
y = x2 - 5 Subtract 5. Rewrite.

However, we cannot define ƒ -11x2 as x 2 - 5. The domain of ƒ is 3 - 5, ∞2, and


its range is 30, ∞2. The range of ƒ is the domain of ƒ -1, so ƒ -1 must be defined
as follows.

Step 3 ƒ -11x2 = x 2 - 5, x Ú 0

As a check, the range of ƒ -1, 3 - 5, ∞2, is the domain of ƒ.


Copyright Pearson. All Rights Reserved.

M04_LHSD7659_12_AIE_C04_pp405-496.indd 413 06/10/15 6:45 PM


NOT FOR SALE
414 Chapter 4  Inverse, Exponential, and Logarithmic Functions

Graphs of ƒ and ƒ -1 are shown in Figures 10 and 11. The line y = x


is included on the graphs to show that the graphs of ƒ and ƒ -1 are mirror images
with respect to this line.

y
f(x) =  x + 5, x ≥ − 5
5
f (x) = x + 5,
10
x–5
y=x

x
–5 0 5 −16.1 16.1

y=x

−10
–5
f –1 (x) = x 2 – 5, x  0 f –1(x) = x2 − 5, x ≥ 0
Figure 10 Figure 11


✔ Now Try Exercise 75.

Important Facts about Inverses

1. If ƒ is one-to-one, then ƒ -1 exists.


y= x2 2. The domain of ƒ is the range of ƒ -1, and the range of ƒ is the domain of ƒ -1.
4.1 3. If the point 1a, b2 lies on the graph of ƒ, then 1b, a2 lies on the graph of ƒ -1.
The graphs of ƒ and ƒ -1 are reflections of each other across the line y = x.
4. To find the equation for ƒ -1, replace ƒ1x2 with y, interchange x and y,
−6.6 6.6
and solve for y. This gives ƒ -11x2.

−4.1

x = y2   Some graphing calculators have the capability of “drawing” the reflection


Despite the fact that y = is not
x2 of a graph across the line y = x. This feature does not require that the function be
one-to-one, the calculator will draw its one-to-one, however, so the resulting figure may not be the graph of a function.
“inverse,” x = y 2. See Figure 12. It is necessary to understand the mathematics to interpret
Figure 12 results correctly.  ■

An Application of Inverse Functions to Cryptography   A one-to-one


function and its inverse can be used to make information secure. The function is
used to encode a message, and its inverse is used to decode the coded message.
In practice, complicated functions are used.
Classroom Example 9
The function
Example 9 Using Functions to Encode and Decode a Message
ƒ1x2 = 3x - 1
was used to encode a message as Use the one-to-one function ƒ1x2 = 3x + 1 and the following numerical values
assigned to each letter of the alphabet to encode and decode the message BE
11 44 74 44 62
MY FACEBOOK FRIEND.
59 68 14 14 59
Find the inverse function and A 1 H  8 O 15 V 22
decode the message. (Use the B 2 I  9 P 16 W 23
same numerical values for the C 3 J 10 Q 17 X 24
letters as in Example 9.) D 4 K 11 R 18 Y 25
1 1
E 5 L 12 S 19 Z 26
Answer:  ƒ -11x2 = 3 x + 3 ; F 6 M 13 T 20
DO YOU TWEET G 7 N 14 U 21
Copyright Pearson. All Rights Reserved.

M04_LHSD7659_12_AIE_C04_pp405-496.indd 414 06/10/15 6:45 PM


NOT FOR SALE
Inverse Functions
4.1  415

A  1 N 14 Solution  The message BE MY FACEBOOK FRIEND would be encoded as


B  2 O 15
C  3 P 16   7  16 40 76 19  4 10 16  7
D  4 Q 17 46 46 34 19 55 28 16 43 13
E  5 R 18 because
F  6 S 19
B corresponds to 2  and  ƒ122 = 3122 + 1 = 7,
G  7 T 20
H  8 U 21 E corresponds to 5  and  ƒ152 = 3152 + 1 = 16,  and so on.
I  9 V 22
J 10 W 23 1 1
Using the inverse ƒ -11x2 = 3 x - 3 to decode yields
K 11 X 24
L 12 Y 25 1 1
M 13 Z 26 ƒ -1172 = 172 - = 2,   which corresponds to B,
3 3
1 1
ƒ -11162 = 1162 - = 5,  which corresponds to E,  and so on.
3 3

✔ Now Try Exercise 97.

4.1 Exercises

2.  not one-to-one Concept Preview  Determine whether the function represented in each table is
1. one-to-one  
one-to-one.
3. one-to-one 4.  x; 1g ∘ ƒ21x2
5. range; domain 1. The table shows the number of registered passenger cars in the United States for the
3
6. 1b, a2 7.  2 x years 2008–2012.
8. y = x 9.  - 3
10. does not; it is not one-to-one Registered Passenger Cars
Year (in thousands)
11. one-to-one 12. one-to-one
2008 137,080
13. not one-to-one
14. not one-to-one 2009 134,880
15. one-to-one 16. one-to-one 2010 139,892
17. one-to-one 18. one-to-one
2011 125,657
19. not one-to-one
20. not one-to-one 2012 111,290
21. one-to-one 22. one-to-one Source: U.S. Federal Highway Administration.
23. one-to-one 24. one-to-one
25. not one-to-one 2. The table gives the number of representatives currently in Congress from each of
26. not one-to-one five New England states.
27. one-to-one 28. one-to-one
Number of
29. no 30. no State Representatives
31. untying your shoelaces Connecticut 5
32. stopping a car Maine 2
33. leaving a room
34. descending the stairs
Massachusetts 9
35. unscrewing a light bulb New Hampshire 2
36. emptying a cup Vermont 1
37. inverses 38.  not inverses Source: www.house.gov
39. not inverses 40. inverses
Concept Preview  Fill in the blank(s) to correctly complete each sentence.
41. inverses 42. inverses
43. not inverses 44.  not inverses 3. For a function to have an inverse, it must be .
45. inverses 46. inverses 4. If two functions ƒ and g are inverses, then 1ƒ ∘ g21x2 = and = x.
47. not inverses 48.  not inverses
49. inverses 50. inverses 5. The domain of ƒ is equal to the of ƒ -1, and the range of ƒ is equal to the
of ƒ -1.
51. 516, - 32, 11, 22, 18, 526
6. If the point 1a, b2 lies on the graph of ƒ, and ƒ has an inverse, then the point
52. E 1- 1, 32, 10, 52, 15, 02, A 3 , 4 B F
2
lies on the graph of ƒ -1.
Copyright Pearson. All Rights Reserved.

M04_LHSD7659_12_AIE_C04_pp405-496.indd 415 06/10/15 6:45 PM


NOT FOR SALE
416 Chapter 4  Inverse, Exponential, and Logarithmic Functions

53. not one-to-one 7. If ƒ1x2 = x 3, then ƒ -11x2 = .


54. not one-to-one
8. If a function ƒ has an inverse, then the graph of ƒ -1 may be obtained by reflecting
55. inverses 56. inverses the graph of ƒ across the line with equation .
57. not inverses 58.  not inverses
9. If a function ƒ has an inverse and ƒ1 - 32 = 6, then ƒ -1162 = .
1 4
59. (a) ƒ -11x2 = 3x + 3 10. If ƒ1 - 42 = 16 and ƒ142 = 16, then ƒ have an inverse because
(b) y . (does/does not)
ƒ
2
ƒ –1 Determine whether each function graphed or defined is one-to-one. See Examples 1
x
–4 0 2 and 2.
–4 11. y 12. y
(c) Domains and ranges of both
ƒ and ƒ -1 are 1- ∞, ∞2.
1 5
60. (a) ƒ -11x2 = 4 x + 4 x x
0 0
(b) y

f –1
x
–5 0
13. y 14. y
–5

(c) Domains and ranges of both


ƒ and ƒ -1 are 1- ∞, ∞2.
1 3 x x
61. (a) ƒ -11x2 = - 4 x + 0 0
4
(b) y

–1
f
x 15. y 16. y
–2 0 3

(c) Domains and ranges of both


ƒ and ƒ -1 are 1- ∞, ∞2. 0
x x
0
1 4
62. (a) ƒ -11x2 = - 6x - 3
(b) y
ƒ 4
4
0 x 17. y = 2x - 8 18. y = 4x + 20 19. y = 236 - x 2
–8
–1
ƒ
20. y = - 2100 - x 2 21. y = 2x 3 - 1 22. y = 3x 3 - 6
–8

(c) Domains and ranges of both -1 4


ƒ and ƒ -1 are 1- ∞, ∞2.
23. y = 24. y = 25. y = 21x + 122 - 6
x+2 x-8
3
63. (a) ƒ -11x2 = 2 x-1 3 3
(b) y
26. y = - 31x - 622 + 8 27. y = 2x + 1 - 3 28. y = - 2x + 2 - 8
ƒ

ƒ –1 Concept Check  Answer each question.


x
0 29. Can a constant function, such as ƒ1x2 = 3, defined over the set of real numbers, be
one-to-one?
(c) Domains and ranges of both 30. Can a polynomial function of even degree defined over the set of real numbers have
ƒ and ƒ -1 are 1- ∞, ∞2. an inverse?
3
64. (a) ƒ -11x2 = 2 -x - 2
(b) y Concept Check  An everyday activity is described. Keeping in mind that an inverse
f operation “undoes” what an operation does, describe each inverse activity.
f –1
31. tying your shoelaces 32. starting a car
x
–2 0

–2
33. entering a room 34. climbing the stairs

(c) Domains and ranges of both 35. screwing in a light bulb 36. filling a cup
ƒ and ƒ -1 are 1- ∞, ∞2.
Copyright Pearson. All Rights Reserved.

M04_LHSD7659_12_AIE_C04_pp405-496.indd 416 06/10/15 6:45 PM


NOT FOR SALE
Inverse Functions
4.1  417

65. not one-to-one Determine whether the given functions are inverses. See Example 4.
66. not one-to-one
1 37. x ƒ1x2 x g1x2 38. x ƒ1x2 x g1 x2
67. (a) ƒ -11x2 = x , x ≠ 0
(b) y 3 -4 -4 3 -2 -8  8 -2
2 -6 -6 2 -1 -1  1 -1
5  8  8 5  0  0  0  0
1 1  9  9 1  1  1 -1  1
x
0 1 4  3  3 4  2  8 -8  2
ƒ = ƒ –1

39. ƒ = 512, 52, 13, 52, 14, 526; g = 515, 226


(c) 
Domains and ranges
of both ƒ and ƒ -1 are 40. ƒ = 511, 12, 13, 32, 15, 526; g = 511, 12, 13, 32, 15, 526
1- ∞, 02 ´ 10, ∞2.
4
68. (a) ƒ -11x2 = x , x ≠ 0
(b) y Use the definition of inverses to determine whether f and g are inverses. See Example 3.
1 1
2 41. ƒ1x2 = 2x + 4, g1x2 = x - 2 42. ƒ1x2 = 3x + 9, g1x2 = x-3
2 3
x
0 2
f = f –1 1 1
43. ƒ1x2 = - 3x + 12, g1x2 = - x - 12 44. ƒ1x2 = - 4x + 2, g1x2 = - x - 2
3 4
(c) 
Domains and ranges
of both ƒ and ƒ -1 are x+1 2x + 1 x-3 4x + 3
45. ƒ1x2 = , g1x2 = 46. ƒ1x2 = , g1x2 =
1- ∞, 02 ´ 10, ∞2. x-2 x-1 x+4 1-x
1 + 3x
69. (a) ƒ -11x2 = x , x≠0 2 6x + 2 -1 1-x
(b) 47. ƒ1x2 = , g1x2 = 48. ƒ1x2 = , g1x2 =
y
ƒ x+6 x x+1 x
ƒ –1 ƒ –1
49. ƒ1x2 = x 2 + 3, x Ú 0; g1x2 = 2x - 3, xÚ3
0 x
50. ƒ1x2 = 2x + 8, x Ú - 8; g1x2 = x 2 - 8, xÚ0
ƒ

Domain of ƒ = range of
(c) 
ƒ -1 = 1- ∞, 32 ´ 13, ∞2.
Domain of ƒ -1 = range of Find the inverse of each function that is one-to-one. See Example 4.
ƒ = 1-∞, 02 ´ 10, ∞2. 2
1 - 2x 51. 51 - 3, 62, 12, 12, 15, 826 52. e 13, - 12, 15, 02, 10, 52, a4, b f
70. (a) ƒ -11x2 = x , x≠0 3
(b) y
53. 511, - 32, 12, - 72, 14, - 32, 15, - 526 54. 516, - 82, 13, - 42, 10, - 82, 15, - 426
ƒ

x
0
–1 ƒ –1
ƒ
ƒ
Determine whether each pair of functions graphed are inverses. See Example 7.
Domain of ƒ = range
(c)  55. y 56. y
of ƒ -1 = 1- ∞, - 22 ´
1- 2, ∞2. 4 y=x 4
 Domain of ƒ -1 = range of 3
ƒ = 1-∞, 02 ´ 10, ∞2.
3x + 1
71. (a) ƒ -11x2 = x - 1 , x≠1 0
x
0
x
3 4 y=x 4
(b) y

5
ƒ –1
ƒ –1 ƒ 57. y 58. y
0 x
–1 3

y=x y=x
ƒ

Domain of ƒ = range of
(c)  2
x x
ƒ -1 = 1- ∞, 32 ´ 13, ∞2. 0
2 2
0
2
Domain of ƒ -1 = range of
ƒ = 1-∞, 12 ´ 11, ∞2.

Copyright Pearson. All Rights Reserved.

M04_LHSD7659_12_AIE_C04_pp405-496.indd 417 06/10/15 6:45 PM


NOT FOR SALE
418 Chapter 4  Inverse, Exponential, and Logarithmic Functions

72. (a) ƒ -11x2 =


x + 2
x≠1 For each function that is one-to-one, (a) write an equation for the inverse function,
x - 1,
(b) y
(b) graph ƒ and ƒ -1 on the same axes, and (c) give the domain and range of both f and
ƒ -1. If the function is not one-to-one, say so. See Examples 5–8.
ƒ = ƒ –1
4
2 59. ƒ1x2 = 3x - 4 60. ƒ1x2 = 4x - 5 61. ƒ1x2 = - 4x + 3
0 x
–2 2 62. ƒ1x2 = - 6x - 8 63. ƒ1x2 = x3 + 1 64. ƒ1x2 = - x 3 - 2
–1
ƒ=ƒ
1
65. ƒ1x2 = x 2 + 8 66. ƒ1x2 = - x 2 + 2 67. ƒ1x2 = , x≠0
Domain of ƒ = range of
(c)  x
ƒ -1 = 1- ∞, 12 ´ 11, ∞2.
4 1 1
Domain of ƒ -1 = range of 68. ƒ1x2 = , x ≠ 0 69. ƒ1x2 = , x ≠ 3 70. ƒ1x2 = , x ≠ -2
ƒ = 1-∞, 12 ´ 11, ∞2. x x-3 x+2

73. (a) ƒ -11x2 =


3x + 6
x≠2 x+1 x+2
x - 2 , 71. ƒ1x2 = , x ≠ 3 72. ƒ1x2 = , x≠1
(b) y x-3 x-1
2x + 6 - 3x + 12
ƒ –1
73. ƒ1x2 = , x ≠ 3 74. ƒ1x2 = , x≠6
ƒ –1
x-3 x-6
ƒ
2
x
3 75. ƒ1x2 = 2x + 6, x Ú - 6 76. ƒ1x2 = - 2x 2 - 16, x Ú 4
ƒ
Graph the inverse of each one-to-one function. See Example 7.
Domain of ƒ = range of
(c) 
ƒ -1 = 1- ∞, 32 ´ 13, ∞2. 77. y 78. y 79. y
Domain of ƒ -1 = range of
ƒ = 1-∞, 22 ´ 12, ∞2.
6x + 12
74. (a) ƒ -11x2 = x + 3 , x ≠ -3
(b) ƒ –1
y ƒ x x x
0 0 0

6
4 ƒ –1
x
3 0 4 6
3 ƒ
80. y 81. y 82. y

Domain of ƒ = range of
(c) 
ƒ -1 = 1- ∞, 62 ´ 16, ∞2.
Domain of ƒ -1 = range of x x x
ƒ = 1-∞, - 32 ´ 1-3, ∞2. 0 0 0

75. (a) ƒ -11x2 = x 2 - 6, x Ú 0


(b) y
–1
ƒ

ƒ
x Concept Check  The graph of a function ƒ is shown in the figure. Use the graph to find
–6 0
each value. y

–6 83. ƒ -1142 84. ƒ -1122 4

Domain of ƒ = range of
(c) 
85. ƒ -1102 86. ƒ -11 - 22 2
ƒ -1 = 3 -6, ∞2.
Domain of ƒ -1 = range of 87. ƒ -11 - 32 88. ƒ -11 - 42 –4 –2 0 2 4
x
ƒ = 30, ∞2.
–2
76. (a) ƒ -11x2 = 2x 2 + 16,
–4
x…0
(b) y
f –1 Concept Check  Answer each of the following.
4
89. Suppose ƒ1x2 is the number of cars that can be built for x dollars. What does
4
0
x ƒ -1110002 represent?
f
90. Suppose ƒ1r2 is the volume (in cubic inches) of a sphere of radius r inches. What
Domain of ƒ = range of
(c)  does ƒ -1152 represent?
ƒ -1 = 34, ∞2. 91. If a line has slope a, what is the slope of its reflection across the line y = x?
Domain of ƒ -1 = range of
ƒ = 1-∞, 04. 92. For a one-to-one function ƒ, find 1ƒ -1 ∘ ƒ2122, where ƒ122 = 3.

Copyright Pearson. All Rights Reserved.

M04_LHSD7659_12_AIE_C04_pp405-496.indd 418 06/10/15 6:45 PM


NOT FOR SALE
Exponential Functions
4.2  419

77. 78.
y y Use a graphing calculator to graph each function defined as follows, using the given
ƒ viewing window. Use the graph to decide which functions are one-to-one. If a function is
ƒ –1 ƒ one-to-one, give the equation of its inverse.
x x
y=x 0 0
y=x
ƒ –1
93. ƒ1x2 = 6x 3 + 11x 2 - 6; 94. ƒ1x2 = x 4 - 5x 2;
3 - 3, 24 by 3 - 10, 104 3 - 3, 34 by 3 - 8, 84
79. 80.
y y

ƒ x-5 -x
95. ƒ1x2 = , x ≠ - 3; 96. ƒ1x2 = , x ≠ 4;
–1 ƒ –1 x+3 x-4
ƒ y=x
y=x x x
0 0
ƒ 3 - 8, 84 by 3 - 6, 84 3 - 1, 84 by 3 - 6, 64

81. 82.
y y Use the following alphabet coding assignment to work each problem. See Example 9.
ƒ
ƒ –1 A 1 H  8 O 15 V 22
y=x
x x B 2 I  9 P 16 W 23
0 0
ƒ C 3 J 10 Q 17 X 24
y=x ƒ –1 D 4 K 11 R 18 Y 25
E 5 L 12 S 19 Z 26
83. 4 84. 3 85. 2 F 6 M 13 T 20
86. 0 87.  - 2 88.  - 4 G 7 N 14 U 21
97. The function ƒ1x2 = 3x - 2 was used to encode a message as
89. It represents the cost, in
dollars, of building 1000 cars. 37 25 19 61 13 34 22 1 55 1 52 52 25 64 13 10.
90. It represents the radius of a
sphere with volume 5 in.3. Find the inverse function and determine the message.
1
91. a 92. 2
98. The function ƒ1x2 = 2x - 9 was used to encode a message as
93. not one-to-one
- 5 9 5 5 9 27 15 29  - 1 21 19 31  - 3 27 41.
94. not one-to-one
95. one-to-one; Find the inverse function and determine the message.
- 5 - 3x
ƒ -11x2 = x - 1 , x ≠ 1
99. Encode the message SEND HELP, using the one-to-one function
96. one-to-one;
4x
ƒ -11x2 = x + 1 ,  x ≠ - 1 ƒ1x2 = x 3 - 1.
1 2
97. ƒ -11x2 = 3 x + 3 ; Give the inverse function that the decoder will need when the message is received.
MIGUEL HAS ARRIVED
1 9
100. Encode the message SAILOR BEWARE, using the one-to-one function
98. ƒ -11x2 = 2 x + 2 ;
BIG GIRLS DONT CRY ƒ1x2 = 1x + 123.
99. 6858 124 2743 63
511 124 1727 4095; Give the inverse function that the decoder will need when the message is received.
3
ƒ -11x2 = 2x + 1
100. 8000 8 1000 2197
4096 6859 27 216
13824 8 6859 216;
3
ƒ -11x2 = 2x - 1

4.2 Exponential Functions

■ Exponents and
Exponents and Properties   Recall the definition of a m/n: If a is a real
Properties n
■ Exponential Functions
number, m is an integer, n is a positive integer, and 2a is a real number, then
■ Exponential Equations m
am/n = A !a B
n
.
■ Compound Interest
■ The Number e 3
and Continuous For example, 16 3/4 = A 2
4
16 B = 23 = 8,
Compounding
■ Exponential Models 1 1 1 1 1 1
27-1/3 = 1/3
= 3 = , and 64-1/2 = 1/2 = = .
27 227 3 64 264 8
Copyright Pearson. All Rights Reserved.

M04_LHSD7659_12_AIE_C04_pp405-496.indd 419 06/10/15 6:45 PM


NOT FOR SALE
420 Chapter 4  Inverse, Exponential, and Logarithmic Functions

Teaching Tip  Use this opportu- In this section, we extend the definition of a r to include all real (not just
nity to emphasize the distinction rational) values of the exponent r. Consider the graphs of y = 2x for different
between exact and approximate domains in Figure 13.
values.
y y y

8 8
8

6 6 6

4 4 4
23
2 2 2
2
x x x
–2 2 –2 2 –2 3

y = 2 x; y = 2 x; y = 2 x;
integers as domain selected rational numbers real numbers
    as domain     as domain

Figure 13

The equations that use just integers or selected rational numbers as domain in
Figure 13 leave holes in the graphs. In order for the graph to be continuous, we
must extend the domain to include irrational numbers such as 23. We might
evaluate 223 by approximating the exponent with the rational numbers 1.7,
1.73, 1.732, and so on. Because these values approach the value of 23 more
and more closely, it is reasonable that 223 should be approximated more and
more closely by the numbers 21.7, 21.73, 21.732, and so on. These expressions can
be evaluated using rational exponents as follows.
10 17
21.7 = 217/10 = Q 2 2R ≈ 3.249009585

Because any irrational number may be approximated more and more closely
using rational numbers, we can extend the definition of a r to include all real
number exponents and apply all previous theorems for exponents. In addition
to the rules for exponents presented earlier, we use several new properties in
this chapter.

Additional Properties of Exponents

For any real number a 7 0, a ≠ 1, the following statements hold.


Property Description
(a) ax is a unique real number y = a x can be considered a function
for all real numbers x. ƒ1x2 = a x with domain 1 - ∞, ∞2.
(b) ab = ac if and only if b = c. The function ƒ1x2 = a x is one-to-one.
(c) If a + 1 and m * n, then Example:  23 6 24  1a 7 12
am * an. Increasing the exponent leads to
a greater number. The function
ƒ1x2 = 2x is an increasing function.
2 3
(d) If 0 * a * 1 and m * n, Example:  A 12 B 7 A 12 B   10 6 a 6 12
then am + an.
Increasing the exponent leads to
a lesser number. The function
x
ƒ1x2 = A 12 B is a decreasing function.

Copyright Pearson. All Rights Reserved.

M04_LHSD7659_12_AIE_C04_pp405-496.indd 420 06/10/15 6:45 PM


NOT FOR SALE
Exponential Functions
4.2  421

Exponential Functions   We now define a function ƒ1x2 = a x whose


domain is the set of all real numbers. Notice how the independent variable x
appears in the exponent in this function. In earlier chapters, this was not
the case.

Exponential Function

If a 7 0 and a ≠ 1, then the exponential function with base a is

ƒ 1 x 2 = a x.

Note  The restrictions on a in the definition of an exponential function


are important. Consider the outcome of breaking each restriction.
1 1
If a 6 0, say a = - 2, and we let x = 2 , then ƒ A 2 B = 1 - 221/2 = 2 - 2,
which is not a real number.
If a = 1, then the function becomes the constant function ƒ1x2 = 1x = 1,
which is not an exponential function.

Classroom Example 1 Example 1 Evaluating an Exponential Function


For ƒ1x2 = 4x, find each of the
following. For ƒ1x2 = 2x, find each of the following.
(a) ƒ1- 22 (b)  ƒ152 5
(a) ƒ1 - 12 (b)  ƒ132 (c)  ƒ a b (d)  ƒ14.922
2
(c) ƒa b (d)  ƒ12.152
2
3
Solution
Answers:
1
(a) 16 (b) 1024 3
(c)  22 2
1
(a) ƒ1 - 12 = 2-1 =    Replace x with -1. (b)  ƒ132 = 23 = 8
2
(d) 19.69831061

(c) ƒ a b = 25/2 = 12521/2 = 321/2 = 232 = 216 # 2 = 4 22


5
2

(d) ƒ14.922 = 24.92 ≈ 30.2738447  Use a calculator.



✔ Now Try Exercises 13, 19, and 23.

We repeat the final graph of y = 2x (with real num- y


bers as domain) from Figure 13 and summarize impor- f(x) = 2 x
8
tant details of the function ƒ1x2 = 2x here.
• The y-intercept is 10, 12. 6

• Because 2x 7 0 for all x and 2x S 0 as x S - ∞, 4 (2, 4)


the x-axis is a horizontal asymptote.
2 (1, 2)
Q–1, 1R
• As the graph suggests, the domain of the function is 2 (0, 1)
1 - ∞, ∞2 and the range is 10, ∞2. –2
x
2

• The function is increasing on its entire domain. Graph of ƒ 1x 2 = 2x with


Therefore, it is one-to-one. domain 1 −H, H2
These observations lead to the following generaliza- Figure 13
tions about the graphs of exponential functions. (repeated)

Copyright Pearson. All Rights Reserved.

M04_LHSD7659_12_AIE_C04_pp405-496.indd 421 06/10/15 6:45 PM


NOT FOR SALE
422 Chapter 4  Inverse, Exponential, and Logarithmic Functions

Exponential Function  f 1 x 2 = ax
Domain: 1 - ∞, ∞2  Range: 10, ∞2
For ƒ1x2 = 2x:
f(x) = ax, a > 1
x
x ƒ1x2 y
f (x) = a , a > 1

-2 1
4
-1 1
(1, a)
0
2
1 (–1, 1a) (0, 1)
1 2 x
0
2 4
3 8
This is the general behavior seen on
a calculator graph for any base a,
for a + 1.
Figure 14

• ƒ 1 x 2= a x, for a + 1, is increasing and continuous on its entire domain,


1 - ∞, ∞2.
• The x-axis is a horizontal asymptote as x S - ∞.
• The graph passes through the points A - 1, 1a B , 10, 12, and 11, a2.
x
For ƒ1x2 = A 12 B : f(x) = ax, 0 < a < 1
y
x ƒ1x2

-3 8 f (x) = a x, 0 < a < 1


-2
-1
4
2
(–1, 1a)
(1, a)
(0, 1)
0 1 x
1 0
1 2
1
2 4

This is the general behavior seen on


a calculator graph for any base a,
for 0 * a * 1.
Figure 15

• ƒ 1 x 2
= a x,
for 0 * a * 1, is decreasing and continuous on its entire
domain, 1 - ∞, ∞2.
• The x-axis is a horizontal asymptote as x S ∞.
• The graph passes through the points A - 1, 1a B , 10, 12, and 11, a2.

Recall that the graph of y = ƒ1 - x2 is the graph of y = ƒ1x2 reflected across


the y-axis. Thus, we have the following.
1 x
If ƒ1x2 = 2x, then ƒ1 - x2 = 2-x = 2-1 # x = 12-12x = a b .
2
This is supported by the graphs in Figures 14 and 15.
The graph of ƒ1x2 = 2x is typical of graphs of ƒ1x2 = a x where a 7 1. For
larger values of a, the graphs rise more steeply, but the general shape is similar
to the graph in Figure 14. When 0 6 a 6 1, the graph decreases in a manner
1 x
similar to the graph of ƒ1x2 = A 2 B in Figure 15.
Copyright Pearson. All Rights Reserved.

M04_LHSD7659_12_AIE_C04_pp405-496.indd 422 06/10/15 6:45 PM


NOT FOR SALE
Exponential Functions
4.2  423

In Figure 16, the graphs of several typical exponential functions illustrate


these facts.
y

(1)
x
f(x) = 10 5 f(x) = 10x
Teaching Tip  The point 10, 12 For 0 < a < 1, For a > 1,
lies on the graph of the function
is decreasing.
f(x) = 3()
1 x
4
f(x) = 3x the function
f(x) = 2x is increasing.
ƒ1x2 = ax f(x) = 2 ()
1 x
3
for all a. We can informally
consider it “home plate” when
graphing translations and
reflections, as seen later in The x-axis is
Example 3. x a horizontal
–3 –2 –1 0 1 2 3 asymptote.
f(x) = ax
Domain: (–∞, ∞); Range: (0, ∞)

Figure 16

In summary, the graph of a function of the form ƒ1x2 = a x has the following
features.

Teaching Tip Encourage
students to learn the characteristics Characteristics of the Graph of f 1 x 2 = a x
of the graph of
1
1. The points A - 1, a B , 10, 12, and 11, a2 are on the graph.
ƒ1x2 = ax.
Ask them how the coordinates 2. If a 7 1, then ƒ is an increasing function.
10, 12 and 11, a2 are affected by If 0 6 a 6 1, then ƒ is a decreasing function.
different variations of the graph of
3. The x-axis is a horizontal asymptote.
ƒ1x2 = -ax - h + k.
4. The domain is 1 - ∞, ∞2, and the range is 10, ∞2.

Classroom Example 2 Example 2 Graphing an Exponential Function


1 x x
Graph ƒ1x2 = a b . Give the
2 Graph ƒ1x2 = A 15 B . Give the domain and range.
domain and range.
Solution The y-intercept is 10, 12, and the x-axis is a horizontal asymptote.
Answer:  y Plot a few ordered pairs, and draw a smooth curve through them as shown in
Figure 17.
x

4
(2)
f (x)  1
x ƒ1x2 y

1
x -2 25 (–2, 25) 25
–4 –2 0 2 4 -1  5
( 1)
x
20 f(x) = 5
1- ∞, ∞2; 10, ∞2 0  1
1 15
1 5 10
2
1
25 (–1, 5) 5
(0, 1)
(1, 15 )
x
–2 –1 0 1 2

Figure 17

This function has domain 1 - ∞, ∞2, range 10, ∞2, and is one-to-one. It is
decreasing on its entire domain.

✔ Now Try Exercise 29.
Copyright Pearson. All Rights Reserved.

M04_LHSD7659_12_AIE_C04_pp405-496.indd 423 07/10/15 7:30 AM


NOT FOR SALE
424 Chapter 4  Inverse, Exponential, and Logarithmic Functions

Classroom Example 3 Example 3 Graphing Reflections and Translations


Graph each function. Give the
domain and range. Graph each function. Show the graph of y = 2x for comparison. Give the domain
(a) ƒ1x2 = - 3x  (b)  ƒ1x2 = 3x - 2 and range.
(c) ƒ1x2 = 3x + 2 - 2 (a) ƒ1x2 = - 2x (b)  ƒ1x2 = 2x + 3 (c)  ƒ1x2 = 2x - 2 - 1
Answers: Solution  In each graph, we show in particular how the point 10, 12 on the
(a) y
x graph of y = 2x has been translated.
f (x) = –3
0
–1 2
x (a) The graph of ƒ1x2 = - 2x is that of ƒ1x2 = 2x reflected across the x-axis. See
Figure 18. The domain is 1 - ∞, ∞2, and the range is 1 - ∞, 02.
–5
(b) The graph of ƒ1x2 = 2x + 3 is the graph of ƒ1x2 = 2x translated 3 units to the
left, as shown in Figure 19. The domain is 1 - ∞, ∞2, and the range is 10, ∞2.

1- ∞, ∞2; 1- ∞, 02 (c) The graph of ƒ1x2 = 2x - 2 - 1 is that of ƒ1x2 = 2x translated 2 units to the
(b) y
right and 1 unit down. See Figure 20. The domain is 1 - ∞, ∞2, and the range
f (x) = 3 x – 2 is 1 - 1, ∞2.

y y y f(x) = 2x2  1
y= 2x

y = 2x 8
1
x 2
0 2 6
–2 (0, 1) 4
0
1- ∞, ∞2; 10, ∞2 x
f(x) = 2 x+3 4
(0, 1) 2
(c) y y = 2x
–2 f (x) = –2 x 2 (0, 1)
f (x) = 3 x+2
2 (3, 1) (2, 0) 4
(0, 1) x
7 –4 x 0
0
–6 –3 2 y  1

2 Figure 18 Figure 19 Figure 20


2
x
0 2 ■
✔ Now Try Exercises 39, 41, and 47.
y = 2

1- ∞, ∞2; 1- 2, ∞2
Exponential Equations   Because the graph of ƒ1x2 = a x is that of a
one-to-one function, to solve a x1 = a x2, we need only show that x1 = x2. This
property is used to solve an exponential equation, which is an equation with
a variable as exponent.

Classroom Example 4 Example 4 Solving an Exponential Equation


1
Solve 5x = . x
125 Solve A 13 B = 81.
Answer:  5 -36 Solution  Write each side of the equation using a common base.

1 x
a b = 81
3
100 13-12x = 81    Definition of negative exponent
3-x = 81    1am2n = amn
−5 1
3-x = 34    Write 81 as a power of 3.
- x = 4    Set exponents equal (Property (b) given earlier).
−100
x = - 4   Multiply by -1.
The x-intercept of the graph of
x Check by substituting - 4 for x in the original equation. The solution set is 5 - 46.
y= A 13 B - 81 can be used to verify the
solution in Example 4. ■
✔ Now Try Exercise 73.
Copyright Pearson. All Rights Reserved.

M04_LHSD7659_12_AIE_C04_pp405-496.indd 424 06/10/15 6:45 PM


NOT FOR SALE
Exponential Functions
4.2  425

Classroom Example 5 Example 5 Solving an Exponential Equation


Solve 3x + 1 = 9 x - 3.
Solve 2x + 4 = 8x - 6 .
Answer:  576
Solution  Write each side of the equation using a common base.

2x + 4 = 8x - 6
Teaching Tip  Now is a good
time to review the properties of 2x + 4 = 1232x - 6    Write 8 as a power of 2.
exponents.
2x + 4 = 23x - 18    1am2n = amn
x + 4 = 3x - 18   Set exponents equal (Property (b)).
- 2x = - 22    Subtract 3x and 4.

x = 11    Divide by -2.
Teaching Tip  Give an example
such as Check by substituting 11 for x in the original equation. The solution set is 5116.
16 x - 2 = 9 1 - x ■
✔ Now Try Exercise 81.
that cannot be solved using the
methods of this section. Show how Later in this chapter, we describe a general method for solving exponential
a graphing calculator can be used equations where the approach used in Examples 4 and 5 is not possible. For
to find the approximate solution instance, the above method could not be used to solve an equation like
1.558.
7x = 12
because it is not easy to express both sides as exponential expressions with the
same base.
In Example 6, we solve an equation that has the variable as the base of an
exponential expression.

Classroom Example 6 Example 6 Solving an Equation with a Fractional Exponent


Solve x 2/3 = 25.
Solve x 4/3 = 81.
Answer:  5{1256
Solution  Notice that the variable is in the base rather than in the exponent.

x 4/3 = 81
Teaching Tip  Caution students
4
that in solving equations like the A2
3
xB = 81 Radical notation for am/n
one in Example 6, they must
Take fourth roots on each side.
consider both even roots. x = { 3 Remember to use {.
3

2
x = { 27 Cube each side.
Check both solutions in the original equation. Both check, so the solution set is
5 { 276.
Alternative Method  There may be more than one way to solve an exponential
equation, as shown here.
x 4/3 = 81
1x 4/323 = 813 Cube each side.

x 4 = 13423 Write 81 as 34.

x 4 = 312 1am2n = a mn

x = {2
4 12
3 Take fourth roots on each side.
x = { 33 Simplify the radical.
x = { 27 Apply the exponent.

The same solution set, 5{ 276, results. ■


✔ Now Try Exercise 83.
Copyright Pearson. All Rights Reserved.

M04_LHSD7659_12_AIE_C04_pp405-496.indd 425 06/10/15 6:45 PM


NOT FOR SALE
426 Chapter 4  Inverse, Exponential, and Logarithmic Functions

Compound Interest   Recall the formula for simple interest, I = Prt, where
P is principal (amount deposited), r is annual rate of interest expressed as a deci-
mal, and t is time in years that the principal earns interest. Suppose t = 1 yr.
Then at the end of the year, the amount has grown to the following. 
P + Pr = P11 + r2   Original principal plus interest
If this balance earns interest at the same interest rate for another year, the bal-
ance at the end of that year will increase as follows.

3P11 + r24 + 3P11 + r24r = 3P11 + r2411 + r2   Factor.


= P11 + r22    a # a = a2

After the third year, the balance will grow in a similar pattern.

3P11 + r22 4 + 3P11 + r22 4r = 3P11 + r22 411 + r2   Factor.


= P11 + r23    a2 # a = a3

Continuing in this way produces a formula for interest compounded annually.


A = P1 1 + r 2 t
The general formula for compound interest can be derived in the same way.

Compound Interest

If P dollars are deposited in an account paying an annual rate of interest r


compounded (paid) n times per year, then after t years the account will con-
tain A dollars, according to the following formula.

r tn
A = P a1 + b
n

Classroom Example 7 Example 7 Using the Compound Interest Formula


Repeat Example 7 if $2500 is
deposited in an account paying Suppose $1000 is deposited in an account paying 4% interest per year com-
3% per year compounded pounded quarterly (four times per year).
semi-annually (twice per year).
(a) Find the amount in the account after 10 yr with no withdrawals.
Answers:
(b) How much interest is earned over the 10-yr period?
(a) $3367.14 (b) $867.14
Solution

r tn
(a) A = P a1 + b Compound interest formula
n
0.04 10142
A = 1000a 1 + b Let P = 1000, r = 0.04, n = 4, and t = 10.
4
A = 100011 + 0.01240 Simplify.

A = 1488.86 Round to the nearest cent.

Thus, $1488.86 is in the account after 10 yr.

(b) The interest earned for that period is


$1488.86 - $1000 = $488.86.

✔ Now Try Exercise 97(a).
Copyright Pearson. All Rights Reserved.

M04_LHSD7659_12_AIE_C04_pp405-496.indd 426 06/10/15 6:46 PM


NOT FOR SALE
Exponential Functions
4.2  427

In the formula for compound interest

r tn
A = P a1 + b ,
n
A is sometimes called the future value and P the present value. A is also called
the compound amount and is the balance after interest has been earned.

Classroom Example 8 Example 8 Finding Present Value


Brendan must pay a lump sum of
$15,000 in 8 yr. Becky must pay a lump sum of $6000 in 5 yr.
(a) What amount deposited (a) What amount deposited today (present value) at 3.1% compounded annually
today (present value) at 2.4% will grow to $6000 in 5 yr?
compounded annually will
give $15,000 in 8 yr? (b) If only $5000 is available to deposit now, what annual interest rate is neces-
(b) If only $12,025 is available sary for the money to increase to $6000 in 5 yr?
to deposit now, what annual
Solution
interest rate is necessary for
the money to increase to r tn
$15,000 in 8 yr? (a) A = P a1 + b    Compound interest formula
n
Answers:
(a) $12,407.71 (b) 2.80%
0.031 5112
6000 = P a 1 + b    Let A = 6000, r = 0.031, n = 1, and t = 5.
1
6000 = P11.03125    Simplify.

6000
P=    Divide by 11.03125 to solve for P.
11.03125
P ≈ 5150.60    Use a calculator.

If Becky leaves $5150.60 for 5 yr in an account paying 3.1% compounded


annually, she will have $6000 when she needs it. Thus, $5150.60 is the
present value of $6000 if interest of 3.1% is compounded annually for 5 yr.
r tn
(b) A = P a1 + b Compound interest formula
n
6000 = 500011 + r25 Let A = 6000, P = 5000, n = 1, and t = 5.
6
= 11 + r25 Divide by 5000.
5
6 1/5
a b =1+r Take the fifth root on each side.
5
6 1/5
a b -1=r Subtract 1.
5
r ≈ 0.0371 Use a calculator.

An interest rate of 3.71% will produce enough interest to increase the $5000
to $6000 by the end of 5 yr.

✔ Now Try Exercises 99 and 103.

Caution  When performing the computations in problems like those in


Examples 7 and 8, do not round off during intermediate steps. Keep all cal-
culator digits and round at the end of the process.

Copyright Pearson. All Rights Reserved.

M04_LHSD7659_12_AIE_C04_pp405-496.indd 427 06/10/15 6:46 PM


NOT FOR SALE
428 Chapter 4  Inverse, Exponential, and Logarithmic Functions

1 n The Number e and Continuous Compounding   The more often inter-


n a1 + b
n est is compounded within a given time period, the more interest will be earned.
(rounded) Surprisingly, however, there is a limit on the amount of interest, no matter how
1 2 often it is compounded.
2 2.25 Suppose that $1 is invested at 100% interest per year, compounded n times
5 2.48832 per year. Then the interest rate (in decimal form) is 1.00, and the interest rate per
10 2.59374 1
100 2.70481 period is n . According to the formula (with P = 1), the compound amount at the
1000 2.71692 end of 1 yr will be
10,000 2.71815 1 n
1,000,000 2.71828 A = a1 + b .
n
A calculator gives the results in the margin for various values of n. The table sug-
1 n
gests that as n increases, the value of A 1 + n B gets closer and closer to some fixed
number. This is indeed the case. This fixed number is called e. (In mathematics,
e is a real number and not a variable.)

Value of e
y
y = 3x
e ? 2.718281828459045
8 y = ex
7
6 Figure 21 shows graphs of the functions
5
4 y = 2x y = 2x,  y = 3x, and y = ex.
3
2 Because 2 6 e 6 3, the graph of y = ex lies “between” the other two graphs.
1 As mentioned above, the amount of interest earned increases with the fre-
x 1 n
–1
0
1 2 quency of compounding, but the value of the expression A 1 + n B approaches e
as n gets larger. Consequently, the formula for compound interest approaches
Figure 21 a limit as well, called the compound amount from continuous compounding.

Continuous Compounding
Teaching Tip  Tell students that
as n increases without bound,
n If P dollars are deposited at a rate of interest r compounded continuously
A 1 + 1n B approaches e. for t years, then the compound amount A in dollars on deposit is given by
the following formula.
A = Pert

Classroom Example 9 Example 9 Solving a Continuous Compounding Problem


Suppose $8000 is deposited in
an account paying 5% interest Suppose $5000 is deposited in an account paying 3% interest compounded con-
compounded continuously for 6 yr. tinuously for 5 yr. Find the total amount on deposit at the end of 5 yr.
Find the total amount on deposit at
Solution
the end of 6 yr.
Answer:  $10,798.87 A = Pert    Continuous compounding formula

A= 5000e0.03152    Let P = 5000, r = 0.03, and t = 5.

A = 5000e0.15    Multiply exponents.

A ≈ 5809.17 or $5809.17   Use a calculator.

Check that daily compounding would have produced a compound amount about
$0.03 less.

✔ Now Try Exercise 97(b).
Copyright Pearson. All Rights Reserved.

M04_LHSD7659_12_AIE_C04_pp405-496.indd 428 06/10/15 6:46 PM


NOT FOR SALE
Exponential Functions
4.2  429

Classroom Example 10 Example 10  omparing Interest Earned as Compounding Is More


C
In Classroom Example 7, we Frequent
found the total amount on deposit
after 10 yr in an account paying In Example 7, we found that $1000 invested at 4% compounded quarterly for
3% semiannually in which $2500 10 yr grew to $1488.86. Compare this same investment compounded annually,
was invested. Find the amounts semiannually, monthly, daily, and continuously.
from the same investment for
Solution  Substitute 0.04 for r, 10 for t, and the appropriate number of com-
interest compounded quarterly,
monthly, daily, and continuously. pounding periods for n into the formulas
Answer: r tn
quarterly: $3370.87;
A = P a1 + b    Compound interest formula
n
monthly: $3373.38;
daily: $3374.61; and A = Pert.    Continuous compounding formula
continuously: $3374.65
The results for amounts of $1 and $1000 are given in the table.

Compounded $1 $1000
Annually 11 + 0.04210 ≈ 1.48024 $1480.24
0.04 10122
Semiannually a1 + b ≈ 1.48595 $1485.95
2
0.04 10142
Quarterly a1 + b ≈ 1.48886 $1488.86
4
0.04 101122
Monthly a1 + b ≈ 1.49083 $1490.83
12
0.04 1013652
Daily a1 + b ≈ 1.49179 $1491.79
365
Continuously e1010.042 ≈ 1.49182 $1491.82

Comparing the results for a $1000 investment, we notice the following.

• Compounding semiannually rather than annually increases the value of the


account after 10 yr by $5.71.
• Quarterly compounding grows to $2.91 more than semiannual compounding
after 10 yr.
• Daily compounding yields only $0.96 more than monthly compounding.
• Continuous compounding yields only $0.03 more than daily compounding.
Looking Ahead To Calculus
In calculus, the derivative allows us Each increase in compounding frequency earns less additional interest.
to determine the slope of a tangent

✔ Now Try Exercise 105.
line to the graph of a function. For the
function

ƒ1x2 = ex, Exponential Models   The number e is important as the base of an expo-
the derivative is the function ƒ itself: nential function in many practical applications. In situations involving growth or
decay of a quantity, the amount or number present at time t often can be closely
ƒ′1x2 = ex.
modeled by a function of the form
Therefore, in calculus the exponential
function with base e is much easier to y = y0ekt,
work with than exponential functions
where y0 is the amount or number present at time t = 0 and k is a constant.
having other bases.
Exponential functions are used to model the growth of microorganisms in a
culture, the growth of certain populations, and the decay of radioactive material.
Copyright Pearson. All Rights Reserved.

M04_LHSD7659_12_AIE_C04_pp405-496.indd 429 06/10/15 6:46 PM


NOT FOR SALE
430 Chapter 4  Inverse, Exponential, and Logarithmic Functions

Classroom Example 11 Example 11 Using Data to Model Exponential Growth


Refer to the model given in
Example 11. Data from recent years indicate that future amounts of carbon dioxide in the
(a) What will be the atmospheric atmosphere may grow according to the table. Amounts are given in parts per
carbon dioxide level in parts million.
per million in 2015? (a) Make a scatter diagram of the data. Do Year Carbon Dioxide (ppm)
(b) Use a graph of this model the carbon dioxide levels appear to grow
to estimate when the carbon 1990  353
exponentially?
dioxide level will be double 2000  375
the level that it was in 2000. (b) One model for the data is the function 2075  590
Answers: y= 0.001942e0.00609x, 2175 1090
(a) 415 ppm (b)  by 2112
2275 2000
where x is the year and 1990 … x … 2275.
Use a graph of this model to estimate Source: International Panel on Climate
Change (IPCC).
when future levels of carbon dioxide will
2100 double and triple over the preindustrial
level of 280 ppm.
Solution

(a) We show a calculator graph for the data in Figure 22(a). The data appear to
1975 2300
300 resemble the graph of an increasing exponential function.
(a)
(b) A graph of y = 0.001942e0.00609x in Figure 22(b) shows that it is very
y = 0.001942e0.00609x close to the data points. We graph y2 = 2 # 280 = 560 in Figure 23(a) and
y2 = 3 # 280 = 840 in Figure 23(b) on the same coordinate axes as the given
2100 function, and we use the calculator to find the intersection points.

y1 = 0.001942e0.00609x y1 = 0.001942e0.00609x

1975 2300
300 2100 2100

(b)

Figure 22

1975 2300 1975 2300

−500 −500

y2 = 560 y2 = 840
(a) (b)
Figure 23

The graph of the function intersects the horizontal lines at x-values of


approximately 2064.4 and 2130.9. According to this model, carbon dioxide
levels will have doubled during 2064 and tripled by 2131.
(a)

✔ Now Try Exercise 107.
y1 = 0.001923(1.006109) x

2100
  Graphing calculators are capable of fitting exponential curves to scatter
diagrams like the one found in Example 11. The TI-84 Plus displays another
(different) equation in Figure 24(a) for the atmospheric carbon dioxide exam-
ple, approximated as follows.
1975
300
2300 y = 0.00192311.0061092x
(b) This calculator form differs from the model in Example 11. Figure 24(b) shows
Figure 24 the data points and the graph of this exponential regression equation.  ■
Copyright Pearson. All Rights Reserved.

M04_LHSD7659_12_AIE_C04_pp405-496.indd 430 06/10/15 6:46 PM


NOT FOR SALE
Exponential Functions
4.2  431

4.2 Exercises

1. 16; 16  
1
2.  rises Concept Preview  Fill in the blank(s) to correctly complete each sentence.
3. falls 1. If ƒ1x2 = 4x, then ƒ122 = and ƒ1 - 22 = .
4. 1- ∞, ∞2; 10, ∞2
1 2. If a 7 1, then the graph of ƒ1x2 = ax from left to right.
5. 8 ; 1; 8   6.  x; 4; 5 (rises/falls)
7. 5 - 36   8.  5{2166 3. If 0 6 a 6 1, then the graph of ƒ1x2 = ax from left to right.
9. 52540.226 10.  50.036 (rises/falls)

1 1 4. The domain of ƒ1x2 = 4x is and the range is  .


11. 9 12. 27 13.  9 14.  27
1 1
15. 16 16.  64 17. 16 18. 64 5. The graph of ƒ1x2 = 8x passes through the points ( - 1,  ), (0,  ), and (1,  ).
23 1 x+4 1 x
19. 323 20.  27 6. The graph of ƒ1x2 = - A 3 B - 5 is that of ƒ1x2 = A3B reflected across the
1
21. 8 22. 32 -axis, translated units to the left and units down.
23. 13.076 24. 0.158
25. 10.267 26. 0.039
Concept Preview  Solve each equation. Round answers to the nearest hundredth
27. y 28. y as needed.
9 1 x
8 7. a b = 64 8. x 2/3 = 36
f (x) = 3 x 4
4
1 f(x) = 4 x
0
x 0.03 8142
2 x 9. A = 2000 a1 + b 10. 10,000 = 500011 + r225
–10 1 4
29. y 30. y

4 x
() A 14 B , find each of the following. Round answers to the nearest
x
() For ƒ1x2 = 3x and g1x2 =
x
3
f(x) = 13 f(x) = 14
1 1
–10 1
x
–10 1
x thousandth as needed. See Example 1.
11. ƒ122 12. ƒ132 13. ƒ1 - 22 14. ƒ1 - 32
31. y 32. y 15. g122 16. g132 17. g1 - 22 18. g1 - 32
3 5 3 5
19. ƒa b 20. ƒa - b 21. ga b 22. ga - b
5
f(x) = ()
5 x 2 2 2 2
1
f(x) = ()3 x
2
3
1
3
x
0
x –1 0 1 23. ƒ12.342 24. ƒ1 - 1.682 25. g1 - 1.682 26. g12.342
3

33. y 34. y
10 Graph each function. See Example 2.
6 1 x
f(x) = 16 ()
–x
27. ƒ1x2 = 3x 28. ƒ1x2 = 4x 29. ƒ1x2 = a b
f(x) = ( ) 1 x 1 3
10 x
1 –1 0 1
x 1 x 3 x 5 x
–2 –1 0 1 2 30. ƒ1x2 = a b 31. ƒ1x2 = a b 32. ƒ1x2 = a b
4 2 3
35. y 36. y
10 1 -x 1 -x
f(x) = 4
–x
33. ƒ1x2 = a b 34. ƒ1x2 = a b 35. ƒ1x2 = 4-x
10 6
4
–1
x f(x) = 10–x
36. ƒ1x2 = 10 -x 37. ƒ1x2 = 2x 38. ƒ1x2 = 2-x
0
1
x
–2 0 2
Graph each function. Give the domain and range. See Example 3.
37. y 38. y
39. ƒ1x2 = 2x + 1 40. ƒ1x2 = 2x - 4 41. ƒ1x2 = 2x + 1
8 1
f(x) = 2–  x 
42. ƒ1x2 = 2x - 4 43. ƒ1x2 = - 2x + 2 44. ƒ1x2 = - 2x - 3
1
2
1
x
45. ƒ1x2 = 2-x 46. ƒ1x2 = - 2-x 47. ƒ1x2 = 2x - 1 + 2
0 x
3 –2 0 2
f(x) = 2x 48. ƒ1x2 = 2x + 3 + 1 49. ƒ1x2 = 2x + 2 - 4 50. ƒ1x2 = 2x - 3 - 1

Copyright Pearson. All Rights Reserved.

M04_LHSD7659_12_AIE_C04_pp405-496.indd 431 06/10/15 6:46 PM


NOT FOR SALE
432 Chapter 4  Inverse, Exponential, and Logarithmic Functions

39. y x
f(x) = 2 + 1
40. y
x Graph each function. Give the domain and range. See Example 3.
f(x) = 2 – 4

8 1 x 1 x 1 x+2
2 51. ƒ1x2 = a b - 2 52. ƒ1x2 = a b + 4 53. ƒ1x2 = a b
x 3 3 3
0 2
2 y=1 –3 1 x-4 1 -x + 1 1 -x - 2
x y = –4 54. ƒ1x2 = a b 55. ƒ1x2 = a b 56. ƒ1x2 = a b
–2 0 2 3 3 3
1- ∞, ∞2; 11, ∞2 1- ∞, ∞2; 1- 4, ∞2
1 -x 1 -x 1 x-2
41. y 42. y
57. ƒ1x2 = a b 58. ƒ1x2 = - a b 59. ƒ1x2 = a b +2
3 3 3
f(x) = 2 x – 4

8 8 1 x-1 1 x+2 1 x+3


60. ƒ1x2 = a b + 3 61. ƒ1x2 = a b - 1 62. ƒ1x2 = a b -2
3 3 3
2 f(x) = 2 x + 1 2

0
x
0
x Connecting Graphs with Equations  Write an equation for the graph given. Each repre-
4 4
sents an exponential function ƒ with base 2 or 3, translated and/or reflected.
1- ∞, ∞2; 10, ∞2 1- ∞, ∞2; 10, ∞2
63. y 64. y 65. y
43. y 44. y
(4, 9)
–2 0 5 (2, 7) (0, 7)
x x
0 2
–4 –4
f(x) = –2 x + 2
(3, 3) (–2, 1)
(1, 1) (–3, 0)
f(x) = –2 x – 3 x (2, 1)
0 x
(0, –1) x 0
1- ∞, ∞2; 1- ∞, 02 1- ∞, ∞2; 1- ∞, 02 0 y = –1
y = –2
45. y 46. y

–2 0 2
66. y 67. y 68. y
–1
x (–2, 2)
4 –x
f(x) = –2 y=3
f(x) = 2 –x –4 (1, 7)
x
–4 0 2 (–1, 4) (–1, 1) (–2, 1)
(0, 5) 0 x
x
0
1- ∞, ∞2; 10, ∞2 1- ∞, ∞2; 1- ∞, 02 y=3 (0, –1) (–1, –1) (0, –2)

47. y 48. y x y = –3
f(x) = 2 x – 1  2 0

4
2 y=1
69. y 70. y
y=2
x x
y=5
0 1 2 2 0 2
f (x) = 2 x + 3  1 (–1, 3) (0, 4)
(–1, 4)
1- ∞, ∞2; 12, ∞2 1- ∞, ∞2; 11, ∞2
y=1 (0, 2)
(1, 43 )
x
x 0
49. 50. 0
y y f (x) = 2 x – 3  1
(–3, –3)
3
2
2 1 0 3
0 2
x x Solve each equation. See Examples 4–6.
2 y = 1
5 x 4 2 x 9
y = 4 71. 4x = 2 72. 125x = 5 73. a b = 74. a b =
2 25 3 4
f(x) = 2 x + 2  4
1- ∞, ∞2; 1- 4, ∞2 1- ∞, ∞2; 1- 1, ∞2 75. 23 - 2x = 8 76. 52 + 2x = 25 77. e4x - 1 = 1e22x 78. e3 - x = 1e32-x
79. 274x = 9 x + 1 80. 322x = 16 x - 1 81. 4x - 2 = 23x + 3 82. 26 - 3x = 8x + 1
51. y 52. y

9 10 83. x 2/3 = 4 84. x 2/5 = 16 85. x 5/2 = 32 86. x 3/2 = 27


5 1 1
3 y=4 87. x -6 = 88. x -4 = 89. x 5/3 = - 243 90. x 7/5 = - 128
0 3
x
64 256
y = –2 x
0 3
() 1 -x 1 x+1 1 x+1
( 3) x+4
1 x
f(x) = 1 x + 4
93. A 22 B
f(x) = –2
3 91. a b = a 2 b 92. ex - 1 = a b = 4x
1- ∞, ∞2; 1- 2, ∞2 1- ∞, ∞2; 14, ∞2
e e e4
-x 1 x+2 1 1
94. A 2 5B
3
= a b 95. = x -3 96. = x -5
5 27 32

Copyright Pearson. All Rights Reserved.

M04_LHSD7659_12_AIE_C04_pp405-496.indd 432 07/10/15 7:28 AM


NOT FOR SALE
Exponential Functions
4.2  433

53. y 54. y Solve each problem. See Examples 7–9.


( 3)
f(x) = 1 x + 2
9 8
( 3)
f(x) = 1 x – 4
97. Future Value  Find the future value and interest earned if $8906.54 is invested for
4 9 yr at 3% compounded
3
0 x (a) semiannually (b) continuously.
x 4
–2 0 2
98. Future Value  Find the future value and interest earned if $56,780 is invested at
1- ∞, ∞2; 10, ∞2 1- ∞, ∞2; 10, ∞2 2.8% compounded
55. y 56. y (a)  quarterly for 23 quarters (b)  continuously for 15 yr.
9 9 99. Present Value Find the present value that will grow to $25,000 if interest is 3.2%
6 compounded quarterly for 11 quarters.
3

0 3
x x 100. Present Value  Find the present value that will grow to $45,000 if interest is 3.6%
–2 –3 –2 0
compounded monthly for 1 yr.
f(x) = 3 ( 1 ) –x + 1 1
()
f(x) = 3 –x – 2
101. Present Value  Find the present value that will grow to $5000 if interest is 3.5%
1- ∞, ∞2; 10, ∞2 1- ∞, ∞2; 10, ∞2
compounded quarterly for 10 yr.
57. y 58. y
102. Interest Rate  Find the required annual interest rate to the nearest tenth of a percent
9 2
0 x for $65,000 to grow to $65,783.91 if interest is compounded monthly for 6 months.
f(x) = () 1 –x
3 –3
–3
3
3 f(x) = – 3 ( 1 ) –x 103. Interest Rate  Find the required annual interest rate to the nearest tenth of a percent
0 3
x for $1200 to grow to $1500 if interest is compounded quarterly for 9 yr.
–9

1- ∞, ∞2; 10, ∞2 1- ∞, ∞2; 1- ∞, 02 104. Interest Rate  Find the required annual interest rate to the nearest tenth of a percent
for $5000 to grow to $6200 if interest is compounded quarterly for 8 yr.
59. 60. y
()
x–2
y
f(x) = 13 2
f(x) = 13 ()
x–1
3
Solve each problem. See Example 10.
6
105. Comparing Loans  Bank A is lending money at 6.4% interest compounded annu-
2
3
y=3 ally. The rate at Bank B is 6.3% compounded monthly, and the rate at Bank C is
y=2
x x 6.35% compounded quarterly. At which bank will we pay the least interest?
0 0 2
2
106. Future Value  Suppose $10,000 is invested at an annual rate of 2.4% for 10 yr.
1- ∞, ∞2; 12, ∞2 1- ∞, ∞2; 13, ∞2 Find the future value if interest is compounded as follows.
61. y (a)  annually    (b)  quarterly    (c)  monthly    (d)  daily (365 days)
8 (Modeling)  Solve each problem. See Example 11.
f(x) = ()
1 x+2
3
1
107. Atmospheric Pressure  The atmospheric pressure (in millibars) at a given altitude
(in meters) is shown in the table.
2 2
0 x
1 Altitude Pressure Altitude Pressure
y = 1
   0 1013 6000 472
1- ∞, ∞2; 1- 1, ∞2
1000  899 7000 411
62. y
2000  795 8000 357
7 3000  701  9000 308
4000  617 10,000 265
()
x+3
f(x) = 13 2
5000  541    
x
5 0 Source: Miller, A. and J. Thompson, Elements of
y = 2 Meteorology, Fourth Edition, Charles E. Merrill
Publishing Company, Columbus, Ohio.
1- ∞, ∞2; 1- 2, ∞2
(a) U
 se a graphing calculator to make a scatter diagram of the data for atmospheric
63. ƒ1x2 = 3x - 2 pressure P at altitude x.
64. ƒ1x2 = 3x - 2 (b) Would a linear or an exponential function fit the data better?
65. ƒ1x2 = 2x + 3 - 1 (c) The following function approximates the data.
66. ƒ1x2 = 2x + 1 + 3
P1x2 = 1013e -0.0001341x
67. ƒ1x2 = -2x + 2 + 3
68. ƒ1x2 = 2-x - 3 Use a graphing calculator to graph P and the data on the same coordinate axes.
69. ƒ1x2 = 3-x + 1 (d) Use P to predict the pressures at 1500 m and 11,000 m, and compare them to
70. ƒ1x2 = -2-x + 5 the actual values of 846 millibars and 227 millibars, respectively.

Copyright Pearson. All Rights Reserved.

M04_LHSD7659_12_AIE_C04_pp405-496.indd 433 06/10/15 6:46 PM


NOT FOR SALE
434 Chapter 4  Inverse, Exponential, and Logarithmic Functions

71.  E 2 F  
1
72.  E 3 F
1 108. World Population Growth  World population in millions closely fits the exponen-
tial function
73. 5 - 26  
74.  5 - 26
75. 506   76.  506 ƒ1x2 = 6084e0.0120x,
where x is the number of years since 2000. (Source: U.S. Census Bureau.)
77.  E 2 F  
78.  E - 2 F
1 3

(a) The world population was about 6853 million in 2010. How closely does the
79.  E 5 F  
80.  E - 3 F
1 2
function approximate this value?
82.  E 2 F (b)  Use this model to predict world population in 2020 and 2030.
1
81. 5 - 76  
83. 5{86   84.  5{10246 109. Deer Population  The exponential
85. 546   86.  596 growth of the deer population in Mas-
87. 5{26   88.  5{46 sachusetts can be approximated using the
89. 5 - 276  
90.  5 - 326 model

91.  E - 3 F  
2
92.  E - 5 F
3 ƒ1x2 = 50,00011 + 0.062x,
where 50,000 is the initial deer population
93.  E 3 F  
4
94.  5 - 36
and 0.06 is the rate of growth. ƒ1x2 is the
95. 536  
96.  526 total population after x years have passed.
Find each value to the nearest thousand.
97. (a)  $11,643.88; $2737.34
(b)  $11,667.25; $2760.71 (a)  Predict the total population after 4 yr.
98. (a)  $66,661.21; $9881.21 (b) If the initial population was 30,000 and
(b)  $86,416.98; $29,636.98 the growth rate was 0.12, how many
99. $22,902.04 100. $43,411.15 deer would be present after 3 yr?
101. $3528.81 102. 2.4% (c) How many additional deer can we e­ xpect in 5 yr if the initial population is 45,000
103. 2.5% 104. 2.7% and the current growth rate is 0.08?
105. Bank A (even though it has 110. Employee Training  A person learning certain skills involving repetition tends
the greatest stated rate) to learn quickly at first. Then learning tapers off and skill acquisition approaches
106. (a) $12,676.51 some upper limit. Suppose the number of symbols per minute that a person using a
(b) $12,703.38 keyboard can type is given by
(c) $12,709.44
(d) $12,712.39 ƒ1t2 = 250 - 12012.82-0.5t,
where t is the number of months the operator has been in training. Find each value
107. (a) 1200
to the nearest whole number.
(a)  ƒ122 (b)  ƒ142 (c)  ƒ1102
(d) What happens to the number of symbols per minute after several months of
−1000 11,000 training?
0
(b) exponential
(c)  P(x) = 1013e−0.0001341x Use a graphing calculator to find the solution set of each equation. Approximate the
1200 solution(s) to the nearest tenth.
111. 5e3x = 75 112. 6 -x = 1 - x 113. 3x + 2 = 4x 114. x = 2x
115. A function of the form ƒ1 x2 = xr, where r is a constant, is a power function.
Discuss the difference between an exponential function and a power function.
−1000 11,000
0
116. Concept Check If ƒ1x2 = ax and ƒ132 = 27, determine each function value.
(d) P115002 ≈ 828 mb;
P111,0002 ≈ 232 mb (a)  ƒ112 (b)  ƒ1 - 12 (c)  ƒ122 (d)  ƒ102
108. (a) The function gives
approximately Concept Check  Give an equation of the form ƒ1x2 = ax to define the exponential func-
6860 million, which tion whose graph contains the given point.
differs by 7 million from
the actual value. 117. 13, 82 118. 13, 1252 119. 1 - 3, 642 120. 1 - 2, 362
(b) 7734 million;
8720 million
Concept Check  Use properties of exponents to write each function in the form
109. (a) 63,000 (b) 42,000 ƒ1t2 = kat, where k is a constant. (Hint: Recall that a x + y = a x # a y.)
(c) 21,000
110. (a) 207  (b) 235  (c) 249 1 1 - 2t 1 1 - 2t
121. ƒ1t2 = 32t + 3 122. ƒ1t2 = 23t + 2 123. ƒ1t2 = a b 124. ƒ1t2 = a b
(d) The number of symbols 3 2
approaches 250.
Copyright Pearson. All Rights Reserved.

M04_LHSD7659_12_AIE_C04_pp405-496.indd 434 06/10/15 6:46 PM


NOT FOR SALE
4.3  Logarithmic Functions 435

111. 50.96 112.  50, 0.76 In calculus, the following can be shown.
113. 5 - 0.5, 1.36 114.  ∅
x2 x3 x4 x5
+g
2# 1 3# 2# 1 4# 3# 2# 1 5# 4# 3# 2# 1
ex = 1 + x + + + +
115. The variable is located in
the base of a power function
Using more terms, one can obtain a more accurate approximation for ex.
and in the exponent of an
exponential function. 125. Use the terms shown, and replace x with 1 to approximate e1 = e to three decimal
1 places. Check the result with a calculator.
116. (a) 3  (b)  3   (c) 9  (d) 1
126. Use the terms shown, and replace x with - 0.05 to approximate e -0.05 to four deci-
117. ƒ1x2 = 2x 118.  ƒ1x2 = 5x mal places. Check the result with a calculator.
x x
119. ƒ1x2 = A 14 B 120.  ƒ1x2 = A 16 B

121. ƒ1t2 = 27 # 9 t Relating Concepts


122. ƒ1t2 = 4 # 8t
For individual or collaborative investigation (Exercises 127—132)
123. ƒ1t2 = A 13 B 9 t 124.  ƒ1t2 = A 12 B 4t
Consider ƒ1x2 = ax, where a 7 1. Work these exercises in order.
125. 2.717 127. Is ƒ a one-to-one function? If so, what kind of related function exists for ƒ?
(A calculator gives 2.718.)
126. 0.9512 128. If ƒ has an inverse function ƒ -1, sketch ƒ and ƒ -1 on the same set of axes.
(A calculator gives 0.9512.)
127. yes; an inverse function
129. If ƒ -1 exists, find an equation for y = ƒ -11x2. (You need not solve for y.)
128. y
y = f(x) 130. If a = 10, what is the equation for y = ƒ -11x2? (You need not solve for y.)
(0, 1) 131. If a = e, what is the equation for y = ƒ -11x2? (You need not solve for y.)
0 x
(1, 0) 132. If the point 1p, q2 is on the graph of ƒ, then the point is on the graph
of ƒ -1.
y=x y = f –1(x)

129. x = a y 130.  x = 10 y
131. x = ey 132.  1q, p2

4.3 Logarithmic Functions

■ Logarithms
Logarithms   The previous section dealt with exponential functions of the
■ Logarithmic Equations form y = a x for all positive values of a, where a ≠ 1. The horizontal line test
■ Logarithmic Functions shows that exponential functions are one-to-one and thus have inverse functions.
■ Properties of The equation defining the inverse of a function is found by interchanging x and y
Logarithms in the equation that defines the function. Starting with y = a x and interchanging
x and y yields
x = a y.
Here y is the exponent to which a must be raised in order to obtain x. We call
this exponent a logarithm, symbolized by the abbreviation “log.” The expres-
sion log a x represents the logarithm in this discussion. The number a is the base
of the logarithm, and x is the argument of the expression. It is read “logarithm
with base a of x,” or “logarithm of x with base a,” or “base a logarithm of x.”

Logarithm

For all real numbers y and all positive numbers a and x, where a ≠ 1,
y = log a x  is equivalent to  x = a y.
The expression log a x represents the exponent to which the base a must be
raised in order to obtain x.

Copyright Pearson. All Rights Reserved.

M04_LHSD7659_12_AIE_C04_pp405-496.indd 435 06/10/15 6:46 PM


NOT FOR SALE
436 Chapter 4  Inverse, Exponential, and Logarithmic Functions

Classroom Example 1 Example 1 Writing Equivalent Logarithmic and Exponential Forms


Complete the missing equivalent
forms in the table. The table shows several pairs of equivalent statements, written in both logarith-
mic and exponential forms.
  Logarithmic Exponential
Form Form Solution
To remember the relationships among
(a)   34 = 81
a, x, and y in the two equivalent forms
(b) log1/2 8 = - 3   Logarithmic Form Exponential Form y = loga x and x = ay, refer to these
(c)   10 3 = 1000 log2 8 = 3 23 = 8 diagrams.
1   -4
A logarithm is an exponent.
(d) log5 125 = -3 log1/2 16 = - 4 A 12 B = 16 Exponent
(e)   121 = 12
log10 100,000 = 5 10 5 = 100,000
(f ) log6 1 = 0   Logarithmic form: y = loga x
1
log3 81 = -4 3-4 =
1
81
Answers: Base
-3
(a) log3 81 = 4 (b)  A 12 B =8 log5 5 = 1 51 = 5 Exponent
0
(c) log10 1000 = 3 (d)  5-3 =
1
125 log3/4 1 = 0 A 34 B =1 Exponential form: ay = x
(e) log12 12 = 1 (f )  60 =1
Base


✔ Now Try Exercises 11, 13, 15, and 17.
Teaching Tip  Have students
verbalize the statements in the left
Logarithmic Equations   The definition of logarithm can be used to solve
column of the table in Example 1.
For instance, “log2 8 is the a logarithmic equation, which is an equation with a logarithm in at least one term.
exponent to which 2 must be
raised in order to obtain 8.” Example 2 Solving Logarithmic Equations

Solve each equation.


Classroom Example 2 8 5 3
Solve each equation. (a) logx = 3    (b)  log4 x =     (c)  log49 2 7=x
27 2
16
(a) logx = -2
9 Solution  Many logarithmic equations can be solved by first writing the
3 equation in exponential form.
(b) log16 x =
4 8
(a) logx =3
(c) log36 26 = x 27
Answers: 8
x3 = Write in exponential form.
(a) E F  (b)  586  (c)  E F
3 1
4 4 27
2 3 8 3
x3 = a b 27 = A 23 B
3
2
x= Take cube roots.
3

8
Teaching Tip  Point out that Check logx = 3   Original equation
Example 2 illustrates three 27
different locations for the 8 ≟
variable: the base, the argument, log2/3 3   Let x = 23 .
and the exponent. Remind students 27
of the restrictions on x in parts (a)
2 3 8
and (b), and that there is no a b ≟   Write in exponential form.
restriction on x in part (c). 3 27
8 8
=   ✓ True
27 27

The solution set is E 3 F.


2

Copyright Pearson. All Rights Reserved.

M04_LHSD7659_12_AIE_C04_pp405-496.indd 436 06/10/15 6:46 PM


NOT FOR SALE
4.3  Logarithmic Functions 437

5 3
(b) log4 x = (c) log49 27 = x
2
Write in exponential
49 x = 27   W rite in exponential
3
45/2 = x form. form.
141/225 = x amn = 1am2n
1722x = 71/3    W rite with the same
base.
25 = x 41/2 = 12221/2 = 2 Power rule for
72x = 71/3    exponents
32 = x Apply the exponent.
1
5 2x =    Set exponents equal.
Check  log4 32 ≟ Let x = 32. 3
2
1
45/2 ≟ 32 x=    Divide by 2.
6
25 ≟ 32
5
45/2 = A 24 B = 25 A check shows that the solution set
is E 6 F.
  32 = 32  ✓ True 1

The solution set is 5326. ■


✔ Now Try Exercises 19, 29, and 35.

Logarithmic Functions   We define the logarithmic function with base a.

Logarithmic Function

If a 7 0, a ≠ 1, and x 7 0, then the logarithmic function with base a is

ƒ 1 x 2 = log a x.

Exponential and logarithmic functions are inverses of each other. To show


this, we use the three steps for finding the inverse of a function.

ƒ1x2 = 2x    Exponential function with base 2

y = 2x    Let y = ƒ1x2.

Step 1 x= 2y    Interchange x and y.

Step 2 y = log2 x   Solve for y by writing in equivalent logarithmic form.


Step 3 ƒ -11x2 = log2 x   Replace y with ƒ -11x2.

The graph of ƒ1x2 = 2x has the x-axis as horizontal asymptote and is shown
in red in Figure 25. Its inverse, ƒ -11x2 = log2 x, has the y-axis as vertical asymp-
tote and is shown in blue. The graphs are reflections of each other across the line
y = x. As a result, their domains and ranges are interchanged.

f(x) = 2 x
Domain: (–∞, ∞)
x ƒ1x2 = 2 x x ƒ−1 1x2 = log2 x y Range: (0, ∞)
y=x
1 1 8
-2 4 4 -2
1 1 6
-1 2 2 -1 (1, 2)
0 1 1 0 4
(0, 1) f –1(x) = log 2 x
1 2 2 1
( )
Domain: (0, ∞)
2 4 4 2 –1, 1 Range: (–∞, ∞)
2 (2, 1)
x
0
–2 (1, 0) 4 6 8
–2
( 12 , –1)
Figure 25

Copyright Pearson. All Rights Reserved.

M04_LHSD7659_12_AIE_C04_pp405-496.indd 437 06/10/15 6:46 PM


NOT FOR SALE
438 Chapter 4  Inverse, Exponential, and Logarithmic Functions

The domain of an exponential function is the set of all real numbers, so the
range of a logarithmic function also will be the set of all real numbers. In the same
way, both the range of an exponential function and the domain of a logarithmic
function are the set of all positive real numbers.

Thus, logarithms can be found for positive numbers only.

Logarithmic Function  f 1 x 2 = log a x


Domain: 10, ∞2  Range: 1 - ∞, ∞2
For ƒ1x2 = log2 x:

x ƒ1x2 y f(x) = loga x, a > 1


1 f (x) = log a x, a > 1
4 -2
1 (a, 1)
2 -1
1 0 x
2 1 0 (1, 0)
4 2 ( 1 , –1
a )
8 3

This is the general behavior seen on


a calculator graph for any base a,
for a + 1.

Figure 26

• ƒ 1 x 2
= log a x, for a + 1, is increasing and continuous on its entire
domain, 10, ∞2.
• The y-axis is a vertical asymptote as x S 0 from the right.
• The graph passes through the points A 1a , - 1 B , 11, 02, and 1a, 12.

For ƒ1x2 = log1/2 x:

x ƒ1x2 y f(x) = loga x, 0 < a < 1


1
4 2
1 (a, 1)
2 1
(1, 0)
1 0 x
0
2 -1
4 -2 ( 1a , –1)
8 -3 f (x) = log a x, 0 < a < 1

This is the general behavior seen on


a calculator graph for any base a,
for 0 * a * 1.

Figure 27

• ƒ 1 x 2
= log a x, for 0 * a * 1, is decreasing and continuous on its entire
domain, 10, ∞2.
• The y-axis is a vertical asymptote as x S 0 from the right.
• The graph passes through the points A 1a , - 1 B , 11, 02, and 1a, 12.

Copyright Pearson. All Rights Reserved.

M04_LHSD7659_12_AIE_C04_pp405-496.indd 438 06/10/15 6:46 PM


NOT FOR SALE
4.3  Logarithmic Functions 439

  Calculator graphs of logarithmic functions sometimes do not give an accu-


rate picture of the behavior of the graphs near the vertical asymptotes. While it
may seem as if the graph has an endpoint, this is not the case. The resolution of
the calculator screen is not precise enough to indicate that the graph approaches
the vertical asymptote as the value of x gets closer to it. Do not draw incorrect
conclusions just because the calculator does not show this behavior.  ■

The graphs in Figures 26 and 27 and the information with them suggest the
following generalizations about the graphs of logarithmic functions of the form
ƒ1x2 = loga x.

Teaching Tip Encourage
students to learn the characteristics Characteristics of the Graph of f 1 x 2 = log a x
of the graph of
ƒ1x2 = loga x. 1. The points A 1a , - 1 B , 11, 02, and 1a, 12 are on the graph.
Ask them how the coordinates 2. If a 7 1, then ƒ is an increasing function.
1
A a, - 1 B , 11, 02, and 1a, 12 are If 0 6 a 6 1, then ƒ is a decreasing function.
affected by different translations
of the graph. Also, emphasize 3. The y-axis is a vertical asymptote.
that horizontal shifts affect the 4. The domain is 10, ∞2, and the range is 1 - ∞, ∞2.
domains of logarithmic functions.

Classroom Example 3 Example 3 Graphing Logarithmic Functions


Graph each function.
Graph each function.
(a) ƒ1x2 = log1/3 x
(b) ƒ1x2 = log5 x (a) ƒ1x2 = log1/2 x (b)  ƒ1x2 = log3 x
Answers: Solution
(a) (b) 1 x
y y (a) One approach is to first graph y = A 2 B , which defines the inverse function of ƒ,
f (x) = log1/3 x f(x) = log 5 x
by plotting points. Some ordered pairs are given in the table with the graph
1 1
shown in red in Figure 28.
1 x
–2
0 1 9
x
0 5
x
The graph of ƒ1x2 = log1/2 x is the reflection of the graph of y = A 2 B
across the line y = x. The ordered pairs for y = log1/2 x are found by inter-
1 x
changing the x- and y-values in the ordered pairs for y = A 2 B . See the graph
in blue in Figure 28.

y
x y= A 12 B
x
x ƒ1 x2 = log1/2 x y= ( 12 ) x y=x x ƒ1x2 = log3 x y

1
-2 4 4 -2 3 -1
-1 2 2 -1 1  0
0 1 1 0 4 3 f(x) = log 3 x
3  1
1 1
1 2 2 1 9  2
x
1 1 x 0
2 2 3 9
4 4 0
4 Think: x = 3y –2
1 1 –2
4 16 16 4
f(x) = log 1/2 x

Figure 28 Figure 29

(b) Another way to graph a logarithmic function is to write ƒ1x2 = y = log3 x


in exponential form as x = 3y, and then select y-values and calculate corre-
sponding x-values. Several selected ordered pairs are shown in the table for
the graph in Figure 29.
■✔ Now Try Exercise 55.
Copyright Pearson. All Rights Reserved.

M04_LHSD7659_12_AIE_C04_pp405-496.indd 439 06/10/15 6:46 PM


NOT FOR SALE
440 Chapter 4  Inverse, Exponential, and Logarithmic Functions

Caution  If we write a logarithmic function in exponential form in


order to graph it, as in Example 3(b), we start first with y-values to calcu-
late corresponding x-values. Be careful to write the values in the ordered
pairs in the correct order.

More general logarithmic functions can be obtained by forming the compo-


sition of ƒ1x2 = loga x with a function g1x2. For example, if ƒ1x2 = log2 x and
g1x2 = x - 1, then

1ƒ ∘ g21x2 = ƒ1g1x22 = log2 1x - 12.

The next example shows how to graph such functions.

Classroom Example 4 Example 4 Graphing Translated Logarithmic Functions


Graph each function. Give the
domain and range. Graph each function. Give the domain and range.
(a) ƒ1x2 = log3 1x + 22 (a) ƒ1x2 = log2 1x - 12 (b)  ƒ1x2 = 1log3 x2 - 1
(b) ƒ1x2 = 1log4 x2 + 2
(c) ƒ1x2 = log4 1x + 22 + 1
(c) ƒ1x2 = log2 1x - 32 + 2
Solution
Answers:
(a) y (a) The graph of ƒ1x2 = log2 1x - 12 is the graph of g1x2 = log2 x translated
f(x) = log 3 (x + 2)
x = –2
2
1 unit to the right. The vertical asymptote has equation x = 1. Because loga-
0 2
x rithms can be found only for positive numbers, we solve x - 1 7 0 to find
the domain, 11, ∞2. To determine ordered pairs to plot, use the equivalent
1- 2, ∞2; 1- ∞, ∞2 exponential form of the equation y = log2 1x - 12.
(b) y
y = log2 1x - 12
2
0 1
x x - 1 = 2y    Write in exponential form.
f(x) = (log 4 x) + 2
x= 2y +1    Add 1.
10, ∞2; 1- ∞, ∞2
(c) y We first choose values for y and then calculate each of the corresponding
x=3
x-values. The range is 1 - ∞, ∞2. See Figure 30.
3

x y y
0 6 x=1
2
4 4
f(x) = log2 (x – 1) f(x) = (log 3 x) – 1
f(x) = log2 (x  3)  2
2 (3, 1) 2
(5, 2) (9, 1)
13, ∞2; 1- ∞, ∞2 (3, 0)
x x
0 0
4 6 8 3 6 9
–2 (2, 0) –2 (1, 1)

Figure 30 Figure 31

(b) The function ƒ1x2 = 1log3 x2 - 1 has the same graph as g1x2 = log3 x
translated 1 unit down. We find ordered pairs to plot by writing the equation
y = 1log3 x2 - 1 in exponential form.

y = 1log3 x2 - 1
y + 1 = log3 x    Add 1.

x= 3y + 1    Write in exponential form.

Again, choose y-values and calculate the corresponding x-values. The graph
is shown in Figure 31. The domain is 10, ∞2, and the range is 1 - ∞, ∞2.
Copyright Pearson. All Rights Reserved.

M04_LHSD7659_12_AIE_C04_pp405-496.indd 440 06/10/15 6:46 PM


NOT FOR SALE
4.3  Logarithmic Functions 441

y
(c) The graph of ƒ1x2 = log4 1x + 22 + 1
is obtained by shifting the graph of
y = log4 x to the left 2 units and up
1 unit. The domain is found by solving 4
f(x)  log4 (x  2)  1
x + 2 7 0,
(1, 1) (2, 2)
which yields 1 - 2, ∞2. The vertical x
asymptote has been shifted to the left 0 2
2 units as well, and it has equation 2

x = - 2. The range is unaffected by


the vertical shift and remains 1 - ∞, ∞2. x  2
See Figure 32.
Figure 32

✔ Now Try Exercises 43, 47, and 61.

Note  If we are given a graph such as the one in Figure 31 and asked to
find its equation, we could reason as follows: The point 11, 02 on the basic
logarithmic graph has been shifted down 1 unit, and the point 13, 02 on the
given graph is 1 unit lower than 13, 12, which is on the graph of y = log3 x.
Thus, the equation will be
y = 1log3 x2 - 1.

Teaching Tip Encourage Properties of Logarithms   The properties of logarithms enable us to


students to memorize the change the form of logarithmic statements so that products can be converted to sums,
properties of logarithms. quotients can be converted to differences, and powers can be converted to products.

Properties of Logarithms

For x 7 0, y 7 0, a 7 0, a ≠ 1, and any real number r, the following


properties hold.
Property Description
Product Property The logarithm of the product of two num-
Teaching Tip  Point out that the log a xy = log a x + log a y bers is equal to the sum of the logarithms
phrase “quotients can be converted
to differences” refers to the
of the numbers.
quotient property. It does NOT Quotient Property The logarithm of the quotient of two num-
say that x bers is equal to the difference between the
log a = log a x − log a y logarithms of the numbers.
log a x y
= log a x - log a y
log a y
Power Property The logarithm of a number raised to a
or log a xr = r log a x power is equal to the exponent multiplied
log a x by the logarithm of the number.
= log a 1x - y2,
log a y
Logarithm of 1 The base a logarithm of 1 is 0.
which are COMMON ERRORS log a 1 = 0
that students make. A quotient or
product of logarithms cannot be Base a Logarithm of a The base a logarithm of a is 1.
simplified using the properties of log a a = 1
logarithms given in the box.

Proof  To prove the product property, let m = loga x and n = loga y.


loga x = m means  a m = x 
Write in exponential form.
loga y = n means a n = y
Copyright Pearson. All Rights Reserved.

M04_LHSD7659_12_AIE_C04_pp405-496.indd 441 06/10/15 6:46 PM


NOT FOR SALE
442 Chapter 4  Inverse, Exponential, and Logarithmic Functions

Now consider the product xy.


Looking Ahead To Calculus xy = a m # a n    x = am and y = an; Substitute.
A technique called logarithmic
differentiation, which uses the xy = a m + n    Product rule for exponents
properties of logarithms, can often be loga xy = m + n    Write in logarithmic form.
used to differentiate complicated
functions. loga xy = loga x + loga y   Substitute.

The last statement is the result we wished to prove. The quotient and power
properties are proved similarly and are left as exercises.

Classroom Example 5 Example 5 Using Properties of Logarithms


Use the properties of logarithms to
rewrite each expression. Assume Use the properties of logarithms to rewrite each expression. Assume all vari-
all variables represent positive real ables represent positive real numbers, with a ≠ 1 and b ≠ 1.
numbers, with a ≠ 1 and b ≠ 1.
(a) log6 17 # 92
15
(b)  log9 (c)  log5 28
(a) log7 18 # 62 (b)  log6
12
7
5
3 5 3 mnq nx 3y 5
(c) log2 29 (d)  loga 2r 3 (d) loga 2 m 2 (e)  loga (f)  logb
p 2t 4 B zm
rs2t mr 3s2
(e) logb (f)  loga Solution
u 3v 5 B t4
(a) log6 17 # 92
15
(b) log9
Answers: 7
Product
(a) log7 8 + log7 6   = log6 7 + log6 9   property Quotient
  = log9 15 - log9 7   property
(b) log6 12 - log6 5
1 3
(c) 3 log2 9 (c) log5 28 (d) loga 2 m2
3
  = log5 181/22  2a = a1/2
n
(d) 5 loga r   = loga m2/3 2am = a m/n
(e) logb r + 2 logb s + logb t -
1 2
  3 logb u - 5 logb v   = log5 8    Power property   = loga m Power property
3 2 4
2 3
(f )  m loga r + m loga s - m loga t
mnq Use parentheses
(e) loga to avoid errors.
p 2t 4
Product and quotient
  = loga m + loga n + loga q - 1loga p 2 + loga t 42    properties
  = loga m + loga n + loga q - 12 loga p + 4 loga t2   Power property
  = loga m + loga n + loga q - 2 loga p - 4 loga t    Distributive property
Be careful
x 3y 5
n with signs.
(f ) logb
B zm
x 3y 5 1/n n
  = logb a b 2a = a 1/n
zm
1 x 3y 5
Teaching Tip  Point out that in   = logb m Power property
n z
Example 5(e), the product and
quotient properties of logarithms 1
were applied before the power   = 1logb x 3 + logb y 5 - logb z m2 Product and quotient properties
n
property. In Example 5(f), the
power property was applied first. 1
In general, properties of
  = 13 logb x + 5 logb y - m logb z2 Power property
n
logarithms should follow the
order of operations with respect 3 5 m
  = logb x + logb y - logb z Distributive property
to exponents, multiplication, and n n n
division. ■
✔ Now Try Exercises 71, 73, and 77.
Copyright Pearson. All Rights Reserved.

M04_LHSD7659_12_AIE_C04_pp405-496.indd 442 06/10/15 6:46 PM


NOT FOR SALE
4.3  Logarithmic Functions 443

Classroom Example 6 Example 6 Using Properties of Logarithms


Write each expression as a single
logarithm with coefficient 1. Write each expression as a single logarithm with coefficient 1. Assume all vari-
Assume all variables represent ables represent positive real numbers, with a ≠ 1 and b ≠ 1.
positive real numbers, with a ≠ 1
(a) log3 1x + 22 + log3 x - log3 2 (b)  2 loga m - 3 loga n
and b ≠ 1.
(a) log4 x - log4 y + log4 z 1 3
(c) logb m + logb 2n - logb m2n
(b) 4 logb r - 5 logb s 2 2
1 2
(c) loga x + loga y - loga xy Solution
3 3
(a) log3 1x + 22 + log3 x - log3 2 (b) 2 loga m - 3 loga n
Answers: Power
xz
(a) log4 y
r4
(b)  logb s5 1x + 22x Product and = loga m2 - loga n3   property

= log3    quotient

3 2 properties m2 Quotient
(c)
2xy 2
loga xy = loga    property
n3
1 3
(c) logb m + logb 2n - logb m2n
2 2
= logb m1/2 + logb 12n23/2 - logb m2n Power property
m1/212n23/2 Use parentheses
= logb around 2n. Product and quotient properties
m 2n
23/2n1/2
= logb Rules for exponents
m3/2
23n 1/2
Classroom Example 7 = logb a b Rules for exponents
Given that log10 7 ≈ 0.8451, find m3
each logarithm without using a
calculator. 8n
= logb Definition of a1/n
(a) log10 49 (b)  log10 70 B m3

✔ Now Try Exercises 83, 87, and 89.
Answers:
(a) 1.6902 (b) 1.8451
Caution  There is no property of logarithms to rewrite a logarithm of
a sum or difference. That is why, in Example 6(a),
log3 1x + 22  cannot be written as  log3 x + log3 2.
The distributive property does not apply here because log3 1x + y2 is one
term. The abbreviation “log” is a function name, not a factor.

Example 7 Using Properties of Logarithms with Numerical Values


Napier’s Rods Given that log10 2 ≈ 0.3010, find each logarithm without using a calculator.
The search for ways to make
(a) log10 4 (b)  log10 5
calculations easier has been a
long, ongoing process. Machines Solution
built by Charles Babbage and
Blaise Pascal, a system of “rods” (a) log10 4 (b) log10 5
used by John Napier, and slide
= log10 22 10
rules were the forerunners of = log10
today’s calculators and computers. = 2 log10 2 2
The invention of logarithms by = log10 10 - log10 2
John Napier in the 16th century ≈ 210.30102
was a great breakthrough in the ≈ 0.6020 ≈ 1 - 0.3010
search for easier calculation
methods. ≈ 0.6990
Source: IBM Corporate Archives. ■
✔ Now Try Exercises 93 and 95.
Copyright Pearson. All Rights Reserved.

M04_LHSD7659_12_AIE_C04_pp405-496.indd 443 06/10/15 6:46 PM


NOT FOR SALE
444 Chapter 4  Inverse, Exponential, and Logarithmic Functions

Note  The values in Example 7 are approximations of logarithms, so the


final digit may differ from the actual 4-decimal-place approximation after
properties of logarithms are applied.

Recall that for inverse functions ƒ and g, 1ƒ ∘ g21x2 = 1g ∘ ƒ21x2 = x. We


can use this property with exponential and logarithmic functions to state two
more properties. If ƒ1x2 = a x and g1x2 = loga x, then

1ƒ ∘ g21x2 = a loga x = x and 1g ∘ ƒ21x2 = loga 1a x2 = x.

Theorem on Inverses

For a 7 0, a ≠ 1, the following properties hold.


Teaching Tip  Point out that the alog a x = x 1 for x + 02  and log a ax = x
theorem on inverses really follows
from the definition of logarithm.
Because loga x is the power to
which a must be raised in order Examples:  7log7 10 = 10,  log5 53 = 3, and logr r k + 1 = k + 1
to obtain x, a raised to that power
will be x. In practice, x will often The second statement in the theorem will be useful when we solve logarithmic
be an algebraic expression. and exponential equations.

4.3 Exercises

1. (a) C (b) A (c) E Concept Preview  Match the logarithm in Column I with its value in Column II.
(d) B (e) F (f ) D Remember that loga x is the exponent to which a must be raised in order to obtain x.
2. (a) F (b) B (c) A I II I II
(d) D (e) C (f ) E 1. (a) log2 16 A. 0 2. (a) log3 81 A. - 2
3. 23 = 8 4.  log10 1000 = 3 1 1
(b) log3 1 B. (b) log3 B. - 1
2 3
5. E 9 F 6.  E 6 F
4 1
(c) log10 0.1 C. 4 (c) log10 0.01 C. 0
(d) log2 22 D. - 3 1
7. y (d) log6 26 D.
2
1
4 f(x) = log 5 x (e) loge E. - 1 9
e2 (e) loge 1 E.
1
x
(f) log1/2 8 F. - 2 2
–2 0 1 5
–2
(f) log3 273/2 F. 4

10, ∞2; 1- ∞, ∞2 Concept Preview  Write each equivalent form.


8. y
3. Write log2 8 = 3 in exponential form. 4. Write 10 3 = 1000 in logarithmic form.
2
5
x
–2 0 1
Concept Preview  Solve each logarithmic equation.
–4 g(x) = log 1/5 x
16 3
5. logx = 2 6. log36 2 6=x
10, ∞2; 1- ∞, ∞2 81

Concept Preview  Sketch the graph of each function. Give the domain and range.

7. ƒ1x2 = log5 x 8. g1x2 = log1/5 x

Copyright Pearson. All Rights Reserved.

M04_LHSD7659_12_AIE_C04_pp405-496.indd 444 06/10/15 6:46 PM


NOT FOR SALE
4.3  Logarithmic Functions 445

9. log10 2 + log10 x - log10 7 Concept Preview  Use the properties of logarithms to rewrite each expression. As-
x3 sume all variables represent positive real numbers.
10. log4 y 5
2x
9. log10 10. 3 log4 x - 5 log4 y
11. log3 81 = 4 12.  log2 32 = 5 7
27
13. log2/3 8 = -3
14. log10 0.0001 = -4 If the statement is in exponential form, write it in an equivalent logarithmic form. If the
15. 6 2 = 36 16.  51 = 5
statement is in logarithmic form, write it in exponential form. See Example 1.
8 1
17. A 23 B = 81 18.  4-3 = 2 -3 27
64 11. 34 = 81 12. 25 = 32 13. a b = 14. 10 -4 = 0.0001
3 8
19. 5 - 46 20.  5 - 46
1
15. log6 36 = 2 16. log5 5 = 1 17. log 23 81 = 8 18. log4 = -3
21.  E F 22.  E 4 F
1 3
2 64
23.  E 4 F 24.  E 5 F
1 1
Solve each equation. See Example 2.
25. 586 26.  556
1 1 1 27
27. 596 28.  5116 19. x = log5 20. x = log3 21. logx = 5 22. logx =3
625 81 32 64
29.  E 5 F 30.  E 4 F
1 1
4 5
23. x = log8 2 8 24. x = log7 2 7 25. x = 3log3 8 26. x = 12log12 5
31. 5646 32.  586
27. x = 2log2 9 28. x = 8log8 11 29. logx 25 = - 2 30. logx 16 = - 2
33.  E 3 F 34.  E 2 F
2 1

3 4
35. 52436 36.  51286 31. log4 x = 3 32. log2 x = 3 33. x = log4 216 34. x = log5 225
37. 5136 38.  536 5 7
39. 536 40.  5236 35. log9 x = 36. log4 x =
2 2
41. 556 42.  566
37. log1/2 1x + 32 = - 4 38. log1/3 1x + 62 = - 2
43. 44. 39. log1x + 32 6 = 1 40. log1x - 42 19 = 1
y y
x = –3
f(x) = (log 2 x) + 3 41. 3x - 15 = logx 1 1x 7 0, x ≠ 12 42. 4x - 24 = logx 1 1x 7 0, x ≠ 12
6 3
4 –2
x
2 0 2 5 Graph each function. Give the domain and range. See Example 4.
x
0 2 4 6 8 43. ƒ1x2 = 1log2 x2 + 3 44. ƒ1x2 = log2 1x + 32 45. ƒ1x2 = 0 log2 1x + 32 0
f(x) = log 2 (x + 3)

10, ∞2; 1- ∞, ∞2 1- 3, ∞2; 1- ∞, ∞2


Graph each function. Give the domain and range. See Example 4.
45. 46.
y y 46. ƒ1x2 = 1log1/2 x2 - 2 47. ƒ1x2 = log1/2 1x - 22 48. ƒ1x2 = 0 log1/2 1x - 22 0
x = –3
4 x
0
–2 2 5 8 Concept Check  In Exercises 49–54, match the function with its graph from choices A–F.
x –4
–2 0 2
f(x) = (log1/2 x) – 2 1
49. ƒ1x2 = log2 x 50. ƒ1x2 = log2 2x 51. ƒ1x2 = log2
x
f(x) = log2 (x + 3)
1
1- 3, ∞2; 30, ∞2 10, ∞2; 1- ∞, ∞2 52. ƒ1x2 = log2 a x b 53. ƒ1x2 = log2 1x - 12 54. ƒ1x2 = log2 1 - x2
2
47. 48.
y y A. y B. y C. y
f(x) = log1/2 (x – 2) x=2

4
2 2 1
x 1 0 1 1 1
0 3 6 9 x x x x
–2 0 3 6 9 0 0
–2 1
x=2 f(x) = log1/2 (x – 2)

12, ∞2; 1- ∞, ∞2 12, ∞2; 30, ∞2


D. y E. y F. y

49. E 50. D
51. B 52. C 1 1 1
x x x
53. F 54. A 0 0 0
2
1 1

Copyright Pearson. All Rights Reserved.

M04_LHSD7659_12_AIE_C04_pp405-496.indd 445 06/10/15 6:46 PM


NOT FOR SALE
446 Chapter 4  Inverse, Exponential, and Logarithmic Functions

55. 56. Graph each function. See Examples 3 and 4.


y y
55. ƒ1x2 = log5 x 56. ƒ1x2 = log10 x 57. ƒ1x2 = log5 1x + 12
f(x) = log 5 x 2
2 1
0
x
0
x 58. ƒ1x2 = log6 1x - 22 59. ƒ1x2 = log1/2 11 - x2 60. ƒ1x2 = log1/3 13 - x2
–2 5 25
–1 5 10
f(x) = log 10 x
–2
61. ƒ1x2 = log3 1x - 12 + 2 62. ƒ1x2 = log2 1x + 22 - 3 63. ƒ1x2 = log1/2 1x + 32 - 2
57. y
64. Concept Check  To graph the function ƒ1x2 = - log5 1x - 72 - 4, reflect the graph
x = –1
4 of y = log5 x across the -axis, then shift the graph units to the right and
2 units down.
x
–2 0 2 4
–3
Connecting Graphs with Equations  Write an equation for the graph given. Each repre-
f(x) = log 5 (x+1) sents a logarithmic function ƒ with base 2 or 3, translated and/or reflected. See the Note
58. y following Example 4.
x=2
4 65. y 66. y 67. y
2
x=3
x x = –1 x=3
–10 3 8
–3 (7, 1) 0
(7, 0) x
x x (–1, 0)
f(x) = log 6 (x – 2) 0 0 (5, 0) (1, –1)
(1, –2) (2, –2)
59. y 60. y (0, –3) (4, –1)
x=1

2 x=3
2
68. y 69. 70. y
( 43 , 1)
x 0 y
0 x
–4 –2 –3 2
–2
(–2, 0)
f(x) = log1/2 (1 – x) (2, 0)
0 x (4, 0)
f(x) = log1/3 (3 – x) x
(1, –2) 0 x
(–1, –1) 0
(4, –1) (–3, –3)
61. 62. (3, –1)
y y
x=1 x =1
x = –2 x=5
x = –3
3 3
2
x x
0 2 4
0 6 Use the properties of logarithms to rewrite each expression. Simplify the result if pos-
3
–3 sible. Assume all variables represent positive real numbers. See Example 5.
f(x) = log3 (x  1)  2 f(x) = log 2 (x + 2) – 3
6x 4p 527
71. log2 72. log3 73. log5
63. y 64. x; 7; 4 y q 3
x = –3
2 223
0 74. log2 75. log4 12x + 5y2 76. log6 17m + 3q2
–4 2 5
x 5
–2

5r 3 3m5n4 ab
f(x) = log1/2 (x + 3) – 2 77. log2 78. log3 79. log2
B z5 B t2 cd
65. ƒ1x2 = log2 1x + 12 - 3
xy 2x # 2y
3 3
2 a # 4
2 b
66. ƒ1x2 = log2 1x - 32 - 1 80. log2 81. log3 82. log4
tqr w 2 2z 2c # 3 2
2d
67. ƒ1x2 = log2 1- x + 32 - 2
68. ƒ1x2 = - log2 1x + 32
Write each expression as a single logarithm with coefficient 1. Assume all variables rep-
69. ƒ1x2 = - log3 1x - 12
resent positive real numbers, with a ≠ 1 and b ≠ 1. See Example 6.
70. ƒ1x2 = - log2 1- x + 52
83. loga x + loga y - loga m 84. logb k + logb m - logb a
71. log2 6 + log2 x - log2 y
85. loga m - loga n - loga t 86. logb p - logb q - logb r
72. log3 4 + log3 p - log3 q
73. 1 +
1
log5 7 - log5 3 1 3 1 2
2 87. logb x 4y 5 - logb x 2y 88. loga p 3q4 - loga p 4q3
1 3 4 2 3
74. 1 + log2 3 - log2 5
2
75. This cannot be simplified. 89. 2 loga 1z + 12 + loga 13z + 22 90. 5 loga 1z + 72 + loga 12z + 92
76. This cannot be simplified. 2 1 3 2
91. - log5 5m2 + log5 25m2 92. - log3 16p 4 - log3 8p 3
3 2 4 3

Copyright Pearson. All Rights Reserved.

M04_LHSD7659_12_AIE_C04_pp405-496.indd 446 06/10/15 6:46 PM


NOT FOR SALE
4.3  Logarithmic Functions 447

1
77. 2 1log2 5 + 3 log2 r - Given that log10 2 ≈ 0.3010 and log10 3 ≈ 0.4771, find each logarithm without using a
calculator. See Example 7.
  5 log2 z2
1
78. 3 15 log3 m + 4 log3 n - 3 2
93. log10 6 94. log10 12 95. log10 96. log10
  2 log3 t2 2 9
79. log2 a + log2 b - log2 c - 9 20
97. log10 98. log10 99. log10 230 100. log10 36 1/3
  log2 d 4 27
80. log2 x + log2 y - log2 t -
  log2 q - log2 r Solve each problem.
1 1
81. 2 log3 x + 3 log3 y - 101. (Modeling) Interest Rates of Treasury Securities The Time Yield
1 table gives interest rates for various U.S. Treasury Securities
2 log3 w -
  log3 z 3-month 0.02%
2 on January 2, 2015.
1
82. 3 log4 a +
1
log4 b - 6-month 0.10%
4 (a) Make a scatter diagram of the data.
1 2 2-year 0.66%
2 log 4 c -
  3 log4 d (b) Which type of function will model this data best: linear,
exponential, or logarithmic? 5-year 1.61%
xy km
83. loga m  84.  logb a
10-year 2.11%
m
85. loga nt  
86.  logb
p 30-year 2.60%
qr
Source:www.federal
87. logb 1x -1/6y 11/122
reserve.gov
88. loga 1p -7/62
102. Concept Check  Use the graph to estimate each y
89. loga 31z + 12213z + 224 logarithm. 1
90. loga 31z + 72512z + 924 (a) log3 0.3    (b)  log3 0.8
0.8
51/3 3 5
91. log5 m1/3 , or log53 m
0.6
1 y = 3x
92. log3 32p 5
0.4

93. 0.7781   94. 1.0791 0.2


95. 0.1761   96.  - 0.6532 x
97. 0.3522   98.  - 0.1303 –2 –1.5 –1 –0.5 0
99. 0.7386 100. 0.5187
103. Concept Check Suppose ƒ1x2 = loga x and ƒ132 = 2. Determine each function
101. (a) 3 value.
1 23
(a)  ƒ a b     (b)  ƒ1272    (c)  ƒ192    (d)  ƒ ¢ ≤
9 3
−5 35

−1 104. Use properties of logarithms to evaluate each expression.


(b) logarithmic (a)  100 log10 3  (b)  log10 10.0123  (c)  log10 10.000125  (d)  1000 log10 5
102. (a) - 1.1 (b)  - 0.2 r tn
105. Using the compound interest formula A = P A 1 + B
n , show that the amount of
103. (a) - 4 (b) 6
time required for a deposit to double is
(c) 4 (d)  - 1
104. (a) 9 (b)  - 6 1
.
(c)  - 20 (d) 125 log2 A 1 +
r n
nB
106. 4 = loga 5
106. Concept Check If 15, 42 is on the graph of the logarithmic function with base a,
107. 50.01, 2.386 108.  51.876 which of the following statements is true:

5 = loga 4 or 4 = loga 5?

Use a graphing calculator to find the solution set of each equation. Give solutions to the
nearest hundredth.
107. log10 x = x - 2 108. 2-x = log10 x
x
109. Prove the quotient property of logarithms:  loga y = loga x - loga y.

110. Prove the power property of logarithms:  loga x r = r loga x.

Copyright Pearson. All Rights Reserved.

M04_LHSD7659_12_AIE_C04_pp405-496.indd 447 06/10/15 6:47 PM


NOT FOR SALE
448 Chapter 4  Inverse, Exponential, and Logarithmic Functions

Summary Exercises on Inverse, Exponential, and Logarithmic


Functions
1. They are inverses. The following exercises are designed to help solidify your understanding of inverse,
2. They are not inverses. exponential, and logarithmic functions from Sections 4.1–4.3.
3. They are inverses.
4. They are inverses. Determine whether the functions in each pair are inverses of each other.
1 4 1
5. y
y=x
6. y 1. ƒ1x2 = 3x - 4, g1x2 = x+ 2. ƒ1x2 = 8 - 5x, g1x2 = 8 + x
y=x 3 3 5

0 0 3. ƒ1x2 = 1 + log2 x, g1x2 = 2x - 1 4. ƒ1x2 = 3x/5 - 2, g1x2 = 5 log3 1x + 22


x x

7. It is not one-to-one.
Determine whether each function is one-to-one. If it is, then sketch the graph of its
8. y inverse function.
y=x
5. y 6.  y
y=x y=x
0 x

x x
1
9. B 10. D
11. C 12. A

13. The functions in Exercises 9 7. y 8.  y


and 12 are inverses of one y=x y=x
another. The functions in
Exercises 10 and 11 are
inverses of one another. x x

14. ƒ -11x2 = 5x

1
15. ƒ -11x2 = 3 x + 2;
Domains and ranges of both ƒ
and ƒ -1 are 1- ∞, ∞2. In Exercises 9–12, match each function with its graph from choices A–D.
9. y = log3 1x + 22 10. y = 5 - 2x
11. y = log2 15 - x2 12. y = 3x - 2
A.  y B. y

1 1
x x
0 0
1 1

C.  y D. y

1
x
0
1 1
x
0
1

13. The functions in Exercises 9–12 form two pairs of inverse functions. Determine
which functions are inverses of each other.
14. Determine the inverse of the function ƒ1x2 = log5 x. (Hint: Replace ƒ1x2 with y,
and write in exponential form.)

Copyright Pearson. All Rights Reserved.

M04_LHSD7659_12_AIE_C04_pp405-496.indd 448 06/10/15 6:47 PM


NOT FOR SALE
4.4  Evaluating Logarithms and the Change-of-Base Theorem 449

3 x For each function that is one-to-one, write an equation for the inverse function. Give the
16. ƒ -11x2 = 3 2 - 1;
domain and range of both ƒ and ƒ -1. If the function is not one-to-one, say so.
Domains and ranges of both ƒ
and ƒ -1 are 1- ∞, ∞2. 15. ƒ1x2 = 3x - 6 16. ƒ1x2 = 21x + 123
17. ƒ is not one-to-one.
2x - 1
5x + 1
18. ƒ -11x2 = 2 + 3x ; 17. ƒ1x2 = 3x 2 18. ƒ1x2 =
5 - 3x
Domain of ƒ = range of
3
5
ƒ -1 = A - ∞, 3 B ´ A 53 , ∞ B . 19. ƒ1x2 = 2 5 - x 4 20. ƒ1x2 = 2x 2 - 9,  x Ú 3
Domain of ƒ -1 = range of
2 2
ƒ = A - ∞, - 3 B ´ A - 3 , ∞ B . Write an equivalent statement in logarithmic form.
19. ƒ is not one-to-one. 1 -3 4
20. ƒ -11x2 = 2x 2 + 9, x Ú 0; 21. a b = 1000 22. a b = c 23. A 23 B = 9
10
Domain of ƒ = range of
ƒ -1 = 33, ∞2. 1
24. 4-3/2 = 25. 2x = 32 26. 274/3 = 81
Domain of ƒ -1 = range of 8
ƒ = 30, ∞2.

21. log1/10 1000 = - 3 Solve each equation.


22. loga c = b
23. log 23 9 = 4
1
27. 3x = 7log7 6 28. x = log10 0.001 29. x = log6
216
1 3
24. log4 8 = - 2
1
25. log2 32 = x 30. logx 5 = 31. log10 0.01 = x 32. logx 3 = - 1
4 2
26. log27 81 = 3
3 1
27. 526 28.  5 - 36 33. logx 1 = 0 34. x = log2 28 35. logx 2 5=
3
29. 5 - 36 30.  5256
25
31. 5 - 26 32.  E 3 F
1 36. log1/3 x = - 5 37. log10 1log2 2102 = x 38. x = log4/5
16
33.  10, 12 ´ 11, ∞2 34.  E 2 F
3
1
39. 2x - 1 = log6 6 x 40. x = log1/2 41. 2x = log2 16
35. 556 36.  52436 B 16
37. 516 38.  5 - 26 1 x+1
42. log3 x = - 2 43. a b = 9 x 44. 52x - 6 = 25x - 3
39. 516 40.  526 3
42.  E 9 F
1
41. 526

43.  E - 3 F
1
44.  1- ∞, ∞2

4.4 Evaluating Logarithms and the Change-of-Base Theorem

■ Common Logarithms
Common Logarithms   Two of the most important bases for logarithms
■ Applications and are 10 and e. Base 10 logarithms are common logarithms. The common loga-
Models with Common
rithm of x is written log x, where the base is understood to be 10.
Logarithms
■ Natural Logarithms
■ Applications and
Models with Natural
Common Logarithm
Logarithms
For all positive numbers x,
■ Logarithms with Other
Bases log x = log10 x.

A calculator with a log key can be used to find the base 10 logarithm of any
positive number.
Copyright Pearson. All Rights Reserved.

M04_LHSD7659_12_AIE_C04_pp405-496.indd 449 06/10/15 6:47 PM


NOT FOR SALE
450 Chapter 4  Inverse, Exponential, and Logarithmic Functions

Classroom Example 1 Example 1 Evaluating Common Logarithms with a Calculator


Use a calculator to find the values
of log 10,000, log 341, and Use a calculator to find the values of
log 0.06894.
log 1000,  log 142, and log 0.005832.
Answers:  4; 2.532754379;
- 1.161528721 Solution  Figure 33 shows that the exact
value of log 1000 is 3 (because 10 3 = 1000),
and that

log 142 ≈ 2.152288344


and log 0.005832 ≈ - 2.234182485.

Most common logarithms that appear in cal-


culations are approximations, as seen in the
second and third displays. Figure 33


✔ Now Try Exercises 11, 15, and 17.

For a + 1, base a logarithms of numbers between 0 and 1 are always


negative, and base a logarithms of numbers greater than 1 are always positive.

Applications and Models with Common Logarithms  In chemistry,


the pH of a solution is defined as
pH = − log 3 H3o+ 4 ,
where 3H3O+ 4 is the hydronium ion concentration in moles* per liter. The
pH value is a measure of the acidity or alkalinity of a solution. Pure water has pH
7.0, substances with pH values greater than 7.0 are alkaline, and substances with
pH values less than 7.0 are acidic. See Figure 34. It is customary to round pH
values to the nearest tenth.

1 7 14

Acidic Neutral Alkaline

Figure 34

Classroom Example 2 Example 2 Finding pH


(a) Find the pH of a solution with
(a) Find the pH of a solution with 3H3O+ 4 = 2.5 * 10 -4.
3H3O+ 4 = 6.8 * 10 -8.
(b) Find the hydronium ion (b) Find the hydronium ion concentration of a solution with pH = 7.1.
concentration of a solution
with pH = 4.3. Solution

Answers: (a) pH = - log3H3O+ 4


(a) 7.2 (b)  5.0 * 10 -5 pH = - log12.5 * 10 -42 Substitute 3H3O+ 4 = 2.5 * 10 -4.

pH = - 1log 2.5 + log 10 -42 Product property


pH = - 10.3979 - 42 log 10 -4 = -4
pH = - 0.3979 + 4 Distributive property

pH ≈ 3.6 Add.

*A mole is the amount of a substance that contains the same number of molecules as the number of atoms
in exactly 12 grams of carbon-12.

Copyright Pearson. All Rights Reserved.

M04_LHSD7659_12_AIE_C04_pp405-496.indd 450 06/10/15 6:47 PM


NOT FOR SALE
4.4  Evaluating Logarithms and the Change-of-Base Theorem 451

(b) pH = - log3H3O+ 4
7.1 = - log3H3O+ 4 Substitute pH = 7.1.
- 7.1 = log3H3O+ 4 Multiply by -1.
3H3O+ 4 = 10 -7.1 Write in exponential form.

3H3O+ 4 ≈ 7.9 * 10 -8 Evaluate 10 -7.1 with a calculator.



✔ Now Try Exercises 29 and 33.

Note  In the fourth line of the solution in Example 2(a), we use the
equality symbol, =, rather than the approximate equality symbol, ≈ , when
replacing log 2.5 with 0.3979. This is often done for convenience, despite
the fact that most logarithms used in applications are indeed approximations.

Example 3 Using pH in an Application

Wetlands are classified as bogs, fens, marshes, and swamps based on pH values.
A pH value between 6.0 and 7.5 indicates that the wetland is a “rich fen.” When
the pH is between 3.0 and 6.0, it is a “poor fen,” and if the pH falls to 3.0 or less,
the wetland is a “bog.” (Source: R. Mohlenbrock, “Summerby Swamp, Michigan,”
Natural History.)
Suppose that the hydronium ion concentration of a sample of water from a
wetland is 6.3 * 10 -5. How would this wetland be classified?
Solution pH = - log3H3O+ 4 Definition of pH

pH = - log16.3 * 10 -52 Substitute for 3H3O+ 4.


Classroom Example 3
Refer to Example 3. The pH = - 1log 6.3 + log 10 -52 Product property
hydronium ion concentration of
a water sample from a wetland is pH = - log 6.3 - 1 - 52 Distributive property; log 10 n = n
4.5 * 10 -3. Classify this wetland. pH = - log 6.3 + 5 Definition of subtraction
Answer:  bog
pH ≈ 4.2 Use a calculator.

The pH is between 3.0 and 6.0, so the wetland is a poor fen.



✔ Now Try Exercise 37.

Classroom Example 4 Example 4 Measuring the Loudness of Sound


Find the decibel rating of a sound
with intensity 10,000,000I0 . The loudness of sounds is measured in decibels. We first assign an intensity of
I0 to a very faint threshold sound. If a particular sound has intensity I, then the
Answer: 70
decibel rating d of this louder sound is given by the following formula.
I
d = 10 log
I0
Find the decibel rating d of a sound with intensity 10,000I0 .
10,000I0
Solution d = 10 log Let I = 10,000I0.
I0
I0
d = 10 log 10,000 I0
=1

d = 10142 log 10,000 = log 10 4 = 4


d = 40 Multiply.

The sound has a decibel rating of 40. ■


✔ Now Try Exercise 63.
Copyright Pearson. All Rights Reserved.

M04_LHSD7659_12_AIE_C04_pp405-496.indd 451 06/10/15 6:47 PM


NOT FOR SALE
452 Chapter 4  Inverse, Exponential, and Logarithmic Functions

Natural Logarithms   In most practical applications of logarithms, the


irrational number e is used as the base. Logarithms with base e are natural
­logarithms because they occur in the life sciences and economics in natural
­situations that involve growth and decay. The base e logarithm of x is written
ln x (read “el-en x”). The expression ln x represents the exponent to which e
must be raised in order to obtain x.

y Natural Logarithm

f(x) = ln x For all positive numbers x,


2

0
x ln x = log e x.
2 4 6 8
–2

Figure 35 A graph of the natural logarithmic function ƒ1x2 = ln x is given in Figure 35.

Classroom Example 5 Example 5 Evaluating Natural Logarithms with a Calculator


Use a calculator to find the values
of ln e4, ln 341, and ln 0.06894. Use a calculator to find the values of
Answers:  4; 5.831882477; ln e3, ln 142,  and  ln 0.005832.
- 2.674518718
Solution  Figure 36 shows that the exact
value of ln e3 is 3, and that

ln 142 ≈ 4.955827058
and ln 0.005832 ≈ - 5.144395284.

Figure 36


✔ Now Try Exercises 45, 51, and 53.

Figure 37 Figure 37 illustrates that ln x is the exponent to which e must be raised in


order to obtain x.

Applications and Models with Natural Logarithms

Classroom Example 6 Example 6 Measuring the Age of Rocks


Refer to the model given in
Example 6. Geologists sometimes measure the age of rocks by using “atomic clocks.” By
(a) How old is a rock in which measuring the amounts of argon-40 and potassium-40 in a rock, it is possible to
A = K? find the age t of the specimen in years with the formula
(b) How old is a rock in which A
A
K = 0.325?
ln A 1 + 8.33 A K B B
t = 11.26 * 10 92 ,
Answers: ln 2
(a) 4.06 billion yr old
where A and K are the numbers of atoms of argon-40 and potassium-40, respec-
(b) 2.38 billion yr old
tively, in the specimen.
(a) How old is a rock in which A = 0 and K 7 0?
A
(b) The ratio K for a sample of granite from New Hampshire is 0.212. How old
is the sample?
Copyright Pearson. All Rights Reserved.

M04_LHSD7659_12_AIE_C04_pp405-496.indd 452 06/10/15 6:47 PM


NOT FOR SALE
4.4  Evaluating Logarithms and the Change-of-Base Theorem 453

Solution
A
Looking Ahead To Calculus (a) If A = 0, then K = 0 and the equation is as follows.
The natural logarithmic function
A
ƒ1x2 = ln x and the reciprocal function ln A 1 + 8.33 A K B B
1
g1x2 = x have an important relation- t = 11.26 * 10 92 Given formula
ship in calculus. The derivative of the
ln 2
natural logarithmic function is the ln 1 A
reciprocal function. Using Leibniz t = 11.26 * 10 92 K = 0, so ln 11 + 02 = ln 1
ln 2
notation (named after one of the
co-inventors of calculus), we write t = 11.26 * 10 92102 ln 1 = 0
d 1
this fact as dx 1ln x2 = x . t=0

The rock is new (0 yr old).


A
(b) Because K = 0.212, we have the following.

ln 11 + 8.3310.21222
t = 11.26 * 10 92    Substitute.
ln 2
t ≈ 1.85 * 10 9    Use a calculator.

The granite is about 1.85 billion yr old. ■


✔ Now Try Exercise 77.

Classroom Example 7 Example 7 Modeling Global Temperature Increase


Refer to the equation given in
Example 7. Suppose that C = 2C0 Carbon dioxide in the atmosphere traps heat from the sun. The additional solar
and k = 14. radiation trapped by carbon dioxide is radiative forcing. It is measured in
(a) Find the radiative forcing watts per square meter 1w/m22. In 1896 the Swedish scientist Svante Arrhenius
given these conditions. modeled radiative forcing R caused by additional atmospheric carbon dioxide,
(b) Find the average global using the logarithmic equation
temperature increase to the C
nearest degree Fahrenheit R = k ln ,
under the same conditions.
C0

Answers:  where C0 is the preindustrial amount of carbon dioxide, C is the current carbon
(a) 9.7 w/m2 (b)  10°F
dioxide level, and k is a constant. Arrhenius determined that 10 … k … 16 when
C = 2C0 . (Source: Clime, W., The Economics of Global Warming, Institute for
International Economics, Washington, D.C.)
(a) Let C = 2C0. Is the relationship between R and k linear or logarithmic?
(b) The average global temperature increase T (in °F) is given by T1R2 = 1.03R.
Write T as a function of k.
Solution
C
(a) If C = 2C0 , then C = 2, so R = k ln 2 is a linear relation, because ln 2 is a
0
constant.

(b) T1R2 = 1.03R


C
T1k2 = 1.03k ln    Use the given expression for R.
C0
■✔ Now Try Exercise 75.

Logarithms with Other Bases   We can use a calculator to find the values
of either natural logarithms (base e) or common logarithms (base 10). However,
sometimes we must use logarithms with other bases. The change-of-base
theorem can be used to convert logarithms from one base to another.
Copyright Pearson. All Rights Reserved.

M04_LHSD7659_12_AIE_C04_pp405-496.indd 453 06/10/15 6:47 PM


NOT FOR SALE
454 Chapter 4  Inverse, Exponential, and Logarithmic Functions

Change-of-Base Theorem
Looking Ahead To Calculus
In calculus, natural logarithms are For any positive real numbers x, a, and b, where a ≠ 1 and b ≠ 1, the fol-
more convenient to work with than lowing holds.
logarithms with other bases. The
log b x
change-of-base theorem enables us to log a x =
convert any logarithmic function to a
log b a
natural logarithmic function.

Proof Let y = loga x.


Then ay = x Write in exponential form.

logb ay = logb x Take the base b logarithm on each side.

y logb a = logb x Power property


logb x
y= Divide each side by logb a.
logb a
logb x
loga x = . Substitute loga x for y.
logb a
Any positive number other than 1 can be used for base b in the change-of-base
theorem, but usually the only practical bases are e and 10 since most calculators
Teaching Tip  Show students
that log2 10 can be evaluated using give logarithms for these two bases only.
log 10 ln 10   Using the change-of-base theorem, we can graph an equation such as
 or  . log x ln x
log 2 ln 2 y = log2 x  by directing the calculator to graph y = log 2 , or, equivalently, y = ln 2 .  ■

Classroom Example 8 Example 8 Using the Change-of-Base Theorem


Use the change-of-base theorem
to find an approximation to four Use the change-of-base theorem to find an approximation to four decimal places
decimal places for each logarithm. for each logarithm.
(a) log4 20 (b)  log2 0.7 (a) log5 17 (b)  log2 0.1
Answers:
Solution
(a) 2.1610 (b)  - 0.5146
(a) We use natural logarithms to approximate this logarithm. Because log5 5 = 1
and log5 25 = 2, we can estimate log5 17 to be a number between 1 and 2.
ln 17
log5 17 = ≈ 1.7604 Check: 51.7604 ≈ 17
ln 5

The first two entries in Figure 38(a) show that the results are the same
whether natural or common logarithms are used.

(a) (b)

Figure 38

Copyright Pearson. All Rights Reserved.

M04_LHSD7659_12_AIE_C04_pp405-496.indd 454 06/10/15 6:47 PM


NOT FOR SALE
4.4  Evaluating Logarithms and the Change-of-Base Theorem 455

(b) We use common logarithms for this approximation.

log 0.1
log2 0.1 = ≈ - 3.3219 Check: 2 -3.3219 ≈ 0.1
log 2
The last two entries in Figure 38(a) show that the results are the same whether
natural or common logarithms are used.
Some calculators, such as the TI-84 Plus, evaluate these logarithms directly
without using the change-of-base theorem. See Figure 38(b).

✔ Now Try Exercises 79 and 81.

Example 9 Modeling Diversity of Species

One measure of the diversity of the species in an ecological community is mod-


eled by the formula

H = - 3P1 log2 P1 + P2 log2 P2 + g + Pn log2 Pn 4,

where P1, P2, c, Pn are the proportions of a sample that belong to each of
n species found in the sample. (Source: Ludwig, J., and J. Reynolds, Statistical
Ecology: A Primer on Methods and Computing, © 1988, John Wiley & Sons, NY.)
Find the measure of diversity in a community with two species where there
are 90 of one species and 10 of the other.
90
Solution  There are 100 members in the community, so P1 = 100 = 0.9 and
10
P2 = 100 = 0.1.

H = - 30.9 log2 0.9 + 0.1 log2 0.14   Substitute for P1 and P2.
Classroom Example 9
Refer to the formula in Example 9. In Example 8(b), we found that log2 0.1 ≈ - 3.32. Now we find log2 0.9.
Find the measure of diversity in a
community with two species where log 0.9
there are 60 of one species and 140 log2 0.9 = ≈ - 0.152  Change-of-base theorem
of the other.
log 2

Answer:  0.881 Now evaluate H.

H = - 30.9 log2 0.9 + 0.1 log2 0.14


H ≈ - 30.91 - 0.1522 + 0.11 - 3.3224 Substitute approximate values.
H ≈ 0.469 Simplify.

Verify that H ≈ 0.971 if there are 60 of one species and 40 of the other.
1
As the proportions of n species get closer to n each, the measure of diversity
increases to a maximum of log2 n.

✔ Now Try Exercise 73.

  We saw previously that graphing calculators are capable of fitting expo-


y = 1.479 + 0.809 ln x nential curves to data that suggest such behavior. The same is true for logarithmic
curves. For example, during the early 2000s on one particular day, interest rates
5 for various U.S. Treasury Securities were as shown in the table.

Time 3-mo 6-mo 2-yr 5-yr 10-yr 30-yr


Yield 0.83% 0.91% 1.35% 2.46% 3.54% 4.58%
−5 35
Source: U.S. Treasury.
−2
Figure 39 shows how a calculator gives the best-fitting natural logarithmic curve
Figure 39 for the data, as well as the data points and the graph of this curve.  ■
Copyright Pearson. All Rights Reserved.

M04_LHSD7659_12_AIE_C04_pp405-496.indd 455 23/10/15 2:02 pm


NOT FOR SALE
456 Chapter 4  Inverse, Exponential, and Logarithmic Functions

4.4 Exercises

1. increasing Concept Preview  Answer each of the following.


2. increasing
3. ƒ - 11x2 = log5 x 1. For the exponential function ƒ1x2 = ax, where a 7 1, is the function increasing or
4. log4 11 decreasing over its entire domain?
5. natural; common 2. For the logarithmic function g1x2 = loga x, where a 7 1, is the function increasing
ln 12
6. ln 3 or decreasing over its entire domain?
7. There is no power of 2 that
3. If ƒ1x2 = 5x, what is the rule for ƒ - 11x2?
yields a result of 0.
8. 3 and 4 4. What is the name given to the exponent to which 4 must be raised to obtain 11?
9. log 8 = 0.90308999
10. ln 2.75 = 1.0116009 5. A base e logarithm is called a(n) logarithm, and a base 10 logarithm is
called a(n) logarithm.
11. 12 12. 7
6. How is log3 12 written in terms of natural logarithms using the change-of-base
13. - 1 14.  - 2 
15. 1.7993 16. 1.9731
theorem?
17. - 2.6576 18.  - 2.2596 7. Why is log2 0 undefined?
19. 3.9494 20. 3.5505
21. 0.1803 22. 0.3503 8. Between what two consecutive integers must log2 12 lie?
23. 3.9494 24. 3.5505
9. The graph of y = log x shows a point on the graph. Write the logarithmic equation
25. 0.1803 26. 0.3503
associated with that point.
27. The logarithm of the product y
of two numbers is equal to the
(8, 0.90308999)
sum of the logarithms of the 1
numbers.
28. The logarithm of the quotient x
0
4 8
of two numbers is equal to y = log x
–1
the difference between the
logarithm of the numerator
and the logarithm of the 10. The graph of y = ln x shows a point on the graph. Write the logarithmic equation
denominator. associated with that point.
y

1 (2.75, 1.0116009)
x
0
2 4 6 8
–1
y = ln x

Find each value. If applicable, give an approximation to four decimal places. See
Example 1.
11. log 10 12 12. log 10 7 13. log 0.1 14. log 0.01
15. log 63 16. log 94 17. log 0.0022 18. log 0.0055
518 643
19. log 1387 * 232 20. log 1296 * 122 21. log 22. log
342 287
23. log 387 + log 23 24. log 296 + log 12
25. log 518 - log 342 26. log 643 - log 287

Answer each question.


27. Why is the result in Exercise 23 the same as that in Exercise 19?
28. Why is the result in Exercise 25 the same as that in Exercise 21?

Copyright Pearson. All Rights Reserved.

M04_LHSD7659_12_AIE_C04_pp405-496.indd 456 06/10/15 6:47 PM


NOT FOR SALE
4.4  Evaluating Logarithms and the Change-of-Base Theorem 457

29. 3.2 30. 1.8 For each substance, find the pH from the given hydronium ion concentration. See
31. 8.4 32. 13.5 Example 2(a).

33. 2.0 * 10 -3 34.  4.0 * 10 -4 29. grapefruit, 6.3 * 10 -4 30. limes, 1.6 * 10 -2
35. 1.6 * 10 -5 36.  3.2 * 10 -7
31. crackers, 3.9 * 10 -9 32. sodium hydroxide (lye), 3.2 * 10 -14
37. poor fen 38.  poor fen
39. bog 40. bog Find the 3H3O+ 4 for each substance with the given pH. See Example 2(b).
41. rich fen 42.  rich fen
33. soda pop, 2.7 34. wine, 3.4
43. (a) 2.60031933 35. beer, 4.8 36. drinking water, 6.5
(b) 1.60031933
(c) 0.6003193298
Suppose that water from a wetland area is sampled and found to have the given hydro-
(d) The whole number parts
nium ion concentration. Determine whether the wetland is a rich fen, a poor fen, or a bog.
will vary, but the decimal
See Example 3.
parts will be the same.
44. 4.3979; Each can be written 37. 2.49 * 10 -5 38. 6.22 * 10 -5 39. 2.49 * 10 -2
as log 25 plus a logarithm of a
power of 10. 40. 3.14 * 10 -2 41. 2.49 * 10 -7 42. 5.86 * 10 -7

45. 1.6 46. 5.8 Solve each problem.


47. - 2 48.  - 4
1 1 43. Use a calculator to find an approximation for each logarithm.
49. 2 50.  3
51. 3.3322 52. 3.6636 (a)  log 398.4 (b)  log 39.84 (c)  log 3.984
53. - 8.9480 54.  - 4.8665 (d) From the answers to parts (a)–(c), make a conjecture concerning the decimal
55. 10.1449 56. 9.8386 values in the approximations of common logarithms of numbers greater than 1
57. 2.0200 58. 1.5976 that have the same digits.
59. 10.1449 60. 9.8386
61. 2.0200 62. 1.5976 44. Given that log 25 ≈ 1.3979, log 250 ≈ 2.3979, and log 2500 ≈ 3.3979, make a
conjecture for an approximation of log 25,000. Why does this pattern continue?
63. (a)  20 (b) 30 (c) 50
(d)  60 (e)  3 decibels Find each value. If applicable, give an approximation to four decimal places. See
64. (a)  21 (b)  70 (c)  91 Example 5.
(d)  120 (e)  140
1
45. ln e1.6 46. ln e5.8 47. ln
e2
1 3
48. ln 49. ln 2e 50. ln 2 e
e4
51. ln 28 52. ln 39 53. ln 0.00013
54. ln 0.0077 55. ln 127 * 9432 56. ln 133 * 5682
98 84
57. ln 58. ln 59. ln 27 + ln 943
13 17
60. ln 33 + ln 568 61. ln 98 - ln 13 62. ln 84 - ln 17

Solve each problem. See Examples 4, 6, 7, and 9.


63. Decibel Levels  Find the decibel ratings of sounds having the following intensities.
(a) 100I0 (b)  1000I0 (c)  100,000I0 (d)  1,000,000I0
(e) If the intensity of a sound is doubled, by how much is the decibel rating
increased? Round to the nearest whole number.
64. Decibel Levels  Find the decibel ratings of the following sounds, having intensities
as given. Round each answer to the nearest whole number.
(a) whisper, 115I0 (b) busy street, 9,500,000I0
(c) heavy truck, 20 m away, 1,200,000,000I0
(d) rock music, 895,000,000,000I0
(e) jetliner at takeoff, 109,000,000,000,000I0

Copyright Pearson. All Rights Reserved.

M04_LHSD7659_12_AIE_C04_pp405-496.indd 457 06/10/15 6:47 PM


NOT FOR SALE
458 Chapter 4  Inverse, Exponential, and Logarithmic Functions

65. (a) 3 (b) 6 (c) 8 65. Earthquake Intensity  The magnitude of an earthquake, measured on the Richter
66. 1,258,900,000I0 I
scale, is log10 I0 , where I is the amplitude registered on a seismograph 100 km from
67. 631,000,000I0
the epicenter of the earthquake, and I0 is the amplitude of an earthquake of a certain
68. 1.995 times greater
(small) size. Find the Richter scale ratings for earthquakes having the following
69. 106.6 thousand; We must amplitudes.
assume that the model
continues to be logarithmic. (a)  1000I0 (b)  1,000,000I0 (c)  100,000,000I0
70. 1590 million person-trips; 66. Earthquake Intensity  On December 26, 2004, an earthquake struck in the Indian
This is very close to the actual Ocean with a magnitude of 9.1 on the Richter scale. The resulting tsunami killed an
number. estimated 229,900 people in several countries. Express this reading in terms of I0 to
the nearest hundred thousand.
67. Earthquake Intensity  On February 27, 2010, a massive earthquake struck Chile
with a magnitude of 8.8 on the Richter scale. Express this reading in terms of I0 to
the nearest hundred thousand.
68. Earthquake Intensity Comparison  Compare the answers to Exercises 66 and 67.
How many times greater was the force of the 2004 earthquake than that of the 2010
earthquake?
69. (Modeling) Bachelor’s Degrees in Psychology  The table gives the number of
bachelor’s degrees in psychology (in thousands) earned at U.S. colleges and univer-
sities for selected years from 1980 through 2012. Suppose x represents the number
of years since 1950. Thus, 1980 is represented by 30, 1990 is represented by 40, and
so on.

Degrees Earned
Year (in thousands)
1980  42.1
1990  54.0
2000  74.2
2010  97.2
2011 100.9
2012 109.0
Source: National Center for
Education Statistics.

The following function is a logarithmic model for the data.

ƒ1x2 = - 273 + 90.6 ln x

Use this function to estimate the number of bachelor’s degrees in psychology earned
in the year 2016 to the nearest tenth thousand. What assumption must we make to
estimate the number of degrees in years beyond 2012?
70. (Modeling) Domestic Leisure Travel  U.S. Domestic Leisure Travel Volume
The bar graph shows numbers of 2000
Person-Trips (in millions)

leisure trips within the United States


(in millions of person-trips of 50 or 1500
more miles one-way) over the years
1000
2009–2014. The function
500
ƒ1t2 = 1458 + 95.42 ln t, t Ú 1,
0
where t represents the number of years 2009 2010 2011 2012 2013 2014
since 2008 and ƒ1t2 is the number Year
of person-trips, in millions, approxi- Source: Statista 2014.
mates the curve reasonably well.
  Use the function to approximate the number of person-trips in 2012 to the nearest
million. How does this approximation compare to the actual number of 1588 million?

Copyright Pearson. All Rights Reserved.

M04_LHSD7659_12_AIE_C04_pp405-496.indd 458 06/10/15 6:47 PM


NOT FOR SALE
4.4  Evaluating Logarithms and the Change-of-Base Theorem 459

71. (a)  2 (b)  2 (c)  2 (d)  1 71. (Modeling) Diversity of Species  The number of species S1n2 in a sample is
72. (a)  4 (b)  4 (c)  5 given by
73. 1 74. 1.589
75. between 7°F and 11°F n
S1n2 = a ln a1 + b,
76. (a) T1x2 = a
35311.0062x - 1990
6.489 ln C 280 D
where n is the number of individuals in the sample, and a is a constant that indicates
(b) C(x) = 353(1.006) x − 1990 the diversity of species in the community. If a = 0.36, find S1n2 for each value of n.
2100 (Hint: S1n2 must be a whole number.)
(a) 100 (b) 200 (c) 150 (d) 10
72. (Modeling) Diversity of Species In Exercise 71, find S1n2 if a changes to 0.88. Use
1990 2275
the following values of n.
300
(a) 50 (b) 100 (c) 250
C
T(x) = 6.489 ln , where 73. (Modeling) Diversity of Species Suppose a sample of a small community shows
280
C(x) is defined as given earlier. two species with 50 individuals each. Find the measure of diversity H.
15
74. (Modeling) Diversity of Species  A virgin forest in northwestern Pennsylvania has
4 species of large trees with the following proportions of each:

hemlock, 0.521;  beech, 0.324;  birch, 0.081;  maple, 0.074.


1990 2275
0
Find the measure of diversity H to the nearest thousandth.
 C is an exponential
function, and T is a 75. (Modeling) Global Temperature Increase In Example 7, we expressed the aver-
linear function over the age global temperature increase T (in °F) as
same time period. While
the carbon dioxide levels C
T1k2 = 1.03k ln ,
in the atmosphere C0
increase at an exponential
rate, the average global where C0 is the preindustrial amount of carbon dioxide, C is the current carbon
temperature will rise at dioxide level, and k is a constant. Arrhenius determined that 10 … k … 16 when C
a linear rate. was double the value C0 . Use T1k2 to find the range of the rise in global tempera-
(c) The slope is 0.0388. ture T (rounded to the nearest degree) that Arrhenius predicted. (Source: Clime,
This means that the W., The Economics of Global Warming, Institute for International Economics,
temperature is expected Washington, D.C.)
to rise at an average
rate of 0.04°F per year 76. (Modeling) Global Temperature Increase  (Refer to Exercise 75.) According to
from 1990 to 2275. one study by the IPCC, future increases in average global temperatures (in °F) can
(d) x ≈ 2208.9; be modeled by
 C ≈ 1307 ppm (about
4.67 times above C
T1C2 = 6.489 ln ,
preindustrial carbon 280
dioxide levels)
77. 1.13 billion yr where C is the concentration of atmospheric carbon dioxide (in ppm). C can be
modeled by the function

C1x2 = 35311.0062x - 1990,

where x is the year. (Source: International Panel on Climate Change (IPCC).)


(a) Write T as a function of x.
(b) Using a graphing calculator, graph C1x2 and T1x2 on the interval [1990, 2275]
using different coordinate axes. Describe the graph of each function. How are C
and T related?
(c) Approximate the slope of the graph of T. What does this slope represent?
(d) Use graphing to estimate x and C1x2 when T1x2 = 10°F.
77. Age of Rocks  Use the formula of Example 6 to estimate the age of a rock sample
A
having K = 0.103. Give the answer in billions of years, rounded to the nearest
hundredth.

Copyright Pearson. All Rights Reserved.

M04_LHSD7659_12_AIE_C04_pp405-496.indd 459 06/10/15 6:47 PM


NOT FOR SALE
460 Chapter 4  Inverse, Exponential, and Logarithmic Functions

78. (a)  6 78. (Modeling) Planets’ Distances from the Sun Planet D P
and Periods of Revolution  The table contains
the planets’ average distances D from the sun and Mercury  0.39   0.24
their periods P of revolution around the sun in Venus  0.72   0.62
−2 4
years. The distances have been normalized so that Earth  1   1
−2 Earth is one unit away from the sun. For example, Mars  1.52   1.89
Let x = ln D and y = ln P since Jupiter’s distance is 5.2, its distance from
for each planet. From the the sun is 5.2 times farther than Earth’s. Jupiter  5.2  11.9
graph, the data appear to Saturn  9.54  29.5
(a) Using a graphing calculator, make a scatter
be linear. Uranus 19.2  84.0
diagram by plotting the point (ln D, ln P) for
(b) y = 1.5x each planet on the xy-coordinate axes. Do Neptune 30.1 164.8
6 the data points appear to be linear?
Source: Ronan, C., The Natural
(b) Determine a linear equation that models the History of the Universe, MacMillan
data points. Graph the line and the data on Publishing Co., New York.
−2 4 the same coordinate axes.
−2 (c) Use this linear model to predict the period of Pluto if its distance is 39.5.
The points 10, 02 and Compare the answer to the actual value of 248.5 yr.
13.40, 5.102 determine
the line y = 1.5x or Use the change-of-base theorem to find an approximation to four decimal places for
ln P = 1.5 ln D. each logarithm. See Example 8.
(Answers will vary.)
(c) P ≈ 248.3 yr   79. log2 5 80. log2 9 81. log8 0.59 82. log8 0.71

79. 2.3219   80. 3.1699   83. log1/2 3 84. log1/3 2 85. logp e 86. logp 22
81. - 0.2537  
82.  - 0.1647  87. log 213 12 88. log 219 5 89. log0.32 5 90. log0.91 8
83. - 1.5850  
84.  - 0.6309
85. 0.8736   86. 0.3028
87. 1.9376   88. 1.0932 Let u = ln a and v = ln b. Write each expression in terms of u and v without using the
89. - 1.4125  
90.  - 22.0488 ln function.

a # b4 B
a3 a3
  91. ln A b 4 2a B 94. ln A 2
3
1
91. 4v + 2 u  
92.  3u - 2v 92. ln 93. ln
b2 B b5
3 5 1
93. 2 u - 2 v  
94.  3 u + 4v
Concept Check  Use the various properties of exponential and logarithmic functions to
1
95. (a) 4 (b) 25 (c)  e evaluate the expressions in parts (a)  –  (c).
96. (a) 2 (b)  ln 3 (c)  ln 9   95. Given g1x2 = ex, find (a)  g1ln 42 (b)  g1ln 522
1
(c)  g A ln e B .
97. (a) 6 (b)  ln 3 (c)  ln 9
98. (a) 7 (b) 1 (c) 2  96. Given ƒ1x2 = 3x, find (a)  ƒ1log3 22 (b)  ƒ1log3 1ln 322 (c)  ƒ1log3 12 ln 322.

99. D 100. D   97. Given ƒ1x2 = ln x, find (a)  ƒ1e62 (b)  ƒ1eln 32 (c)  ƒ1e2 ln 32.
  98. Given ƒ1x2 = log2 x, find (a)  ƒ1272 (b)  ƒ12log2 22 (c)  ƒ122 log2 22.
101. domain: 1- ∞, 02 ´ 10, ∞2;
range: 1- ∞, ∞2; symmetric
with respect to the y-axis Work each problem.
102. (a)  1- ∞, 02 ´ 10, ∞2 99. Concept Check  Which of the following is equivalent to 2 ln 13x2 for x 7 0?
(b) f(x) = log3  x  A.  ln 9 + ln x B.  ln 6x C.  ln 6 + ln x D.  ln 9x 2
4
100. Concept Check  Which of the following is equivalent to ln 14x2 - ln 12x2 for x 7 0?
ln 4x
−4 4 A.  2 ln x B.  ln 2x C.  D.  ln 2
ln 2x

−4 101. The function ƒ1x2 = ln 0 x 0 plays a prominent role in calculus. Find its domain, its
(c) Because of limited reso- range, and the symmetries of its graph.
lution, the graph appears 102. Consider the function ƒ1x2 = log3 0 x 0 .
to show a point with
x-value of 0, which does (a) What is the domain of this function?
not exist on this graph. (b) Use a graphing calculator to graph ƒ1x2 = log3 0 x 0 in the window 3 - 4, 44 by
3 - 4, 44.
103. ƒ1x2 = 2 + ln x, so it is
(c) How might one easily misinterpret the domain of the function by merely
the graph of g1x2 = ln x
translated 2 units up. observing the calculator graph?

Copyright Pearson. All Rights Reserved.

M04_LHSD7659_12_AIE_C04_pp405-496.indd 460 06/10/15 6:47 PM


NOT FOR SALE
4.5  Exponential and Logarithmic Equations 461

104. ƒ1x2 = ln x - 1, so it is the Use properties of logarithms to rewrite each function, and describe how the graph of the
graph of g1x2 = ln x trans- given function compares to the graph of g1x2 = ln x.
lated 1 unit down.
x x
105. ƒ1x2 = ln x - 2, so it is the 103. ƒ1x2 = ln 1e2x2 104. ƒ1x2 = ln 105. ƒ1x2 = ln
graph of g1x2 = ln x trans- e e2
lated 2 units down.

Chapter 4 Quiz (Sections 4.1—4.4)


3
[4.1] 1. For the one-to-one function ƒ1x2 = 2 3x - 6, find ƒ -11x2.
x3 + 6
1. ƒ - 11x2 = 3 2. Solve 42x + 1 = 83x - 6.
[4.2] 3. Graph ƒ1x2 = - 3x. Give the domain and range.
2. 546
3. y domain: 4. Graph ƒ1x2 = log4 1x + 22. Give the domain and range.
2 1- ∞, ∞2;
0 x range:
5. Future Value  Suppose that $15,000 is deposited in a bank certificate of deposit at
–3 3
–3 1- ∞, 02 an annual rate of 2.7% for 8 yr. Find the future value if interest is compounded as
f(x) = –3
x follows.
–9 (a) annually    (b) quarterly    (c) monthly    (d)  daily (365 days)

[4.3] 6. Use a calculator to evaluate each logarithm to four decimal places.


4. y domain: (a)  log 34.56 (b)  ln 34.56
1- 2, ∞2;
4
range: 7. What is the meaning of the expression log6 25?
–2
02
x 1- ∞, ∞2
8. Solve each equation.
f(x) = log4 (x + 2)
(a)  x = 3log3 4 (b)  logx 25 = 2 (c)  log4 x = - 2
9. Assuming all variables represent positive real numbers, use properties of logarithms
[4.2] to rewrite
5. (a) $18,563.28 (b) $18,603.03
(c) $18,612.02 (d) $18,616.39 2x # y
log3 .
pq4
[4.4]
6. (a) 1.5386 (b) 3.5427 10. Given logb 9 = 3.1699 and logb 5 = 2.3219, find the value of logb 225.

[4.3] 11. Find the value of log3 40 to four decimal places.


7. The expression log6 25 repre-
12. If ƒ1x2 = 4x, what is the value of ƒ1log4 122?
sents the exponent to which 6
must be raised to obtain 25. [4.4]
8. (a)  546  (b)  556  (c)  E 16 F
1 1
9. 2 log3 x + log3 y - log3 p - 4 log3 q  10. 7.8137  11. 3.3578  12. 12

4.5 Exponential and Logarithmic Equations

■ Exponential Equations
Exponential Equations   We solved exponential equations in earlier sec-
■ Logarithmic Equations tions. General methods for solving these equations depend on the property below,
■ Applications and which follows from the fact that logarithmic functions are one-to-one.
Models

Property of Logarithms

If x 7 0, y 7 0, a 7 0, and a ≠ 1, then the following holds.

x = y  is equivalent to  log a x = log a y.

Copyright Pearson. All Rights Reserved.

M04_LHSD7659_12_AIE_C04_pp405-496.indd 461 06/10/15 6:47 PM


NOT FOR SALE
462 Chapter 4  Inverse, Exponential, and Logarithmic Functions

Classroom Example 1 Example 1 Solving an Exponential Equation


Solve 8x = 21. Give the solution to
the nearest thousandth. Solve 7x = 12. Give the solution to the nearest thousandth.
Answer:  51.4646 Solution  The properties of exponents cannot be used to solve this equation,
so we apply the preceding property of logarithms. While any appropriate base b
can be used, the best practical base is base 10 or base e. We choose base e (natural)
logarithms here.
15 7x = 12
ln 7x = ln 12 Property of logarithms
−2 5

x ln 7 = ln 12 Power property
This is exact.
ln 12
−20 x= Divide by ln 7.
ln 7
As seen in the display at the bottom
of the screen, when rounded to three
x ≈ 1.277 Use a calculator.
decimal places, the solution of This is approximate.
7x - 12 = 0 agrees with that found The solution set is 51.2776.
in Example 1. ■
✔ Now Try Exercise 11.

ln 12 12
Caution  Do not confuse a quotient like ln 7 in Example 1 with ln 7 ,
which can be written as ln 12 - ln 7. We cannot change the quotient of two
logarithms to a difference of logarithms.

ln 12 12
≠ ln
ln 7 7

Classroom Example 2 Example 2 Solving an Exponential Equation


Solve 52x - 3 = 8x + 1. Give the
solution to the nearest thousandth. Solve 32x - 1 = 0.4x + 2. Give the solution to the nearest thousandth.
Answer:  56.0626 Solution 32x - 1 = 0.4x + 2
ln 32x - 1 = ln 0.4x + 2 Take the natural logarithm on each side.

Teaching Tip  Caution students 12x - 12 ln 3 = 1x + 22 ln 0.4 Power property


against applying the quotient
2x ln 3 - ln 3 = x ln 0.4 + 2 ln 0.4 Distributive property
property of logarithms incorrectly
Write so that the terms with x are
in Example 2. 2x ln 3 - x ln 0.4 = 2 ln 0.4 + ln 3 on one side.
x12 ln 3 - ln 0.42 = 2 ln 0.4 + ln 3 Factor out x.

2 ln 0.4 + ln 3
x= Divide by 2 ln 3 - ln 0.4.
2 ln 3 - ln 0.4
ln 0.42 + ln 3
x= Power property
ln 32 - ln 0.4
ln 0.16 + ln 3
3 x= Apply the exponents.
ln 9 - ln 0.4
This is exact.
ln 0.48
−4 4
x= Product and quotient properties
ln 22.5
x ≈ - 0.236 Use a calculator.
−3
This is approximate.
This screen supports the solution The solution set is 5 - 0.2366.
found in Example 2. ■
✔ Now Try Exercise 19.
Copyright Pearson. All Rights Reserved.

M04_LHSD7659_12_AIE_C04_pp405-496.indd 462 06/10/15 6:47 PM


NOT FOR SALE
4.5  Exponential and Logarithmic Equations 463

Classroom Example 3 Example 3 Solving Base e Exponential Equations


Solve each equation. Give
solutions to the nearest Solve each equation. Give solutions to the nearest thousandth.
thousandth. (b)  e2x + 1 # e -4x = 3e
2
(a) e x = 200
(a) e 0 x 0 = 50
(b) e4x # e x - 1 = 5e Solution
2
Answers: (a) e x = 200
(a)  5 {3.9126 (b)  50.7226
2
ln e x = ln 200 Take the natural logarithm on each side.

x2 = ln 200 ln e x = x 2
2

Remember
both roots.
x = { 2ln 200 Square root property
x ≈ { 2.302 Use a calculator.

The solution set is 5{ 2.3026.

(b) e2x + 1 # e -4x = 3e


e -2x + 1 = 3e am # an = am + n
e -2x + 1
e -2x = 3 Divide by e; e1 = e -2x + 1 - 1 = e -2x.
ln e -2x = ln 3 Take the natural logarithm on each side.

- 2x ln e = ln 3 Power property

- 2x = ln 3 ln e = 1
1
x= - ln 3 Multiply by - 12 .
2
x ≈ - 0.549 Use a calculator.

The solution set is 5 - 0.5496.


 ■
✔ Now Try Exercises 21 and 23.

Classroom Example 4 Example 4 S


 olving an Exponential Equation (Quadratic in Form)
Solve e2x - 6e x + 5 = 0. Give
exact value(s) for x. Solve e2x - 4e x + 3 = 0. Give exact value(s) for x.
Answer:  50, ln 56 Solution  If we substitute u = e x, we notice that the equation is quadratic in
form.

e2x - 4e x + 3 = 0
1e x22 - 4e x + 3 = 0 am n = 1an2m

u 2 - 4u + 3 = 0 Let u = e x.
1u - 121u - 32 = 0 Factor.
u-1=0  or u - 3 = 0 Zero-factor property
u=1  or  u=3 Solve for u.

ex = 1  or  ex = 3 Substitute e x for u.


ln e x = ln 1 ln e x = ln 3 T
or   ake the natural logarithm
on each side.
x=0  or  x = ln 3 ln e x = x; ln 1 = 0

Both values check, so the solution set is 50, ln 36.



✔ Now Try Exercise 35.
Copyright Pearson. All Rights Reserved.

M04_LHSD7659_12_AIE_C04_pp405-496.indd 463 06/10/15 6:47 PM


NOT FOR SALE
464 Chapter 4  Inverse, Exponential, and Logarithmic Functions

Logarithmic Equations   The following equations involve logarithms of


variable expressions.

Classroom Example 5 Example 5 Solving Logarithmic Equations


Solve each equation. Give exact
values. Solve each equation. Give exact values.
(a) 4 ln x = 36 (a) 7 ln x = 28 (b)  log2 1x 3 - 192 = 3
(b) log3 1x 3 - 52 = 1
Solution
Answers:
(a) 5e9 6 (b)  526 (a) 7 ln x = 28
loge x = 4 ln x = loge x; Divide by 7.
x = e4 Write in exponential form.

The solution set is 5e4 6.

(b) log2 1x 3 - 192 = 3


x 3 - 19 = 23 Write in exponential form.

x 3 - 19 = 8 Apply the exponent.

x3 = 27 Add 19.
3
x = 227 Take cube roots.
x=3 3
2 27 = 3

The solution set is 536.


 ■
✔ Now Try Exercises 41 and 49.

Classroom Example 6 Example 6 Solving a Logarithmic Equation


Solve
Solve log 1x + 62 - log 1x + 22 = log x. Give exact value(s).
log 12x + 12 + log x = log 1x + 82.
Solution  Recall that logarithms are defined only for nonnegative numbers.
Give exact value(s).
Answer:  526 log 1x + 62 - log 1x + 22 = log x
x+6
log = log x Quotient property
x+2
Teaching Tip  Tell students that
a logarithmic equation cannot be x+6
solved with two logarithms on the =x Property of logarithms
x+2
same side of the equation. The
equation must be transformed x + 6 = x1x + 22 Multiply by x + 2.
so that each side contains an
expression with at most one x + 6 = x 2 + 2x Distributive property
logarithm. x2 + x - 6 = 0 Standard form

1x + 321x - 22 = 0 Factor.

x + 3 = 0  or x - 2 = 0      Zero-factor property

x = - 3 or  x = 2 Solve for x.

The proposed negative solution 1 - 32 is not in the domain of log x in the origi-
nal equation, so the only valid solution is the positive number 2. The solution set
is 526.

✔ Now Try Exercise 69.
Copyright Pearson. All Rights Reserved.

M04_LHSD7659_12_AIE_C04_pp405-496.indd 464 06/10/15 6:47 PM


NOT FOR SALE
4.5  Exponential and Logarithmic Equations 465

Caution  Recall that the domain of y = loga x is 10, ∞2. For this reason,
it is always necessary to check that proposed solutions of a logarithmic
equation result in logarithms of positive numbers in the original equation.

Classroom Example 7 Example 7 Solving a Logarithmic Equation


Solve log3 314x + 121x + 124 = 3.
Give exact value(s). Solve log2 313x - 721x - 424 = 3. Give exact value(s).
Answer:  E -
13
4 , 2F Solution log2 313x - 721x - 424 = 3
13x - 721x - 42 = 23 Write in exponential form.
3x 2 - 19x + 28 = 8 Multiply. Apply the exponent.
3x 2 - 19x + 20 = 0 Standard form
13x - 421x - 52 = 0 Factor.
3x - 4 = 0 or x - 5 = 0 Zero-factor property

4
x =  or  x = 5 Solve for x.
3
A check is necessary to be sure that the argument of the logarithm in the given
equation is positive. In both cases, the product 13x - 721x - 42 leads to 8, and
log2 8 = 3 is true. The solution set is E 3 , 5 F.
4


✔ Now Try Exercise 53.

Classroom Example 8 Example 8 Solving a Logarithmic Equation


Solve
Solve log 13x + 22 + log 1x - 12 = 1. Give exact value(s).
log2 12x - 52 + log2 1x - 32 = 3.
Solution log 13x + 22 + log 1x - 12 = 1
Give exact value(s).
11 + 265
log10 313x + 221x - 124 = 1 log x = log10 x; product property
Answer:  U 4 V
13x + 221x - 12 = 10 1 Write in exponential form.
3x 2 - x - 2 = 10 Multiply; 10 1 = 10.
3x 2 - x - 12 = 0 Subtract 10.
- b { 2b 2 - 4ac
x=
2a
Quadratic formula

- 1 - 12 { 21 - 122 - 41321 - 122


x=
2132
Substitute a = 3, b = - 1, c = - 12.
The two proposed solutions are
1 - 2145 1 + 2145
 and  .
6 6
1 - 2145
The first proposed solution, 6 , is negative. Substituting for x in
log 1x - 12 results in a negative argument, which is not allowed. Therefore, this
solution must be rejected.
1 + 2145
The second proposed solution, 6 , is positive. Substituting it for x in
log 13x + 22 results in a positive argument. Substituting it for x in log 1x + 12
also results in a positive argument. Both are necessary conditions. Therefore, the
solution set is E F.
1 + 2145
6 ■
✔ Now Try Exercise 77.
Copyright Pearson. All Rights Reserved.

M04_LHSD7659_12_AIE_C04_pp405-496.indd 465 06/10/15 6:47 PM


NOT FOR SALE
466 Chapter 4  Inverse, Exponential, and Logarithmic Functions

Note  We could have replaced 1 with log10 10 in Example 8 by first


writing

log 13x + 22 + log 1x - 12 = 1    Equation from Example 8

log10 313x + 221x - 124 = log10 10   Substitute.


13x + 221x - 12 = 10,    Property of logarithms

and then continuing as shown on the preceding page.

Classroom Example 9 Example 9 Solving a Base e Logarithmic Equation


Solve ln eln x - ln 1x - 42 = ln 5.
Give exact value(s). Solve ln eln x - ln 1x - 32 = ln 2. Give exact value(s).
Answer:  556 Solution  This logarithmic equation differs from those in Examples 7 and 8
because the expression on the right side involves a logarithm.

ln eln x - ln 1x - 32 = ln 2
ln x - ln 1x - 32 = ln 2    eln x = x
x
ln = ln 2    Quotient property
x-3
x
=2    Property of logarithms
x-3
x = 21x - 32   Multiply by x - 3.
x = 2x - 6    Distributive property
x=6    Solve for x.

Check that the solution set is 566.



✔ Now Try Exercise 79.

Solving an Exponential or Logarithmic Equation

To solve an exponential or logarithmic equation, change the given equation


into one of the following forms, where a and b are real numbers, a 7 0 and
a ≠ 1, and follow the guidelines.
1. aƒ 1x2 = b
Solve by taking logarithms on each side.
2. log a ƒ 1 x 2 = b
Solve by changing to exponential form a b = ƒ1x2.
3. log a ƒ 1 x 2 = log a g 1 x 2
The given equation is equivalent to the equation ƒ1x2 = g1x2. Solve
algebraically.
4. In a more complicated equation, such as
e2x + 1 # e -4x = 3e,   See Example 3(b).
it may be necessary to first solve for a ƒ1x2 or loga ƒ1x2 and then solve the
resulting equation using one of the methods given above.
5. Check that each proposed solution is in the domain.

Copyright Pearson. All Rights Reserved.

M04_LHSD7659_12_AIE_C04_pp405-496.indd 466 06/10/15 6:47 PM


NOT FOR SALE
4.5  Exponential and Logarithmic Equations 467

Applications and Models

Classroom Example 10 Example 10 Applying an Exponential Equation to the Strength


The equation T = T0 + Ce -kt can of a Habit
be used to describe Newton’s law
of cooling. Solve this equation The strength of a habit is a function of the number of times the habit is repeated.
for k. If N is the number of repetitions and H is the strength of the habit, then, accord-
1 T - T0 ing to psychologist C.L. Hull,
Answer:  k = - t ln A C B
H = 100011 - e -kN2,

where k is a constant. Solve this equation for k.


Solution H = 100011 - e -kN2 First solve for
e -kN.
H
= 1 - e -kN Divide by 1000.
1000
H
- 1 = - e -kN Subtract 1.
1000
Classroom Example 11 H
e -kN = 1 - Multiply by - 1 and rewrite.
Refer to the model in Example 11. 1000
(a) Use the function ƒ1t2 to Now solve
estimate the number of tablets for k. H Take the natural logarithm
ln e -kN = ln a 1 - b 
on each side.
sold in 2011. How does this 1000
figure compare to the actual
figure of 24.1 million? H
- kN = ln a 1 - b ln ex = x
(b) If this trend continues, 1000
approximately when will
annual sales reach 58 million 1 H
tablets? k= - ln a 1 - b Multiply by - N1 .
N 1000
Answers:
With the final equation, if one pair of values for H and N is known, k can be found,
(a) 24.8 million; This is fairly
close to the actual figure. and the equation can then be used to find either H or N for given values of the
(b) Annual sales will reach other variable.

✔ Now Try Exercise 91.
58 million in 2019.

Example 11 Modeling PC Tablet Sales in the U.S.

Sales The table gives U.S. tablet sales (in millions) for several years. The data can be
Year (in millions) modeled by the function
2010 10.3
ƒ1t2 = 20.57 ln t + 10.58, t Ú 1,
2011 24.1
2012 35.1 where t is the number of years after 2009.
2013 39.8 (a) Use the function to estimate the number of tablets sold in the United States
2014 42.1 in 2015.
Source: Forrester Research. (b) If this trend continues, approximately when will annual sales reach 60 million?
Solution

(a) The year 2015 is represented by t = 2015 - 2009 = 6.

ƒ1t2 = 20.57 ln t + 10.58 Given function

ƒ162 = 20.57 ln 6 + 10.58 Let t = 6.


ƒ162 ≈ 47.4 Use a calculator.

Based on this model, 47.4 million tablets were sold in 2015.


Copyright Pearson. All Rights Reserved.

M04_LHSD7659_12_AIE_C04_pp405-496.indd 467 06/10/15 6:47 PM


NOT FOR SALE
468 Chapter 4  Inverse, Exponential, and Logarithmic Functions

(b) Replace ƒ1t2 with 60 and solve for t.

ƒ1t2 = 20.57 ln t + 10.58 Given function


60 = 20.57 ln t + 10.58 Let ƒ1t2 = 60.
49.42 = 20.57 ln t Subtract 10.58.

49.42
ln t = Divide by 20.57 and rewrite.
20.57
t = e49.42/20.57 Write in exponential form.

t ≈ 11.05 Use a calculator.

Adding 11 to 2009 gives the year 2020. Based on this model, annual sales
will reach 60 million in 2020.

✔ Now Try Exercise 111.

4.5 Exercises

1. B  
2. F Concept Preview  Match each equation in Column I with the best first step for
3. E  
4. A solving it in Column II.
5. D  
6. C
I II
log 19 ln 19
7. log7 19; log 7 ; ln 7
1. 10 x = 150 A. Use the product rule for exponents.
log 10 1 ln 10 2. e2x - 1 = 24 B. Take the common logarithm on each side.
8. log3 10; log 3 , or log 3 ; ln 3
C. Write the sum of logarithms as the
log 12 ln 12 3. log4 1x 2 - 102 = 2
9. log1/2 12; ; logarithm of a product.
4. e2x # ex = 2e
1 1
log 2 ln 2
D. Let u = ex and write the equation in
log 4 ln 4 quadratic form.
10. log1/3 4; ; 1 5. 2e2x - 5ex -3=0
1
log 3 ln 3 E. Change to exponential form.
6. log 12x - 12 + log 1x + 42 = 1
F. Take the natural logarithm on each side.
11. 51.7716 12.  51.5946
13. 5 - 2.3226 14.  5 - 1.6316 Concept Preview  An exponential equation such as
15. 5 - 6.2136 16.  5 - 2.1516 5x = 9
17. 5 - 1.7106 18.  50.8236 can be solved for its exact solution using the meaning of logarithm and the change-of-
19. 53.2406 base theorem. Because x is the exponent to which 5 must be raised in order to obtain 9,
the exact solution is
log 9 ln 9
log5 9, or  , or  .
log 5 ln 5
For each equation, give the exact solution in three forms similar to the forms above.
1 x 1 x
7. 7x = 19 8. 3x = 10 9. a b = 12 10. a b = 4
2 3

Solve each equation. In Exercises 11–34, give irrational solutions as decimals correct
to the nearest thousandth. In Exercises 35– 40, give solutions in exact form. See
Examples 1–4.
1 x
11. 3x = 7 12. 5x = 13 13. a b = 5
2
1 x
14. a b = 6 15. 0.8x = 4 16. 0.6 x = 3
3
17. 4x - 1 = 32x 18. 2x + 3 = 52x 19. 6 x + 1 = 42x - 1

Copyright Pearson. All Rights Reserved.

M04_LHSD7659_12_AIE_C04_pp405-496.indd 468 06/10/15 6:47 PM


NOT FOR SALE
4.5  Exponential and Logarithmic Equations 469

20. 5 - 5.0576 20. 3x - 4 = 72x + 5


2
21. ex = 100
4
22. ex = 1000
21. 5{2.1466 22.  5{1.6216
23. e3x - 7 # e -2x = 4e 24. e1 - 3x # e5x = 2e
1 x
23. 59.3866 24.  50.3476 25. a b = - 3
3
25.  ∅ 26.  ∅
1 x
27. 532.9506 28.  56.5796 26. a b = - 9 27. 0.0511.152x = 5 28. 1.210.92x = 0.6
9
29. 57.0446 30.  516
31. 525.6776 32.  59.0956 29. 3122x - 2 + 1 = 100 30. 511.223x - 2 + 1 = 7 31. 211.052x + 3 = 10
33. 52011.5686 34.  51917.0966 32. 311.42x - 4 = 60 33. 511.0152x - 1980 = 8 34. 611.0242x - 1900 = 9
35. 5ln 2, ln 46 36.  5ln 3, ln 56
35. e2x - 6ex + 8 = 0 36. e2x - 8ex + 15 = 0 37. 2e2x + ex = 6
37.  E ln F 38.  E ln F
3 1
2 3 38. 3e2x + 2ex = 1 39. 52x + 315x2 = 28 40. 32x - 1213x2 = - 35
39. 5log5 46
40. 5log3 5, log3 76
Solve each equation. Give solutions in exact form. See Examples 5–9.
41. 5e2 6 42.  5e3 6
41. 5 ln x = 10 42. 3 ln x = 9
43.  E F 44.  E 2 F
e1.5 e5
4
43. ln 4x = 1.5 44. ln 2x = 5
45.  E 2 - 210 F
45. log 12 - x2 = 0.5 46. log 13 - x2 = 0.75
46.  E 3 - 2 1000 F
4
47. log6 12x + 42 = 2 48. log5 18 - 3x2 = 3
47. 5166 48.  5 - 396
49. log4 1x 3 + 372 = 3 50. log7 1x 3 + 652 = 0
49. 536 50.  5 - 46
51. 5e6 52.  5106 51. ln x + ln x 2 = 3 52. log x + log x 2 = 3

54.  E - 8F
14 53. log3 31x + 521x - 324 = 2 54. log4 313x + 821x - 624 = 3
53. 5 - 6, 46 3 ,

55. 5 - 8, 06 56.  E - 3 , 0 F
8 55. log2 312x + 821x + 424 = 5 56. log5 313x + 521x + 124 = 1

57. 556 58.  526 57. log x + log 1x + 152 = 2 58. log x + log 12x + 12 = 1
59. 5 - 56 60.  5 - 16 59. log 1x + 252 = log 1x + 102 + log 4 60. log 13x + 52 - log 12x + 42 = 0
61.  ∅ 62.  ∅
61. log 1x - 102 - log 1x - 62 = log 2 62. log 1x 2 - 92 - log 1x - 32 = log 5
63. 5 - 26 64.  5 - 56
63. ln 17 - x2 + ln 11 - x2 = ln 125 - x2 64. ln 13 - x2 + ln 15 - x2 = ln 150 - 6x2
66.  E 2 F
9
65. 506
65. log8 1x + 22 + log8 1x + 42 = log8 8 66. log2 15x - 62 - log2 1x + 12 = log2 3
67. 5126 68.  536
69. 5256 70.  556 67. log2 1x 2 - 1002 - log2 1x + 102 = 1 68. log2 1x - 22 + log2 1x - 12 = 1
71.  ∅ 72.  ∅ 69. log x + log 1x - 212 = log 100 70. log x + log 13x - 132 = log 10
73. E F
5
2 74.  546 71. log 19x + 52 = 3 + log 1x + 22 72. log 111x + 92 = 3 + log 1x + 32
75. 536 76.  586
73. ln 14x - 22 - ln 4 = - ln 1x - 22 74. ln 15 + 4x2 - ln 13 + x2 = ln 3
1 + 241 1 + 285
77.  U 4 V 78.  U 6 V 75. log5 1x + 22 + log5 1x - 22 = 1 76. log2 1x - 72 + log2 x = 3
79. 566 80.  566
77. log2 12x - 32 + log2 1x + 12 = 1 78. log5 13x + 22 + log5 1x - 12 = 1
81. 546 82.  51, 106
79. ln ex - 2 ln e = ln e4 80. ln ex - ln e3 = ln e3
83. 51, 1006 84.  5 - 2, 26
81. log2 1log2 x2 = 1 82. log x = 2log x
85. Proposed solutions that cause
any argument of a logarithm 3
to be negative or zero must be 83. log x 2 = 1log x22 84. log2 22x 2 =
2
rejected. The statement is not
correct. For example, the 85. Concept Check  Consider the following statement: “We must reject any negative
solution set of proposed solution when we solve an equation involving logarithms.” Is this correct?
Why or why not?
log 1- x + 992 = 2
is 5 - 16. 86. Concept Check  What values of x could not possibly be solutions of the following
equation?
86. any real numbers less than or
7
equal to 4 loga 14x - 72 + loga 1x 2 + 42 = 0

Copyright Pearson. All Rights Reserved.

M04_LHSD7659_12_AIE_C04_pp405-496.indd 469 06/10/15 6:47 PM


NOT FOR SALE
470 Chapter 4  Inverse, Exponential, and Logarithmic Functions

87. x = ek/1p - a2 Solve each equation for the indicated variable. Use logarithms with the appropriate
88. t = e1 p - r2/k bases. See Example 10.
1 T - T0
89. t = - k log A T1 - T0 B k
87. p = a + ,  for x 88. r = p - k ln t, for t
A - Pr ln x
log A A B
90. n = - Pr
log 11 + r2 89. T = T0 + 1T1 - T0210 -kt, for t 90. A = , for n
2 RI 1 - 11 + r2-n
91. t = - R ln A 1 - E B
E K
ln A
K - y
B 91. I = 11 - e -Rt/22, for t 92. y = , for b
ay R 1 + ae -bx
92. b =
-x
M
ln A
A + B - y 93. y = A + B11 - e -Cx2, for x 94. m = 6 - 2.5 log , for M
B B M0
93. x =
-C
I
94. M = M0 # 10 16 - m2/2.5
95. log A = log B - C log x, for A 96. d = 10 log , for I
I0
B
95. A = xC r tn
96. I = I0 # 10 d/10
97. A = P a1 + b , for t 98. D = 160 + 10 log x, for x
n
A
log P
97. t = To solve each problem, refer to the formulas for compound interest.
r
n log A 1 + n B
r tn
A = P a1 + b and A = Pert
98. x = 10 D/10 - 16 n

99. $11,611.84 100.  $6885.64 99. Compound Amount  If $10,000 is invested in an account at 3% annual interest
101. 2.6 yr 102.  5.55 yr compounded quarterly, how much will be in the account in 5 yr if no money is
103. 2.64% 104. 3.09% withdrawn?

105. (a)  10.9% (b) 35.8% 100. Compound Amount  If $5000 is invested in an account at 4% annual interest com-
(c) 84.1% pounded continuously, how much will be in the account in 8 yr if no money is
106. (a) $3891 (b) $6990 withdrawn?
(c) $8495 101. Investment Time  Kurt wants to buy a $30,000 truck. He has saved $27,000. Find
the number of years (to the nearest tenth) it will take for his $27,000 to grow to
$30,000 at 4% interest compounded quarterly.
102. Investment Time Find t to the nearest hundredth of a year if $1786 becomes
$2063 at 2.6%, with interest compounded monthly.
103. Interest Rate  Find the interest rate to the nearest hundredth of a percent that will
produce $2500, if $2000 is left at interest compounded semiannually for 8.5 yr.
104. Interest Rate  At what interest rate, to the nearest hundredth of a percent, will
$16,000 grow to $20,000 if invested for 7.25 yr and interest is compounded
quarterly?

(Modeling)  Solve each application. See Example 11.


105. In the central Sierra Nevada (a mountain range in California), the percent of mois-
ture that falls as snow rather than rain is approximated reasonably well by
ƒ1x2 = 86.3 ln x - 680,
where x is the altitude in feet and ƒ1x2 is the percent of moisture that falls as snow.
Find the percent of moisture, to the nearest tenth, that falls as snow at each altitude.
(a)  3000 ft (b)  4000 ft (c)  7000 ft
106. Northwest Creations finds that its total sales in dollars, T1x2, from the distribution
of x thousand catalogues is approximated by
T1x2 = 5000 log 1x + 12.
Find the total sales, to the nearest dollar, resulting from the distribution of each
number of catalogues.
(a) 5000 (b) 24,000 (c) 49,000

Copyright Pearson. All Rights Reserved.

M04_LHSD7659_12_AIE_C04_pp405-496.indd 470 06/10/15 6:47 PM


NOT FOR SALE
4.5  Exponential and Logarithmic Equations 471

107. 2019 107. Average Annual Public University Costs The Year Average Annual Cost
108. (a)  11.65 m per sec table shows the cost of a year’s tuition, room
and board, and fees at 4-year public colleges 2006 $12,837
(b)  2.48 sec
109. (a)  62% (b)  1989 for the years 2006–2014. Letting y represent 2007 $13,558
110. (a) The graph of ƒ1- x2 is the cost in dollars and x the number of years 2008 $14,372
symmetric to the graph since 2006, the function 2009 $15,235
of ƒ1x2 with respect to ƒ1x2 = 13,01711.052x 2010 $16,178
the y-axis.
(b)  984 ft (c)  39 ft models the data quite well. According to this 2011 $17,156
function, in what year will the 2006 cost be 2012 $17,817
doubled? 2013 $18,383
2014 $18,943
Source: The College Board, Annual
Survey of Colleges.

108. Race Speed  At the World Championship races held at Rome’s Olympic Stadium
in 1987, American sprinter Carl Lewis ran the 100-m race in 9.86 sec. His speed in
meters per second after t seconds is closely modeled by the function

ƒ1t2 = 11.6511 - e -t/1.272.

(Source: Banks, Robert B., Towing Icebergs, Falling Dominoes, and Other Adven-
tures in Applied Mathematics, Princeton University Press.)
(a)  How fast, to the nearest hundredth, was he running as he crossed the finish line?
(b) After how many seconds, to the nearest hundredth, was he running at the rate
of 10 m per sec?
109. Women in Labor Force  The percent of women in the U.S. civilian labor force can
be modeled fairly well by the function

67.21
ƒ1x2 = ,
1 + 1.081e -x/24.71

where x represents the number of years since 1950. (Source: Monthly Labor
Review, U.S. Bureau of Labor Statistics.)
(a) What percent, to the nearest whole number, of U.S. women were in the civilian
labor force in 2014?
(b) In what year were 55% of U.S. women in the civilian labor force?
110. Height of the Eiffel Tower  One side of y
the Eiffel Tower in Paris has a shape that
can be approximated by the graph of the
function
x
f(x) = –301 ln ,x>0
x 207
ƒ1x2 = - 301 ln , x 7 0,
207

where x and ƒ1x2 are both measured in


100
feet. (Source: Banks, Robert B., Towing x
Icebergs, Falling Dominoes, and Other 100
Adventures in Applied Mathematics,
Princeton University Press.)
(a) Why does the shape of the left side of the graph of the Eiffel Tower have the
formula given by ƒ1 - x2?
(b) The short horizontal segment at the top of the figure has length 7.8744 ft. How
tall, to the nearest foot, is the Eiffel Tower?
(c) How far from the center of the tower is the point on the right side that is 500 ft
above the ground? Round to the nearest foot.

Copyright Pearson. All Rights Reserved.

M04_LHSD7659_12_AIE_C04_pp405-496.indd 471 06/10/15 6:47 PM


NOT FOR SALE
472 Chapter 4  Inverse, Exponential, and Logarithmic Functions

111. (a)  P1x2 = 1 - e -0.0034 - 0.0053x 111. CO2 Emissions Tax  One action that government could take to reduce carbon
(b) P(x) = 1 − e−0.0034 − 0.0053x emissions into the atmosphere is to levy a tax on fossil fuel. This tax would be
1 based on the amount of carbon dioxide emitted into the air when the fuel is burned.
The cost-benefit equation
ln 11 - P2 = - 0.0034 - 0.0053x

0 1000 models the approximate relationship between a tax of x dollars per ton of carbon
0 and the corresponding percent reduction P (in decimal form) of emissions of car-
(c)  P1602 ≈ 0.275, or 27.5%; bon dioxide. (Source: Nordhause, W., “To Slow or Not to Slow: The Economics of
The reduction in carbon the Greenhouse Effect,” Yale University, New Haven, Connecticut.)
emissions from a tax of (a) Write P as a function of x.
$60 per ton of carbon is (b) Graph P for 0 … x … 1000. Discuss the benefit of continuing to raise taxes on
27.5%.
carbon.
(d) x = $130.14
112. (a)  R ≈ 4.4 w/m2 (c) Determine P, to the nearest tenth, when x = $60. Interpret this result.
(b)  T ≈ 4.5°F; (d) What value of x will give a 50% reduction in carbon emissions?

113. ƒ - 11x2 = ln x + 5; 112. Radiative Forcing  Radiative forcing, R, measures the influence of carbon dioxide
domain: 10, ∞2;
in altering the additional solar radiation trapped in Earth’s atmosphere. The Inter-
national Panel on Climate Change (IPCC) in 1990 estimated k to be 6.3 in the
range: 1- ∞, ∞2
radiative forcing equation
114. ƒ - 11x2 = ln 1x - 102;
domain: 110, ∞2; C
R = k ln ,
range: 1- ∞, ∞2 C0
115. ƒ - 11x2 = ln 1x + 42 - 1; where C0 is the preindustrial amount of carbon dioxide and C is the current level.
domain: 1- 4, ∞2; (Source: Clime, W., The Economics of Global Warming, Institute for International
range: 1- ∞, ∞2 Economics, Washington, D.C.)
116. ƒ - 11x2 = e x - 2; C
(a) Use the equation R = 6.3 ln C0 to determine the radiative forcing R (in watts
domain: 1- ∞, ∞2; per square meter to the nearest tenth) expected by the IPCC if the carbon diox-
range: 1- 2, ∞2 ide level in the atmosphere doubles from its preindustrial level.
1
117. ƒ - 11x2 = 3 ex/2; (b) Determine the global temperature increase T, to the nearest tenth, that the IPCC
domain: 1- ∞, ∞2; predicted would occur if atmospheric carbon dioxide levels were to double,
range: 10, ∞2 given T1R2 = 1.03R.
118. ƒ - 11x2 = e x - 6 + 1;
domain: 1- ∞, ∞2; Find ƒ -11x2, and give the domain and range.
range: 11, ∞2
113. ƒ1x2 = ex - 5 114. ƒ1x2 = ex + 10 115. ƒ1x2 = ex + 1 - 4
119. 51.526 116. ƒ1x2 = ln 1x + 22 117. ƒ1x2 = 2 ln 3x 118. ƒ1x2 = ln 1x - 12 + 6
120. 5 - 0.93, 1.356
121. 506 Use a graphing calculator to solve each equation. Give irrational solutions correct to
122. 50.69, 1.106 the nearest hundredth.
123. 52.45, 5.666 119. ex + ln x = 5 120. ex - ln 1x + 12 = 3 121. 2ex + 1 = 3e -x
124. 50.236 3
122. ex + 6e -x = 5 123. log x = x 2 - 8x + 14 124. ln x = - 2 x+3
125. When dividing each side by 125. Find the error in the following “proof ” that 2 6 1.
1
log 3 , the direction of the
inequality symbol should be 1 1
1
6 True statement
reversed because log 3 is 9 3
negative.
1 2 1
a b 6 Rewrite the left side.
3 3
1 2 1
log a b 6 log Take the logarithm on each side.
3 3
1 1
2 log 6 1 log Property of logarithms; identity property
3 3
1
261 Divide each side by log 3 .
Copyright Pearson. All Rights Reserved.

M04_LHSD7659_12_AIE_C04_pp405-496.indd 472 20/10/15 11:03 AM


NOT FOR SALE
4.6  Applications and Models of Exponential Growth and Decay 473

4.6 Applications and Models of Exponential Growth


and Decay
■ The Exponential
The Exponential Growth or Decay Function   In many situations in
Growth or Decay
Function ecology, biology, economics, and the social sciences, a quantity changes at a
■ Growth Function
rate proportional to the amount present. The amount present at time t is a special
Models function of t called an exponential growth or decay function.
■ Decay Function
Models
Exponential Growth or Decay Function

Let y0 be the amount or number present at time t = 0. Then, under certain


conditions, the amount y present at any time t is modeled by
Looking Ahead To Calculus
The exponential growth and decay
y = y0 e kt,  where k is a constant.
function formulas are studied in
calculus in conjunction with the
topic known as differential equations.
The constant k determines the type of function.
• When k 7 0, the function describes growth. Examples of exponential growth
include compound interest and atmospheric carbon dioxide.
• When k 6 0, the function describes decay. One example of exponential decay
is radioactive decay.

Growth Function Models   The amount of time it takes for a quantity that
grows exponentially to become twice its initial amount is its doubling time.

Example 1 Determining a Function to Model Exponential Growth

Carbon Dioxide Earlier in this chapter, we discussed the growth of atmospheric carbon dioxide
Year (ppm) over time using a function based on the data from the table. Now we determine
1990  353 such a function from the data.
2000  375 (a) Find an exponential function that gives the amount of carbon dioxide y in
2075  590 year x.
2175 1090 (b) Estimate the year when future levels of carbon dioxide will be double the
2275 2000 preindustrial level of 280 ppm.
Source: International Panel on Solution
Climate Change (IPCC).
(a) The data points exhibit exponential growth, so the equation will take the form

y = y0 ekx.

We must find the values of y0 and k. The data begin with the year 1990, so
to simplify our work we let 1990 correspond to x = 0, 1991 correspond
to x = 1, and so on. Here y0 is the initial amount and y0 = 353 in 1990 when
x = 0. Thus the equation is

y = 353ekx.   Let y0 = 353.

From the last pair of values in the table, we know that in 2275 the carbon
dioxide level is expected to be 2000 ppm. The year 2275 corresponds to
2275 - 1990 = 285. Substitute 2000 for y and 285 for x, and solve for k.
Copyright Pearson. All Rights Reserved.

M04_LHSD7659_12_AIE_C04_pp405-496.indd 473 06/10/15 6:47 PM


NOT FOR SALE
474 Chapter 4  Inverse, Exponential, and Logarithmic Functions

Classroom Example 1 y = 353ekx Solve for k.


Refer to the table given with
Example 1. 2000 = 353ek12852 Substitute 2000 for y and 285 for x.
(a) Find a different exponential
2000
model using the data for 2000 = e285k Divide by 353.
and 2175. Let the year 2000 353
correspond to x = 0. 2000
(b) Use the model from part (a) ln = ln e285k Take the natural logarithm on each side.
to estimate when future levels
353
of carbon dioxide will triple 2000
from the preindustrial level of ln = 285k ln ex = x, for all x.
353
280 ppm.
Answers: k=
1 # ln 2000 1
Multiply by 285 and rewrite.
(a) y = 375e0.00610x 285 353
(b) It will triple from the k ≈ 0.00609 Use a calculator.
preindustrial level during
2132. A function that models the data is

y = 353e0.00609x.

(b) y = 353e0.00609x Solve the model from part (a) for the year x.
560 = 353e0.00609x To double the level 280, let y = 212802 = 560.
560
= e0.00609x Divide by 353.
353
560
ln = ln e0.00609x Take the natural logarithm on each side.
353
Teaching Tip In Example 1(b), 560
stress that 75.8 does not represent ln = 0.00609x ln ex = x, for all x.
353
the time for carbon dioxide to
double. It represents the time since
1990 that the preindustrial level of x=
1 # ln 560 1
Multiply by 0.00609 and rewrite.
0.00609 353
280 ppm took to double. Mention
that the solution to x ≈ 75.8 Use a calculator.
2= e0.00609x
Since x = 0 corresponds to 1990, the preindustrial carbon dioxide level will
yields a doubling time of 114 yr. double in the 75th year after 1990, or during 2065, according to this model.

✔ Now Try Exercise 43.

Classroom Example 2 Example 2 Finding Doubling Time for Money


How long will it take for money
in an account that accrues interest How long will it take for money in an account that accrues interest at a rate of
at a rate of 2.25%, compounded 3%, compounded continuously, to double?
continuously, to double?
Solution A = Per t Continuous compounding formula
Answer:  about 31 yr
2P = Pe0.03t Let A = 2P and r = 0.03.

2 = e0.03t Divide by P.
Teaching Tip  Have students
solve Example 2 using interest ln 2 = ln e0.03t Take the natural logarithm on each side.
rates of 2%, 4%, and 10%.
Explain that the Rule of 72, ln 2 = 0.03t ln ex = x
72 ln 2
t= , =t Divide by 0.03.
r 0.03
is often used to approximate 23.10 ≈ t Use a calculator.
doubling times. Have students check
their answers against this rule. It will take about 23 yr for the amount to double. ■
✔ Now Try Exercise 31.
Copyright Pearson. All Rights Reserved.

M04_LHSD7659_12_AIE_C04_pp405-496.indd 474 06/10/15 6:47 PM


NOT FOR SALE
4.6  Applications and Models of Exponential Growth and Decay 475

Example 3 Using an Exponential Function to Model


Population Growth

According to the U.S. Census Bureau, the world population reached 6 billion
people during 1999 and was growing exponentially. By the end of 2010, the pop-
ulation had grown to 6.947 billion. The projected world population (in billions
of people) t years after 2010 is given by the function

ƒ1t2 = 6.947e0.00745t.

(a) Based on this model, what will the world population be in 2025?
(b) If this trend continues, approximately when will the world population reach
Classroom Example 3 9 billion?
Refer to the model in Example 3. Solution
(a) Based on the model, what will
the world population be in (a) Since t = 0 represents the year 2010, in 2025, t would be 2025 - 2010 =
2020? 15 yr. We must find ƒ1t2 when t is 15.
(b) If this trend continues,
approximately when will ƒ1t2 = 6.947e0.00745t    Given function
the world population reach ƒ1152 = 6.947e0.007451152   Let t = 15.
8 billion?
ƒ1152 ≈ 7.768    Use a calculator.
Answers:
(a) 7.484 billion The population will be 7.768 billion at the end of 2025.
(b) It will reach 8 billion during
the year 2028. (b) ƒ1t2 = 6.947e0.00745t Given function
9 = 6.947e0.00745t Let ƒ1t2 = 9.
9
= e0.00745t Divide by 6.947.
6.947
9
ln = ln e0.00745t Take the natural logarithm on each side.
6.947
9
ln = 0.00745t ln ex = x, for all x.
6.947
9
ln 6.947
t= Divide by 0.00745 and rewrite.
0.00745
t ≈ 34.8 Use a calculator.

Thus, 34.8 yr after 2010, during the year 2044, world population will reach
9 billion.
■✔ Now Try Exercise 39.

Decay Function Models  Half-life is the amount of time it takes for a


quantity that decays exponentially to become half its initial amount.

Note In Example 4 on the next page, the initial amount of substance is


given as 600 g. Because half-life is constant over the lifetime of a decaying
1
quantity, starting with any initial amount, y0 , and substituting 2 y0 for y in
y = y0 ek t would allow the common factor y0 to be divided out. The rest of
the work would be the same.

Copyright Pearson. All Rights Reserved.

M04_LHSD7659_12_AIE_C04_pp405-496.indd 475 06/10/15 6:47 PM


NOT FOR SALE
476 Chapter 4  Inverse, Exponential, and Logarithmic Functions

Classroom Example 4 Example 4 D


 etermining an Exponential Function to Model
Suppose 800 g of a radioactive Radioactive Decay
substance are present initially and
2.5 yr later only 400 g remain. Suppose 600 g of a radioactive substance are present initially and 3 yr later only
(a) Determine an exponential 300 g remain.
function that models this (a) Determine an exponential function that models this decay.
decay.
(b) How much of the substance (b) How much of the substance will be present after 6 yr?
will be present after 4 yr? Solution
Answers:
(a) We use the given values to find k in the exponential equation
(a)  y = 800e -0.277t (b)  264 g
y = y0 ekt.
Because the initial amount is 600 g, y0 = 600, which gives y = 600ekt. The
initial amount (600 g) decays to half that amount (300 g) in 3 yr, so its half-
life is 3 yr. Now we solve this exponential equation for k.
y = 600ekt Let y0 = 600.
300 = 600e3k Let y = 300 and t = 3.
0.5 = e3k Divide by 600.

ln 0.5 = ln e3k Take the natural logarithm on each side.

ln 0.5 = 3k ln e x = x, for all x.


ln 0.5
=k Divide by 3.
3
k ≈ - 0.231 Use a calculator.
A function that models the situation is
y = 600e -0.231t.

(b) To find the amount present after 6 yr, let t = 6.


y = 600e -0.231t Model from part (a)

y = 600e -0.231162 Let t = 6.


y = 600e -1.386 Multiply.
y ≈ 150 Use a calculator.

After 6 yr, 150 g of the substance will remain. ■


✔ Now Try Exercise 19.

Classroom Example 5 Example 5 Solving a Carbon Dating Problem


Suppose that the skeleton of a
woman who lived in the Classical Carbon-14, also known as radiocarbon, is a radioactive form of carbon that is
Greek period was discovered in found in all living plants and animals. After a plant or animal dies, the radiocar-
2005. Carbon-14 testing at that bon disintegrates. Scientists can determine the age of the remains by comparing
time determined that the skeleton the amount of radiocarbon with the amount present in living plants and animals.
3
contained 4 of the carbon-14 of This technique is called carbon dating. The amount of radiocarbon present
a living woman of the same size. after t years is given by
Estimate the year in which the
Greek woman died. y = y0 e -0.0001216t,
Answer: 361 bce where y0 is the amount present in living plants and animals.
(a) Find the half-life of carbon-14.
1
(b) Charcoal from an ancient fire pit on Java contained 4 the carbon-14 of a
living sample of the same size. Estimate the age of the charcoal.
Copyright Pearson. All Rights Reserved.

M04_LHSD7659_12_AIE_C04_pp405-496.indd 476 06/10/15 6:47 PM


NOT FOR SALE
4.6  Applications and Models of Exponential Growth and Decay 477

Solution
1
(a) If y0 is the amount of radiocarbon present in a living thing, then 2 y0 is half
this initial amount. We substitute and solve the given equation for t.
y = y0 e -0.0001216t Given equation

1
y = y0 e -0.0001216t Let y = 12 y0 .
2 0
1
= e -0.0001216t Divide by y0 .
2
1
ln = ln e -0.0001216t Take the natural logarithm on each side.
2
1
ln = - 0.0001216t ln e x = x, for all x.
2
1
ln 2
=t Divide by -0.0001216.
- 0.0001216
5700 ≈ t Use a calculator.

The half-life is 5700 yr.


1
(b) Solve again for t, this time letting the amount y = 4 y0 .

y = y0 e -0.0001216t Given equation


1
y = y0 e -0.0001216t Let y = 14 y0 .
4 0
1
= e -0.0001216t Divide by y0 .
4
1
ln = ln e -0.0001216t Take the natural logarithm on each side.
4
1
ln 4
=t ln e x = x; Divide by - 0.0001216.
- 0.0001216
t ≈ 11,400 Use a calculator.

The charcoal is 11,400 yr old. ■


✔ Now Try Exercise 23.

Example 6 Modeling Newton’s Law of Cooling

Newton’s law of cooling says that the rate at which a body cools is proportional
to the difference in temperature between the body and the environment around
it. The temperature ƒ1t2 of the body at time t in appropriate units after being
introduced into an environment having constant temperature T0 is

ƒ1t2 = T0 + Ce -kt, where C and k are constants.

A pot of coffee with a temperature of 100°C is set down in a room with a tem-
perature of 20°C. The coffee cools to 60°C after 1 hr.
(a) Write an equation to model the data.
(b) Find the temperature after half an hour.
(c) How long will it take for the coffee to cool to 50°C?
Copyright Pearson. All Rights Reserved.

M04_LHSD7659_12_AIE_C04_pp405-496.indd 477 06/10/15 6:47 PM


NOT FOR SALE
478 Chapter 4  Inverse, Exponential, and Logarithmic Functions

Classroom Example 6 Solution


Newton’s law of cooling also
applies to warming. Mary took a (a) We must find values for C and k in the given formula. As given, when t = 0,
leg of lamb out of her refrigerator, T0 = 20, and the temperature of the coffee is ƒ102 = 100.
which is set at 34°F, and placed
it in her oven, which she had ƒ1t2 = T0 + Ce -kt    Given function
preheated to 350°F. After 1 hr, her
100 = 20 + Ce -0k   Let t = 0, ƒ102 = 100, and T0 = 20.
meat thermometer registered 70°F.
(a) Write an equation to model 100 = 20 + C    e0 = 1
the data.
80 = C    Subtract 20.
(b) Find the temperature 90 min
after the leg of lamb was The following function models the data.
placed in the oven.
(c) Mary wants to serve the ƒ1t2 = 20 + 80e -kt   Let T0 = 20 and C = 80.
leg of lamb medium rare,
The coffee cools to 60°C after 1 hr, so when t = 1, ƒ112 = 60.
which requires an internal
temperature of 145°F. What is ƒ1t2 = 20 + 80e -kt    Above function with T0 = 20 and C = 80
the total amount of time it will
take to cook the leg of lamb? 60 = 20 + 80e -1k   Let t = 1 and ƒ112 = 60.
Answers: 40 = 80e -k    Subtract 20.
(a) ƒ1t2 = 350 - 316e -0.121t 1
(b) 86.5°F = e -k    Divide by 80.
2
(c) 3.576 hr, or about 3 hr, 35 min
1
ln = ln e -k    Take the natural logarithm on each side.
2
1
ln = -k    ln e x = x, for all x.
2
k ≈ 0.693    Multiply by -1, rewrite, and use a calculator.

Thus, the model is ƒ1t2 = 20 + 80e -0.693t.


1 1
(b) To find the temperature after 2 hr, let t = 2 in the model from part (a).

ƒ1t2 = 20 + 80e -0.693t    Model from part (a)

1
ƒ a b = 20 + 80e1-0.693211/22   Let t = 12 .
2
1
ƒ a b ≈ 76.6°C    Use a calculator.
2
(c) To find how long it will take for the coffee to cool to 50°C, let ƒ1t2 = 50.
ƒ1t2 = 20 + 80e -0.693t   Model from part (a)
50 = 20 + 80e -0.693t   Let ƒ1t2 = 50.
30 = 80e -0.693t    Subtract 20.
3
= e -0.693t    Divide by 80.
8
3
ln = ln e -0.693t    Take the natural logarithm on each side.
8
3
ln = - 0.693t    ln e x = x, for all x.
8
3
ln 8
t=    Divide by -0.693 and rewrite.
- 0.693
t ≈ 1.415 hr, or about 1 hr, 25 min ■
✔ Now Try Exercise 27.
Copyright Pearson. All Rights Reserved.

M04_LHSD7659_12_AIE_C04_pp405-496.indd 478 06/10/15 6:47 PM


NOT FOR SALE
4.6  Applications and Models of Exponential Growth and Decay 479

4.6 Exercises

1. B  
2.  D Concept Preview  Population Growth  A population is increasing according to the
3. C  
4.  A exponential function

5. B  
6.  D y = 2e0.02x,
7. C  
8.  A where y is in millions and x is the number of years. Match each question in Column I
1 1 1 1 with the correct procedure in Column II to answer the question.
9. 3 ln 3 10.  ln
6 3
I II
1 1 1 1
11. 100 ln 2 12.  200 ln 2
1. How long will it take for the population to A. Evaluate y = 2e0.0211/32.
1
13. 2 ln 4
1 1
14.  4 ln 9
1 triple?
2. When will the population reach 3 million? B. Solve 2e0.02x = 6.
15. (a)  440 g (b)  387 g
(c)  264 g (d)  22 yr 3. How large will the population be in 3 yr? C. Evaluate y = 2e0.02132.
4. How large will the population be in 4 months? D. Solve 2e0.02x = 3.

Concept Preview  Radioactive Decay  Strontium-90 decays according to the expo-


nential function
y = y0 e -0.0241t,
where t is time in years. Match each question in Column I with the correct procedure in
Column II to answer the question.
I II
5. If the initial amount of Strontium-90 is 200 g, A.  Solve 0.75y0 = y0 e -0.0241t.
how much will remain after 10 yr?
6. If the initial amount of Strontium-90 is 200 g, B.  Evaluate y = 200e -0.02411102.
how much will remain after 20 yr?
1
7. What is the half-life of Strontium-90? C.  Solve 2 y0 = y0 e -0.0241t .
8. How long will it take for any amount of D.  Evaluate y = 200e -0.02411202.
Strontium-90 to decay to 75% of its initial
amount?

(Modeling)  The exercises in this set are grouped according to discipline. They involve
exponential or logarithmic models. See Examples 1–6.
Physical Sciences  (Exercises 9–28)
An initial amount of a radioactive substance y0 is given, along with information about
the amount remaining after a given time t in appropriate units. For an equation of the
form y = y0 ekt that models the situation, give the exact value of k in terms of natural
logarithms.
9. y0 = 60 g; After 3 hr, 20 g remain. 10. y0 = 30 g; After 6 hr, 10 g remain.
11. y0 = 10 mg; The half-life is 100 days. 12. y0 = 20 mg; The half-life is 200 days.
13. y0 = 2.4 lb; After 2 yr, 0.6 lb remains. 14. y0 = 8.1 kg; After 4 yr, 0.9 kg remains.

Solve each problem.


15. Decay of Lead  A sample of 500 g of radioactive lead-210 decays to polonium-210
according to the function
A1t2 = 500e -0.032t,
where t is time in years. Find the amount of radioactive lead remaining after
(a) 4 yr,    (b) 8 yr,    (c) 20 yr.    (d)  Find the half-life.

Copyright Pearson. All Rights Reserved.

M04_LHSD7659_12_AIE_C04_pp405-496.indd 479 06/10/15 6:47 PM


NOT FOR SALE
480 Chapter 4  Inverse, Exponential, and Logarithmic Functions

16. (a)  404 g (b)  327 g 16. Decay of Plutonium Repeat Exercise 15 for 500 g of plutonium-241, which decays
(c)  173 g (d)  13 yr according to the function A1t2 = A0 e -0.053t, where t is time in years.
17. 1600 yr 18.  12 yr
17. Decay of Radium  Find the half-life of radium-226, which decays according to the
19. 3.6 g 20.  0.88 g function A1t2 = A0 e -0.00043t, where t is time in years.
21. 16 days
22. Magnitude 1 is about 6.3 18. Decay of Tritium  Find the half-life of tritium, a radioactive isotope of hydrogen,
times as great as magnitude 3. which decays according to the function A1t2 = A0 e -0.056t, where t is time in years.
23. 9000 yr 24.  4200 yr 19. Radioactive Decay  If 12 g of a radioactive substance are present initially and 4 yr
25. 15,600 yr 26.  43°C later only 6.0 g remain, how much of the substance will be present after 7 yr?
27. 6.25°C 28.  81.25°C
20. Radioactive Decay  If 1 g of strontium-90 is present initially, and 2 yr later 0.95 g
29. (a)  3% compounded quarterly remains, how much strontium-90 will be present after 5 yr?
(b) $826.95 21. Decay of Iodine  How long will it take any quantity of iodine-131 to decay to 25%
of its initial amount, knowing that it decays according to the exponential function
A1t2 = A0 e -0.087t, where t is time in days?
22. Magnitude of a Star  The magnitude M
of a star is modeled by

5 I
M=6- log ,
2 I0

where I0 is the intensity of a just-visible


star and I is the actual intensity of the
star being measured. The dimmest stars
are of magnitude 6, and the brightest
are of magnitude 1. Determine the ratio
of light intensities between a star of
magnitude 1 and a star of magnitude 3.
23. Carbon-14 Dating  Suppose an Egyptian mummy is discovered in which the
amount of carbon-14 present is only about one-third the amount found in living
human beings. How long ago did the Egyptian die?
24. Carbon-14 Dating  A sample from a refuse deposit near the Strait of Magellan had
60% of the carbon-14 of a contemporary sample. How old was the sample?
25. Carbon-14 Dating  Paint from the Lascaux caves of France contains 15% of the
normal amount of carbon-14. Estimate the age of the paintings.
26. Dissolving a Chemical  The amount of a chemical that will dissolve in a solution
increases exponentially as the (Celsius) temperature t is increased according to the
model
A1t2 = 10e0.0095t.

At what temperature will 15 g dissolve?


27. Newton’s Law of Cooling  Boiling water, at 100°C, is placed in a freezer at 0°C.
The temperature of the water is 50°C after 24 min. Find the temperature of the water
to the nearest hundredth after 96 min. (Hint: Change minutes to hours.)
28. Newton’s Law of Cooling  A piece of metal is heated to 300°C and then placed in
a cooling liquid at 50°C. After 4 min, the metal has cooled to 175°C. Find its tem-
perature to the nearest hundredth after 12 min. (Hint: Change minutes to hours.)

Finance  (Exercises 29–34)


29. Comparing Investments  Russ, who is self-employed, wants to invest $60,000 in
a pension plan. One investment offers 3% compounded quarterly. Another offers
2.75% compounded continuously.
(a) Which investment will earn more interest in 5 yr?
(b) How much more will the better plan earn?

Copyright Pearson. All Rights Reserved.

M04_LHSD7659_12_AIE_C04_pp405-496.indd 480 06/10/15 6:48 PM


NOT FOR SALE
4.6  Applications and Models of Exponential Growth and Decay 481

30. 5.61 yr 30. Growth of an Account  If Russ (see Exercise 29) chooses the plan with continuous
31. 27.73 yr compounding, how long will it take for his $60,000 to grow to $70,000?
32. The time will be divided by 3. 31. Doubling Time  Find the doubling time of an investment earning 2.5% interest if
33. 36.62 yr interest is compounded continuously.
34. 39 yr, 347 days
32. Doubling Time  If interest is compounded continuously and the interest rate is
35. (a) 315 (b) 229 (c) 142 tripled, what effect will this have on the time required for an investment to double?
36. 1968 33. Growth of an Account  How long will it take an investment to triple if interest is
37. (a) P0 = 1; a ≈ 1.01355 compounded continuously at 3%?
(b) 1.3 billion
(c) 2030 34. Growth of an Account  Use the Table feature of a graphing calculator to find how
38. (a) 961,000 long it will take $1500 invested at 2.75% compounded daily to triple in value.
(b) 7.2 yr Zoom in on the solution by systematically decreasing the increment for x. Find the
(c) 17.3 yr answer to the nearest day. (Find the answer to the nearest day by eventually letting
1
the increment of x equal 365 . The decimal part of the solution can be multiplied by
365 to determine the number of days greater than the nearest year. For example,
if the solution is determined to be 16.2027 yr, then multiply 0.2027 by 365 to get
73.9855. The solution is then, to the nearest day, 16 yr, 74 days.) Confirm the
answer algebraically.

Social Sciences  (Exercises 35–44)


35. Legislative Turnover  The turnover of leg-
islators is a problem of interest to political
scientists. It was found that one model of
legislative turnover in a particular body was
M1t2 = 434e -0.08t,

where M1t2 represents the number of con-


tinuously serving members at time t. Here,
t = 0 represents 1965, t = 1 represents 1966,
and so on. Use this model to approximate the
number of continuously serving members in
each year.
(a) 1969   (b) 1973   (c) 1979

36. Legislative Turnover  Use the model in Exercise 35 to determine the year in which
the number of continuously serving members was 338.
37. Population Growth  In 2000 India’s population reached 1 billion, and it is projected
to be 1.4 billion in 2025. (Source: U.S. Census Bureau.)
(a) Find values for P0 and a so that P1x2 = P0 a x - 2000 models the population of India
in year x. Round a to five decimal places.
(b) Predict India’s population in 2020 to the nearest tenth of a billion.
(c) In what year is India’s population expected to reach 1.5 billion?
38. Population Decline  A midwestern city finds its residents moving to the suburbs. Its
population is declining according to the function
P1t2 = P0 e -0.04t,

where t is time measured in years and P0 is the population at time t = 0. Assume that
P0 = 1,000,000.
(a) Find the population at time t = 1 to the nearest thousand.
(b) How long, to the nearest tenth of a year, will it take for the population to decline
to 750,000?
(c) 
How long, to the nearest tenth of a year, will it take for the population to decline
to half the initial number?

Copyright Pearson. All Rights Reserved.

M04_LHSD7659_12_AIE_C04_pp405-496.indd 481 06/10/15 6:48 PM


NOT FOR SALE
482 Chapter 4  Inverse, Exponential, and Logarithmic Functions

39. (a) $8412 39. Health Care Spending  Out-of-pocket spending in the United States for health care
(b) 2010 increased between 2008 and 2012. The function
40. $998 billion
ƒ1x2 = 7446e0.0305x
41. (a) $14,542
(b) $16,162 models average annual expenditures per household, in dollars. In this model, x rep-
(c) $17,494 resents the year, where x = 0 corresponds to 2008. (Source: U.S. Bureau of Labor
42. (a) 348 yr Statistics.)
(b) 3974 yr (a) Estimate out-of-pocket household spending on health care in 2012 to the nearest
(c) 2288 yr dollar.
(b) In what year did spending reach $7915 per household?
40. Recreational Expenditures Personal
consumption expenditures for recre-
ation in billions of dollars in the United
States during the years 2000–2013 can
be approximated by the function
A1t2 = 632.37e0.0351t,
where t = 0 corresponds to the year
2000. Based on this model, how much
were personal consumption expenditures
in 2013 to the nearest billion? (Source:
U.S. Bureau of Economic Analysis.)
41. Housing Costs  Average annual per-household spending on housing over the years
2000–2012 is approximated by
H = 12,744e0.0264t,
where t is the number of years since 2000. Find H to the nearest dollar for each year.
(Source: U.S. Bureau of Labor Statistics.)
(a) 2005     (b) 2009     (c) 2012
42. Evolution of Language  The number of years, n, since two independently evolving
languages split off from a common ancestral language is approximated by
n ≈ - 7600 log r,
where r is the proportion of words from the ancestral language common to both
languages. Find each of the following to the nearest year.
(a) Find n if r = 0.9. (b)  Find n if r = 0.3.
(c) How many years have elapsed since the split if half of the words of the ancestral
language are common to both languages?
43. School District Growth  Student enrollment in the Wentzville School District, one
of the fastest-growing school districts in the state of Missouri, has projected growth
as shown in the graph.

y Enrollment Projections

20,000 19,915
19,000
Projected Enrollment

18,000

17,000

16,000

15,000

14,000 14,225
0 x
13–14 14–15 15–16 16–17 17–18 18–19 19–20 20–21 21–22 22–23 23–24
School Year
Source: Wentzville School District.

Copyright Pearson. All Rights Reserved.

M04_LHSD7659_12_AIE_C04_pp405-496.indd 482 06/10/15 6:48 PM


NOT FOR SALE
4.6  Applications and Models of Exponential Growth and Decay 483

43. (a) y = 14,225e0.034x (a) Use the model y = y0 ekx to find an exponential function that gives the projected
(b) 2025–26 enrollment y in school year x. Let the school year 2013–14 correspond to x = 0,
44. (a) y = 100e0.309x 2014–15 correspond to x = 1, and so on, and use the two points indicated on the
(b) 1,061,475 graph.
(b) Estimate the school year for which projected enrollment will be 21,500 students.
45. (a) 15,000
(b) 9098 44. YouTube Views  The number of views of a YouTube video increases after the num-
(c) 5249 ber of hours posted as shown in the table.
46. 14 days
47. (a) 611 million Hour Number of Views
(b) 746 million
20 100
(c) 1007 million
48. 6.9 days 25 517
49. 13.2 hr 30 2015
35 10,248

(a) Use the model y = y0 ekx to find an exponential function that gives projected
number of views y after number of hours x. Let hour 20 correspond to x = 0,
hour 25 correspond to x = 5, and so on, and use the first and last data values
given in the table.
(b) Estimate the number of views after 50 hr.

Life Sciences  (Exercises 45–50)


45. Spread of Disease  During an epidemic, the number of people who have never had
the disease and who are not immune (they are susceptible) decreases exponentially
according to the function
ƒ1t2 = 15,000e -0.05t,
where t is time in days. Find the number of susceptible people at each time.
(a) at the beginning of the epidemic   (b)  after 10 days   (c)  after 3 weeks
46. Spread of Disease  Refer to Exercise 45 and determine how long it will take, to
the nearest day, for the initial number of people susceptible to decrease to half its
amount.
47. Growth of Bacteria  The growth of bacteria makes it necessary to time-date some
food products so that they will be sold and consumed before the bacteria count is
too high. Suppose for a certain product the number of bacteria present is given by
ƒ1t2 = 500e0.1t,
where t is time in days and the value of ƒ1t2 is in millions. Find the number of bac-
teria, in millions, present at each time.
(a) 2 days   (b)  4 days   (c)  1 week
48. Growth of Bacteria  How long will it take the bacteria population in Exercise 47 to
double? Round the answer to the nearest tenth.
49. Medication Effectiveness  Drug effectiveness decreases over time. If, each hour, a
drug is only 90% as effective as the previous hour, at some point the patient will not
be receiving enough medication and must receive another dose. If the initial dose
was 200 mg and the drug was administered 3 hr ago, the expression 20010.9023,
which equals 145.8, represents the amount of effective medication still in the system.
(The exponent is equal to the number of hours since the drug was administered.)
The amount of medication still available in the system is given by the function
ƒ1t2 = 20010.902t.
In this model, t is in hours and ƒ1t2 is in milligrams. How long will it take for this
initial dose to reach the dangerously low level of 50 mg? Round the answer to the
nearest tenth.

Copyright Pearson. All Rights Reserved.

M04_LHSD7659_12_AIE_C04_pp405-496.indd 483 06/10/15 6:48 PM


NOT FOR SALE
484 Chapter 4  Inverse, Exponential, and Logarithmic Functions

50. (a) 250,000 50. Population Size  Many environmental situations place effective limits on the growth
G(x) =
100+ 2400e−x of the number of an organism in an area. Many such limited-growth situations are
2500
described by the logistic function
MG0
G1x2 = ,
G0 + 1M - G02e -kMx
0 8
0 where G0 is the initial number present, M is the maximum possible size of the popu-
(b) 590; 589 lation, and k is a positive constant. The screens illustrate a typical logistic function
(c) 2.8; 2.7726 calculation and graph.

51. 2020
52. (a) S112 ≈ 45,000;
70
S132 ≈ 37,000
(b) S122 ≈ 72,000;
S1102 ≈ 49,000
53. 6.9 yr 54.  27.5 yr
−1 65
    −5

Assume that G0 = 100, M = 2500, k = 0.0004, and x = time in decades (10-yr


periods).
(a) Use a calculator to graph the function, using 0 … x … 8 and 0 … y … 2500.
(b) Estimate the value of G122 from the graph. Then evaluate G122 algebraically to
find the population after 20 yr.
(c) 
Find the x-coordinate of the intersection of the curve with the horizontal line
y = 1000 to estimate the number of decades required for the population to reach
1000. Then solve G1x2 = 1000 algebraically to obtain the exact value of x.

Economics  (Exercises 51–56)


51. Consumer Price Index  The U.S. Consumer Price Index for the years 1990–2013 is
approximated by
A1t2 = 100e0.0264t,
where t represents the number of years after 1990. (Since A1162 is about 153, the
amount of goods that could be purchased for $100 in 1990 cost about $153 in 2006.)
Use the function to determine the year in which costs will be 125% higher than in
1990. (Source: U.S. Bureau of Labor Statistics.)
52. Product Sales  Sales of a product, under relatively stable market conditions but
in the absence of promotional activities such as advertising, tend to decline at a
constant yearly rate. This rate of sales decline varies considerably from product to
product, but it seems to remain the same for any particular product. The sales
decline can be expressed by the function
S1t2 = S0 e -at,
where S1t2 is the rate of sales at time t measured in years, S0 is the rate of sales at
time t = 0, and a is the sales decay constant.
(a) Suppose the sales decay constant for a particular product is a = 0.10. Let
S0 = 50,000 and find S112 and S132 to the nearest thousand.
(b) Find S122 and S1102 to the nearest thousand if S0 = 80,000 and a = 0.05.
53. Product Sales  Use the sales decline function given in Exercise 52. If a = 0.10,
S0 = 50,000, and t is time measured in years, find the number of years it will take
for sales to fall to half the initial sales. Round the answer to the nearest tenth.
54. Cost of Bread  Assume the cost of a loaf of bread is $4. With continuous compound-
ing, find the number of years, to the nearest tenth, it would take for the cost to triple
at an annual inflation rate of 4%.

Copyright Pearson. All Rights Reserved.

M04_LHSD7659_12_AIE_C04_pp405-496.indd 484 06/10/15 6:48 PM


NOT FOR SALE
  Summary Exercises on Functions: Domains and Defining Equations 485

55. 11.6 yr 56.  34.7 yr 55. Electricity Consumption  Suppose that in a certain area the consumption of elec-
tricity has increased at a continuous rate of 6% per year. If it continued to increase
57. (a) 0.065; 0.82; Among at this rate, find the number of years, to the nearest tenth, before twice as much elec-
people age 25, 6.5% have tricity would be needed.
some CHD, while among
people age 65, 82% have 56. Electricity Consumption  Suppose a conservation campaign, together with higher
some CHD. rates, caused demand for electricity to increase at only 2% per year. (See Exercise 55.)
(b) 48 yr Find the number of years, to the nearest tenth, before twice as much electricity
58. (a) would be needed.

(Modeling)  Solve each problem that uses a logistic function.


57. Heart Disease  As age increases, so does the likelihood of coronary heart disease
(CHD). The fraction of people x years old with some CHD is modeled by
The maximum height 0.9
appears to be 50 ft. ƒ1x2 = .
1 + 271e -0.122x
(b) y=
50
1+ 47.5e−0.22x (Source: Hosmer, D., and S. Lemeshow, Applied Logistic Regression, John Wiley
60
and Sons.)
(a) Evaluate ƒ1252 and ƒ1652 to the nearest hundredth. Interpret the results.
(b) At what age, to the nearest year, does this likelihood equal 50%?
10 70 58. Tree Growth  The height of a certain tree in feet
0
The horizontal asymptote after x years is modeled by
is y = 50. It indicates
50
that this tree cannot grow ƒ1x2 = .
taller than 50 ft. 1 + 47.5e -0.22x
(c) 19.4 yr
(a) Make a table for ƒ starting at x = 10, and
incrementing by 10. What appears to be the
maximum height of the tree?
(b) Graph ƒ and identify the horizontal asymptote.
Explain its significance.
(c) After how many years was the tree 30 ft tall?
Round to the nearest tenth.

Summary Exercises on Functions: Domains and Defining Equations

Finding the Domain of a Function: A Summary   To find the domain of


a function, given the equation that defines the function, remember that the value
of x input into the equation must yield a real number for y when the function is
evaluated. For the functions studied so far in this book, there are three cases to
consider when determining domains.

Guidelines for Domain Restrictions

1. No input value can lead to 0 in a denominator, because division by 0 is


undefined.
2. No input value can lead to an even root of a negative number, because this
situation does not yield a real number.
3. No input value can lead to the logarithm of a negative number or 0,
because this situation does not yield a real number.

Copyright Pearson. All Rights Reserved.

M04_LHSD7659_12_AIE_C04_pp405-496.indd 485 06/10/15 6:48 PM


NOT FOR SALE
486 Chapter 4  Inverse, Exponential, and Logarithmic Functions

Unless otherwise specified, we determine domains as follows.


• The domain of a polynomial function is the set of all real numbers.
• The domain of an absolute value function is the set of all real numbers
for which the expression inside the absolute value bars (the argument) is
defined.
• If a function is defined by a rational expression, the domain is the set of
all real numbers for which the denominator is not zero.
• The domain of a function defined by a radical with even root index is the
set of all real numbers that make the radicand greater than or equal to zero.
If the root index is odd, the domain is the set of all real numbers for which
the radicand is itself a real number.
• For an exponential function with constant base, the domain is the set of all
real numbers for which the exponent is a real number.
• For a logarithmic function, the domain is the set of all real numbers that
make the argument of the logarithm greater than zero.

Determining Whether an Equation Defines y as a Function of x 


For y to be a function of x, it is necessary that every input value of x in
the domain leads to one and only one value of y.
To determine whether an equation such as

x - y 3 = 0 or x - y 2 = 0

represents a function, solve the equation for y. In the first equation above, doing
so leads to
3
y= 2 x.

Notice that every value of x in the domain (that is, all real numbers) leads to one
and only one value of y. So in the first equation, we can write y as a function of x.
However, in the second equation above, solving for y leads to

y = { 2x.

If we let x = 4, for example, we get two values of y: - 2 and 2. Thus, in the


1. 1- ∞, ∞2
second equation, we cannot write y as a function of x.
2. C 72 , ∞ B
3. 1- ∞, ∞2
4. 1- ∞, 62 ´ 16, ∞2 Exercises
5. 1- ∞, ∞2 Find the domain of each function. Write answers using interval notation.
6. 1- ∞, - 34 ´ 33, ∞2 1. ƒ1x2 = 3x - 6 2. ƒ1x2 = 22x - 7 3. ƒ1x2 = 0 x + 4 0
7. 1- ∞, - 32 ´ 1- 3, 32 ´ 13, ∞2
x+2 -2
8. 1- ∞, ∞2 4. ƒ1x2 = 5. ƒ1x2 = 6. ƒ1x2 = 2x 2 - 9
x-6 x2 + 7
9. 1- 4, 42
x2 + 7 3 3
10. 1- ∞, - 72 ´ 13, ∞2 7. ƒ1x2 = 2 8. ƒ1x2 = 2x + 7x - 4 9. ƒ1x2 = log5116 - x 22
x -9
11. 1- ∞, - 14 ´ 38, ∞2
12. 1- ∞, 02 ´ 10, ∞2 x+7
10. ƒ1x2 = log 11. ƒ1x2 = 2x 2 - 7x - 8 12. ƒ1x2 = 21/x
13. 1- ∞, ∞2 x-3
14. 1- ∞, - 52 ´ 1- 5, ∞2 1 x 2 - 25
13. ƒ1x2 = 14. ƒ1x2 = 15. ƒ1x2 = 2x 3 - 1
15. 31, ∞2 2x 2 -x+7 x+5

Copyright Pearson. All Rights Reserved.

M04_LHSD7659_12_AIE_C04_pp405-496.indd 486 06/10/15 6:48 PM


NOT FOR SALE
  Summary Exercises on Functions: Domains and Defining Equations 487

16. A - ∞, - 25 B ´ A - 25, 25 B x3 - 1
16. ƒ1x2 = ln 0 x 2 - 5 0 17. ƒ1x2 = e x + x + 4
2
18. ƒ1x2 =
x2 - 1
´ A 25, ∞ B
17. 1- ∞, ∞2 -1 31
19. ƒ1x2 = 20. ƒ1x2 =
18. 1- ∞, - 12 ´ 1- 1, 12 ´ 11, ∞2 B x 3 - 1
B x 3 - 8

19. 1- ∞, 12
21. ƒ1x2 = ln 1x 2 + 12 22. ƒ1x2 = 21x - 321x + 221x - 42
20. 1- ∞, 22 ´ 12, ∞2
21. 1- ∞, ∞2 x+2 2 12
23. ƒ1x2 = log a b 24. ƒ1x2 = 214 - x221x + 32
22. 3 - 2, 34 ´ 34, ∞2 x-3
23. 1- ∞, - 22 ´ 1- 2, 32 ´ 13, ∞2 1
25. ƒ1x2 = e 0 1/x 0 26. ƒ1x2 =
24. 3 - 3, ∞2 0 x2 - 70
25. 1- ∞, 02 ´ 10, ∞2
27. ƒ1x2 = x 100 - x 50 + x 2 + 5 28. ƒ1x2 = 2- x 2 - 9
26. A - ∞, - 27 B ´ A - 27, 27 B
4 3
29. ƒ1x2 = 216 - x 4 30. ƒ1x2 = 216 - x 4
´ A 27, ∞ B
27. 1- ∞, ∞2 28.  ∅ x 2 - 2x - 63 5
31. ƒ1x2 = 32. ƒ1x2 = 25 - x
29. 3 - 2, 24 30.  1- ∞, ∞2 B x 2 + x - 12
31. 1- ∞, - 74 ´ 1- 4, 32 ´ 39, ∞2
-1
32. 1- ∞, ∞2 33. ƒ1x2 = P 25 - x P 34. ƒ1x2 =
B -3
x
33. 1- ∞, 54 34.  1- ∞, 32
35. 1- ∞, 42 ´ 14, ∞2 1
36. ƒ1x2 = 6 x - 9
2
35. ƒ1x2 = log ` `
36. 1- ∞, ∞2
4-x
3
37. 1- ∞, - 54 ´ 35, ∞2 37. ƒ1x2 = 6 2x - 25
2
38. ƒ1x2 = 6 2 x - 25
2

38. 1- ∞, ∞2
-3 -2
39. 1- 2, 62 39. ƒ1x2 = ln a b 40. ƒ1x2 =
1x + 221x - 62 log x
40. 10, 12 ´ 11, ∞2

41. A 42. B
43. C 44. D Determine which one of the choices (A, B, C, or D) is an equation in which y can be
45. A 46. B written as a function of x.
47. D 48. C 41. A.  3x + 2y = 6 B.  x = 2 0 y 0 C.  x = 0 y + 3 0 D.  x 2 + y 2 = 9
49. C 50. B
42. A.  3x 2 + 2y 2 = 36 B.  x 2 + y - 2 = 0 C.  x - 0 y 0 = 0 D.  x = y 2 - 4
1
43. A.  x = 2y 2 B.  x = log y 2 C.  x 3 + y 3 = 5 D.  x =
y2 + 3
x2 y2 x2 y2
44. A.  + = 1 B.  x = 5y 2 - 3 C.  - = 1 D.  x = 10 y
4 4 4 9
2-y
C.  2x = 0 y + 1 0 D.  2x = y 2
4
45. A.  x = B.  x = ln 1y + 122
y+3
B.  ey + 2 = x C.  e 0 y 0 = x D.  10 0 y + 2 0 = x
2
46. A.  ey = x
1 1 1 1
47. A.  x 2 = B.  x + 2 = C.  3x = D.  2x =
y2 y2 y4 y3
1
48. A.  0 x 0 = 0 y 0 B.  x = 0 y 2 0 C.  x = D.  x 4 + y 4 = 81
y
x2 y2 y2 x2 x y x2 y2
49. A.  - = 1 B.  - = 1 C.  - = 0 D.  - =0
4 9 4 9 4 9 4 9

50. A.  y 2 - 21x + 222 = 0 B.  y - 21x + 222 = 0

C.  y 6 - 21x + 122 = 0 D.  y 4 - 2x 2 = 0

Copyright Pearson. All Rights Reserved.

M04_LHSD7659_12_AIE_C04_pp405-496.indd 487 06/10/15 6:48 PM


NOT FOR SALE
488 Chapter 4  Inverse, Exponential, and Logarithmic Functions

Chapter 4 Test Prep

Key Terms
4.1 one-to-one function future value 4.3 logarithm 4.4 common logarithm
inverse function present value base pH
4.2 exponential function compound amount argument natural logarithm
exponential equation continuous logarithmic equation 4.6 doubling time
compound interest compounding logarithmic function half-life

New Symbols
ƒ−1 1 x2 inverse of ƒ1x2 log x common (base 10) logarithm of x
e a constant, approximately 2.718281828459045 ln x natural (base e) logarithm of x
log a x logarithm of x with the base a

Quick Review
Concepts Examples

4.1 Inverse Functions


One-to-One Function
In a one-to-one function, each x-value corresponds to only The function y = ƒ1x2 = x 2 is not one-to-one, because
one y-value, and each y-value corresponds to only one y = 16, for example, corresponds to both x = 4 and
x-value. x = - 4.
A function ƒ is one-to-one if, for elements a and b in the
domain of ƒ,
a 3 b implies ƒ1 a2 3 ƒ1 b2 .

Horizontal Line Test


A function is one-to-one if every horizontal line intersects The graph of ƒ1x2 = 2x - 1 is a straight line with slope 2.
the graph of the function at most once. ƒ is a one-to-one function by the horizontal line test.
Inverse Functions
Let ƒ be a one-to-one function. Then g is the inverse func- Find the inverse of ƒ.
tion of ƒ if
ƒ1x2 = 2x - 1 Given function
1 ƒ ° g2 1 x2 = x  for every x in the domain of g y = 2x - 1 Let y = ƒ1x2.
and x = 2y - 1 Interchange x and y.
1 g ° ƒ2 1 x2 = x  for every x in the domain of ƒ. x+1
y= Solve for y.
2
To find g1x2, interchange x and y in y = ƒ1x2, solve for y,
and replace y with g1x2, which is ƒ -11x2. x+1
ƒ - 11x2 = Replace y with ƒ - 11x2.
2
1 1 x + 1
ƒ - 11x2 = x + 2 =
x
2 +
1
2
1
= 2x +
1
2
2 2

Copyright Pearson. All Rights Reserved.

M04_LHSD7659_12_AIE_C04_pp405-496.indd 488 06/10/15 6:48 PM


NOT FOR SALE
chapter 4  Test Prep 489

Concepts Examples

4.2 Exponential Functions


Additional Properties of Exponents
For any real number a 7 0, a ≠ 1, the following hold true.
(a)  a x is a unique real number for all real numbers x. (a)  2x is a unique real number for all real numbers x.
(b)  a b = a c if and only if b = c. (b)  2x = 23 if and only if x = 3.
(c) If a 7 1 and m 6 n, then a m 6 a n. (c)  25 6 210, because 2 7 1 and 5 6 10.
5 10
(d) If 0 6 a 6 1 and m 6 n, then a m 7 a n. (d)  A 12 B 7 A 12 B because 0 6
1
2 6 1 and 5 6 10.
Exponential Function
If a 7 0 and a ≠ 1, then the exponential function with base ƒ1x2 = 3x is the exponential function with base 3.
a is ƒ1 x2 = ax.
x y y
Graph of ƒ1 x2 = ax f(x) = 3 x
1
1 -1
1. The points A - 1, a B , 10, 12, and 11, a2 are on the graph. 3 4
0 1 (1, 3)
2. If a 7 1, then ƒ is an increasing function. 1 3 2
  Q–1, 1R (0, 1)
3
If 0 6 a 6 1, then ƒ is a decreasing function. x
0 1
3. The x-axis is a horizontal asymptote.
4. The domain is 1 - ∞, ∞2, and the range is 10, ∞2.

4.3 Logarithmic Functions


Logarithm
For all real numbers y and all positive numbers a and x, log3 81 = 4  is equivalent to  34 = 81.
where a ≠ 1, y = log a x is equivalent to x = a y.
Logarithmic Function
If a 7 0, a ≠ 1, and x 7 0, then the logarithmic function ƒ1x2 = log3 x is the logarithmic function with base 3.
with base a is ƒ1 x2 = log a x.
x y y
Graph of ƒ1 x2 = log a x f(x) = log 3 x
1
1 -1
1. The points A a, - 1 B , 11, 02, and 1a, 12 are on the graph. 3
1 0 (1, 0) (3, 1)
1
2. If a 7 1, then ƒ is an increasing function. 3 1 x
0 1 3
If 0 6 a 6 1, then ƒ is a decreasing function.
–2 Q 1 , –1R
3
3. The y-axis is a vertical asymptote.
4. The domain is 10, ∞2, and the range is 1 - ∞, ∞2.
Properties of Logarithms
For x 7 0, y 7 0, a 7 0, a ≠ 1, and any real number r, the
following properties hold.
log a xy = log a x + log a y  Product property log2 13 # 52 = log2 3 + log2 5
x 3
log a = log a x − log a y  Quotient property log2 = log2 3 - log2 5
y 5
log a xr = r log a x   Power property log6 35 = 5 log6 3
log a 1 = 0   Logarithm of 1 log10 1 = 0
log a a = 1   Base a logarithm of a log10 10 = 1
Theorem on Inverses
For a 7 0 and a ≠ 1, the following properties hold.
alog a x = x  1 x + 02  and log a ax = x 2log2 5 = 5 and log2 25 = 5

Copyright Pearson. All Rights Reserved.

M04_LHSD7659_12_AIE_C04_pp405-496.indd 489 06/10/15 6:48 PM


NOT FOR SALE
490 Chapter 4  Inverse, Exponential, and Logarithmic Functions

Concepts Examples

4.4 Evaluating Logarithms and the Change-of-Base Theorem


Common and Natural Logarithms
For all positive numbers x, base 10 logarithms and base e Approximate log 0.045 and ln 247.1.
logarithms are written as follows.
log x = log10 x  Common logarithm log 0.045 ≈ - 1.3468 
Use a calculator.
ln x = log e x   Natural logarithm ln 247.1 ≈ 5.5098

Change-of-Base Theorem
For any positive real numbers x, a, and b, where a ≠ 1 and Approximate log8 7.
b ≠ 1, the following holds.
log b x log 7 ln 7
log a x = log8 7 = = ≈ 0.9358  Use a calculator.
log b a log 8 ln 8

4.5 Exponential and Logarithmic Equations


Property of Logarithms
If x 7 0, y 7 0, a 7 0, and a ≠ 1, then the following holds. Solve.
x = y  is equivalent to  log a x = log a y. e5x = 10
ln e5x = ln 10  Take natural logarithms.
5x = ln 10   ln e x = x, for all x.
ln 10
x=   Divide by 5.
5
x ≈ 0.461  Use a calculator.
The solution set can be written with the exact value,
E ln510 F, or with the approximate value, 50.4616.
log2 1x 2 - 32 = log2 6
x2 - 3 = 6   Property of logarithms
x2 =9   Add 3.
x = {3   Take square roots.
Both values check, so the solution set is 5{ 36.

4.6 Applications and Models of Exponential Growth and Decay


Exponential Growth or Decay Function
Let y0 be the amount or number present at time t = 0. The formula for continuous compounding,
Then, under certain conditions, the amount present at any
A = Pe r t,
time t is modeled by
is an example of exponential growth. Here, A is the
y = y0 ekt, where k is a constant.
compound amount if P dollars are invested at an annual
interest rate r for t years.
If P = $200, r = 3%, and t = 5 yr, find A.
A = Pert
A = 200e0.03152  Substitute.
A ≈ $232.37   Use a calculator.

Copyright Pearson. All Rights Reserved.

M04_LHSD7659_12_AIE_C04_pp405-496.indd 490 20/10/15 11:05 AM


NOT FOR SALE
chapter 4  Review Exercises 491

Chapter 4 Review Exercises


Determine whether each function as graphed or defined is one-to-one.
1. not one-to-one
2. one-to-one 1. y 2. y 3. y

3. one-to-one
4. one-to-one
5. not one-to-one x x x
6. not one-to-one 0 0 0

3
7. ƒ - 11x2 = 2 x+3
8. not possible
4. y = x 3 + 1 5. y = 1x + 322 6. y = 23x 2 + 2
9. It represents the number
of years after 2004 for the
investment to reach $50,000. Find the inverse of each function that is one-to-one.
10. yes
7. ƒ1x2 = x 3 - 3 8. ƒ1x2 = 225 - x 2
11. one-to-one 12. true

13. B 14.  A Concept Check  Work each problem.


15. C 16.  D
9. Suppose ƒ1t2 is the amount an investment will grow to t years after 2004. What does
17. log2 32 = 5 ƒ - 11$50,0002 represent?
1
18. log100 10 = 2 10. The graphs of two functions are shown. Based on their graphs, are these functions
4
19. log3/4 3 = -1 inverses?
20. y

10
6 1
f (x) = Q R x + 2  1
5
4
2 −16.1 16.1
3 3
0 x
y = 1 –2
−10
1- ∞, ∞2; 1- 1, ∞2
11. To have an inverse, a function must be a(n) function.
21. 10 3 = 1000 22.  9 3/2 = 27
23. e1/2 = 2e 12. True or false? The x-coordinate of the x-intercept of the graph of y = ƒ1x2 is the
y-coordinate of the y-intercept of the graph of y = ƒ - 11x2.

Match each equation with the figure that most closely resembles its graph.
13. y = log0.3 x 14. y = ex 15. y = ln x 16. y = 0.3x
A. y B. y C. y D.  y

x x x x
0 0 0 0

Write each equation in logarithmic form.


3 -1 4
17. 25 = 32 18. 100 1/2 = 10 19. a b =
4 3
x+2
20. Graph ƒ1x2 = A 15 B - 1. Give the domain and range.

Write each equation in exponential form.


3 1
21. log 1000 = 3 22. log9 27 = 23. ln 2e =
2 2

Copyright Pearson. All Rights Reserved.

M04_LHSD7659_12_AIE_C04_pp405-496.indd 491 06/10/15 6:48 PM


NOT FOR SALE
492 Chapter 4  Inverse, Exponential, and Logarithmic Functions

24. 3 24. Concept Check  What is the base of the logarithmic function whose graph contains
25. 2 the point 181, 42?

26. 2 log5 x + 4 log5 y + 25. Concept Check  What is the base of the exponential function whose graph contains
1

1
5 13 log5 m + log5 p2 the point A - 4, 16 B ?
27. log3 m + log3 n -
  log3 5 - log3 r
28. This cannot be simplified. Use properties of logarithms to rewrite each expression. Simplify the result if possible.
Assume all variables represent positive real numbers.
29. - 1.3862 30.  1.6590 mn
26. log5 A x 2y 4 2m3p B
5
31. 11.8776 32.  6.1527 27. log3 28. log7 17k + 5r 22
5r
33. 1.1592 34.  6.0486

35. E F
22
5 36. 51.7926 Use a calculator to find an approximation to four decimal places for each logarithm.
37. 53.6676 38. 52.2696
29. log 0.0411 30. log 45.6 31. ln 144,000
39. 5 - 13.2576 40. 52.3866
5
41. 5 - 0.4856 42. 5 - 0.1236 32. ln 470 33. log2/3 34. log3 769
8
43. 52.1026 44. 5 - 17.5316
45. 5 - 2.4876 46. 526
47. 536 48. 5140.0116 Solve each equation. Unless otherwise specified, give irrational solutions as decimals
49. 5ln 36 50.  ∅
correct to the nearest thousandth.
51. 56.9596 35. 16 x + 4 = 83x - 2 36. 4x = 12 37. 32x - 5 = 13

52. A, C, D 38. 2x + 3 = 5x 39. 6 x + 3 = 4x 40. e x - 1 = 4


41. e2 - x = 12 42. 2e5x + 2 = 8 43. 10e3x - 7 = 5
54.  E F
e16
53. 5e13/3 6
46. e8x # e2x = e20
5
4
210 - 7
44. 5x + 2 = 22x - 1 45. 6 x - 3 = 34x + 1
55. U V 56. 5e3 6
47. e6x # e x = e21
2
48. 10011.022x/4 = 200 49. 2e2x - 5e x - 3 = 0
57. 536 58. 5{66 (Give exact form.)
59. E - 3 , 5 F
4
60. 566 1 x
50. a b + 2 = 0 51. 411.062x + 2 = 8
2
61. E 1, F
10
3 62. 546
52. Concept Check  Which one or more of the following choices is the solution set of
63. 526 64. 536
5x = 9?
65. 5 - 36
log 9 ln 9
66. n = a1eS/a - 12 A.  5log5 96    B.  5log9 56    C.  e f     D.  e f
I
log 5 ln 5
67. I0 = 10 d/10
68. x = 10 D/100 - 2
Solve each equation. Give solutions in exact form.
69. 51.3156
53. 3 ln x = 13 54. ln 5x = 16
55. log 12x + 72 = 0.25 56. ln x + ln x 3 = 12
57. log2 1x 3 + 52 = 5 58. log3 1x 2 - 92 = 3
59. log4 313x + 121x - 424 = 2 60. ln eln x - ln 1x - 42 = ln 3
61. log x + log 113 - 3x2 = 1 62. log7 13x + 22 - log7 1x - 22 = 1
1
63. ln 16x2 - ln 1x + 12 = ln 4 64. log16 2x + 1 =
4
n
65. ln 3ln e -x 4 = ln 3 66. S = a ln a1 + b, for n
a
I
67. d = 10 log , for I0 68. D = 200 + 100 log x, for x
I0

69. Use a graphing calculator to solve the equation e x = 4 - ln x. Give solution(s) to


the nearest thousandth.

Copyright Pearson. All Rights Reserved.

M04_LHSD7659_12_AIE_C04_pp405-496.indd 492 06/10/15 6:48 PM


NOT FOR SALE
chapter 4  Review Exercises 493

70. 89 decibels is about twice as Solve each problem.


loud as 86 decibels. This is a
100% increase. 70. (Modeling) Decibel Levels  Decibel rating of the loudness of a sound is modeled by
71. (a) 1,000,000I0 I
(b)  158,500,000I0 d = 10 log ,
I0
(c)  158.5 times greater
72. (a) 199,500,000I0 where I is the intensity of a particular sound, and I0 is the intensity of a very faint
(b) 12,600,000I0 threshold sound. A few years ago, there was a controversy about a proposed gov-
(c) The 1906 earthquake ernment limit on factory noise. One group wanted a maximum of 89 decibels,
had a magnitude almost while another group wanted 86. Find the percent by which the 89-decibel intensity
16 times greater than the exceeds that for 86 decibels.
1989 earthquake.
71. Earthquake Intensity  The magnitude of an earthquake, measured on the Richter
73. 3.0% 74.  4.0 yr I
scale, is log I0 , where I is the amplitude registered on a seismograph 100 km from
75. $20,363.38
76. $17,458.04 the epicenter of the earthquake, and I0 is the amplitude of an earthquake of a certain
(small) size. On August 24, 2014, the Napa Valley in California was shaken by an
77. 17.3 yr
earthquake that measured 6.0 on the Richter scale.
78. (a)  For t = x,
A(x) = x2 − x + 350 (a) Express this reading in terms of I0.
500 (b) O
 n April 1, 2014, a quake measuring 8.2 on the Richter scale struck off the coast
of Chile. It was the largest earthquake in 2014. Express the magnitude of an 8.2
reading in terms of I0 to the nearest hundred thousand.
(c) H
 ow much greater than the force of the 6.0 earthquake was the force of the earth-
0 10 quake that measured 8.2?
0

(b)  72. Earthquake Intensity  The San Francisco earthquake of 1906 had a Richter scale
For t = x,
A(x) = 350 log (x + 1) rating of 8.3.
500 (a) E
 xpress the magnitude of this earthquake in terms of I0 to the nearest hundred
thousand.
(b) In 1989, the San Francisco region experienced an earthquake with a Richter
scale rating of 7.1. Express the magnitude of this earthquake in terms of I0 to the
0 10 nearest hundred thousand.
0
(c) Compare the magnitudes of the two San Francisco earthquakes discussed in
(c)  For t = x,
parts (a) and (b).
A(x) = 350 (0.75)x
500 73. Interest Rate  What annual interest rate, to the nearest tenth, will produce $4700 if
$3500 is left at interest compounded annually for 10 yr?
74. Growth of an Account  Find the number of years (to the nearest tenth) needed for
$48,000 to become $53,647 at 2.8% interest compounded semiannually.
0 10
0
75. Growth of an Account  Manuel deposits $10,000 for 12 yr in an account paying 3%
(d)  For t = x, interest compounded annually. He then puts this total amount on deposit in another
A(x) = 100 (0.95)x account paying 4% interest compounded semiannually for another 9 yr. Find the
500 total amount on deposit after the entire 21-yr period.
76. Growth of an Account  Anne deposits $12,000 for 8 yr in an account paying 2.5%
interest compounded annually. She then leaves the money alone with no further
deposits at 3% interest compounded annually for an additional 6 yr. Find the total
0 10
0 amount on deposit after the entire 14-yr period.
Function (c) best describes A1t2.
77. Cost from Inflation  Suppose the inflation rate is 4%. Use the formula for continu-
ous compounding to find the number of years, to the nearest tenth, for a $1 item to
cost $2.
78. (Modeling) Drug Level in the Bloodstream  After a medical drug is injected
directly into the bloodstream, it is gradually eliminated from the body. Graph the
following functions on the interval 30, 104. Use 30, 5004 for the range of A1t2.
Determine the function that best models the amount A1t2 (in milligrams) of a drug
remaining in the body after t hours if 350 mg were initially injected.
(a)  A1t2 = t 2 - t + 350 (b)  A1t2 = 350 log1t + 12
(c)  A1t2 = 35010.752t (d)  A1t2 = 10010.952t

Copyright Pearson. All Rights Reserved.

M04_LHSD7659_12_AIE_C04_pp405-496.indd 493 06/10/15 6:48 PM


NOT FOR SALE
494 Chapter 4  Inverse, Exponential, and Logarithmic Functions

79. 2016 79. (Modeling) Chicago Cubs’ Payroll  The table shows the total payroll (in millions
80. (a)  See the graph in part (c). of dollars) of the Chicago Cubs baseball team for the years 2010–2014.
(b) An exponential function
best describes the data.
(c) Answers will vary. Total Payroll
One answer is Year (millions of dollars)
ƒ1x2 = 973,716(1.395)x. 2010 145.4
1.5 × 109 2011 134.3
2012 111.0
2013 107.4
0 25 2014  92.7
−108
Source: www.baseballprospectus.com/
(d)  7,797,000,000 compensation
81. (a) $15,207
(b) $10,716
Letting ƒ1x2 represent the total payroll and x represent the number of years since
(c) $4491
(d)  They are the same. 2010, we find that the function
ƒ1x2 = 146.02e - 0.112x
models the data quite well. According to this function, when will the total payroll
halve its 2010 value?
80. (Modeling) Transistors on Computer Chips  Computing power has increased dra-
matically as a result of the ability to place an increasing number of transistors on
a single processor chip. The table lists the number of transistors on some popular
computer chips made by Intel.

Year Chip Transistors


1989 486DX 1,200,000
1994 Pentium 3,300,000
2000 Pentium 4 42,000,000
2006 Core 2 Duo 291,000,000
2008 Core 2 Quad 820,000,000
2010 Core (2nd gen.) 1,160,000,000
2012 Core (3rd gen.) 1,400,000,000
Source: Intel.

 ake a scatter diagram of the data. Let the x-axis represent the year, where x = 0
(a) M
corresponds to 1989, and let the y-axis represent the number of transistors.
(b) D
 ecide whether a linear, a logarithmic, or an exponential function best describes
the data.
(c) D
 etermine a function ƒ that approximates these data. Plot ƒ and the data on the
same coordinate axes.
(d) A
 ssuming that this trend continues, use ƒ to estimate the number of transistors
on a chip, to the nearest million, in the year 2016.
81. Financial Planning  The traditional IRA (individual retirement account) is a com-
mon tax-deferred saving plan in the United States. Earned income deposited into an
IRA is not taxed in the current year, and no taxes are incurred on the interest paid
in subsequent years. However, when the money is withdrawn from the account after
1
age 59 2 , taxes must be paid on the entire amount withdrawn.
Suppose we deposited $5000 of earned income into an IRA, we can earn an
annual interest rate of 4%, and we are in a 25% tax bracket. (Note: Interest rates and
tax brackets are subject to change over time, but some assumptions must be made
to evaluate the investment.) Also, suppose that we deposit the $5000 at age 25 and
withdraw it at age 60, and that interest is compounded continuously.

Copyright Pearson. All Rights Reserved.

M04_LHSD7659_12_AIE_C04_pp405-496.indd 494 06/10/15 6:48 PM


NOT FOR SALE
chapter 4  Test 495

82. (a)  log412x 2 - x2 = (a) How much money will remain after we pay the taxes at age 60?

ln 12x 2 - x2 (b) S
 uppose that instead of depositing the money into an IRA, we pay taxes on
ln 4
the money and the annual interest. How much money will we have at age 60?
(b) ln (2x2 − x)
y= (Note: We effectively start with $3750 (75% of $5000), and the money earns 3%
ln 4
2.5 (75% of 4%) interest after taxes.)
(c) To the nearest dollar, how much additional money will we earn with the IRA?
−2.5 2.5
(d) Suppose we pay taxes on the original $5000 but are then able to earn 4% in a
tax-free investment. Compare the balance at age 60 with the IRA balance.
−5 82. Consider ƒ1x2 = log4 12x 2 - x2.
1
(c)  A - 2 , 0 B , 11, 02 (a) Use the change-of-base theorem with base e to write log4 12x 2 - x2 in a suitable
1 form to graph with a calculator.
(d)  x = 0, x = 2
(e) There is no y-intercept  raph the function using a graphing calculator. Use the window 3 - 2.5, 2.54 by
(b) G
because the domain of 3 - 5, 2.54.
a logarithm must be a (c) What are the x-intercepts?
positive number, and the (d) Give the equations of the vertical asymptotes.
argument 2x 2 - x is
(e) Why is there no y-intercept?
positive for 1- ∞, 02 ´ A 12 , ∞ B
only.

Chapter 4 Test
3
[4.1] 1. Consider the function ƒ1x2 = 22x - 7.
1. (a)  1- ∞, ∞2; 1- ∞, ∞2 (a) What are the domain and range of ƒ?
(b) The graph is a stretched
3 (b) Explain why ƒ - 1 exists.
translation of y = 2 x,
which passes the horizon- (c) Write an equation for ƒ - 11x2.
tal line test and is thus a (d) What are the domain and range of ƒ - 1?
one-to-one function.
x3 + 7 (e) Graph both ƒ and ƒ - 1. How are the two graphs related with respect to the line
(c)  ƒ - 11x2 = 2 y = x?
(d)  1- ∞, ∞2; 1- ∞, ∞2
(e)  3
2. Match each equation with its graph.
y –1 x +7 f (x) =
2 1 x
10 (a) y = log1/3 x  (b) y = e x  (c)  y = ln x  (d)  y = a b
3
f(x) = 2x – 7 3
–10 0 x
10 A. y B. C.
y y D. y

–10

The graphs are reflections


of each other across the x x x x
0 0 0 0
line y = x.
[4.2, 4.3]
2. (a) B (b) A
(c) C (d) D 1 2x - 3
3. Solve a b = 16 x + 1.
3. E 2 F 8
1

3
4. (a)  log4 8 = 2
4. (a) Write 43/2 = 8 in logarithmic form.
(b)  82/3 = 4 2
(b) Write log8 4 = 3 in exponential form.
[4.1–4.3] x
5. y 5. Graph ƒ1x2 = A 12 B and g1x2 = log1/2 x on the same axes. What is their relationship?
4 6. Use properties of logarithms to rewrite the expression. Assume all variables repre-
()
x
f(x) = 12
1 sent positive real numbers.
x
0 1 4 4
g(x) = log1/2 x
x 2 2y
log7
z3
They are inverses.
Copyright Pearson. All Rights Reserved.

M04_LHSD7659_12_AIE_C04_pp405-496.indd 495 06/10/15 6:48 PM


NOT FOR SALE
496 Chapter 4  Inverse, Exponential, and Logarithmic Functions

[4.3] Use a calculator to find an approximation to four decimal places for each logarithm.
1
6. 2 log7 x + 4 log7 y - 3 log7 z
7. log 2388 8. ln 2388  
9. log9 13
[4.4] 10. Solve x 2/3 = 25.
7. 3.3780 8.  7.7782
9. 1.1674
Solve each equation. Give irrational solutions as decimals correct to the nearest thou-
[4.2] sandth.
10. 5{1256
11. 12x = 1 12. 9 x = 4 13. 16 2x + 1 = 83x
11. 506
14. 2x + 1 = 3x - 4 15. e0.4x = 4x - 2
[4.5]
16. 2e2x - 5ex + 3 = 0  (Give both exact and approximate values.)
12. 50.6316

[4.2]
Solve each equation. Give solutions in exact form.
13. 546
9
[4.5] 17. logx = 2 18. log2 31x - 421x - 224 = 3
16
14. 512.5486 1
19. log2 x + log2 1x + 22 = 3 20. ln x - 4 ln 3 = ln x
15. 52.8116 5
16.  E 0, ln 2 F ; 50, 0.4056
3
21. log3 1x + 12 - log3 1x - 32 = 2
22. A friend is taking another mathematics course and says, “I have no idea what an
[4.3]
expression like log5 27 really means.” Write an explanation of what it means, and
17.  E 4 F
3
tell how we can find an approximation for it with a calculator.

[4.5]
18. 50, 66 Solve each problem.
19. 526 23. (Modeling) Skydiver Fall Speed  A skydiver in free fall travels at a speed modeled by
2 0.  ∅
v1t2 = 17611 - e -0.18t2
21.  E 2 F
7

feet per second after t seconds. How long, to the nearest second, will it take for the
[4.4] skydiver to attain a speed of 147 ft per sec (100 mph)?
22. The expression log5 27 repre-
sents the exponent to which 5 24. Growth of an Account  How many years, to the nearest tenth, will be needed for
must be raised in order to $5000 to increase to $18,000 at 3.0% annual interest compounded (a) monthly
obtain 27. To approximate (b) continuously?
it with a calculator, use the
25. Tripling Time For any amount of money invested at 2.8% annual interest com-
change-of-base theorem.
pounded continuously, how long, to the nearest tenth of a year, will it take to triple?
log 27
log5 27 = ≈ 2.0478 26. (Modeling) Radioactive Decay  The amount of a certain radioactive material, in
log 5 grams, present after t days is modeled by
[4.6] A1t2 = 600e -0.05t.
23. 10 sec
(a) Find the amount present after 12 days, to the nearest tenth of a gram.
24. (a)  42.8 yr (b)  42.7 yr
25. 39.2 yr (b) Find the half-life of the material, to the nearest tenth of a day.
26. (a)  329.3 g (b)  13.9 days

Copyright Pearson. All Rights Reserved.

M04_LHSD7659_12_AIE_C04_pp405-496.indd 496 06/10/15 6:48 PM

You might also like